You are on page 1of 175

CHEMISTRY

Some Basic Concept of Chemistry

1. The density of 1 M solution of a compound (a) 1.25 g


'X' is 1.25 g mL−1. The correct option for the (b) 1.32 g
molality of solution is (Molar mass of (c) 3.65 g
compound X = 85 g): (2023) (d) 9.50 g
(a) 0.705 m 7. An organic compound contains 78% (by wt.)
(b) 1.208 carbon and remaining percentage of
(c) 1.165 m hydrogen. The right option for the empirical
(d) 0.585 m formula of this compound is : [Atomic wt. of
2. The right option for the mass of CO2 C is 12, H is 1] (2021)
produced by heating 20 g of 20% pure (a) CH2
limestone is (b) CH3
(Atomic mass of Ca = 40) (2023) (c) CH4
1200K (d) CH
[CaCO3 → CaO + CO2 ]
8. Which one of the followings has maximum
(a) 1.76 g number of atoms? (2020)
(b) 2.64 g (a) 1 g of Mg(s) [Atomic mass of Mg = 24]
(c) 1.32 g (b) 1 g of O2(g) [Atomic mass of O = 16]
(d) 1.12 g (c) 1 g of Li(s) [Atomic mass of Li = 7]
3. The density of the solution is 2.15 g mL-1, 1 g of Ag(s) [Atomic mass of Ag = 108]
then mass of 2.5 mL solution in correct
9. One mole of carbon atom weighs 12g, the
significant figures is: (2022) number of atoms in it is equal to.
(a) 53.75 g (Mass of carbon-12 is 1.9926 × 10–23 g)
(b) 5375 × 10-3 g (2020 Covid Re-NEET)
(c) 5.4 g (a) 6.022 × 10 22
(d) 5.38 g (b) 12 × 1022
4. What fraction of Fe exists as Fe(III) in (c) 6.022 × 1023
Fe0.96O? (d) 12 × 1023
(Consider Fe0.96O to be made up of Fe(II) and 10. The number of moles of hydrogen molecules
Fe(III) only) (2022) required to produce 20 moles of ammonia
(a) 1/20 through Haber’s process is : (2019)
(b) 1/12 (a) 10
(c) 0.08 (b) 20
(d) 1/16 (c) 30
5. In one molal solution that contains 0.5 mole (d) 40
of a solution, there is (2022) 11. A mixture of 2.3 g formic acid and 4.5 g
(a) 500 mL of solvent oxalic acid is treated with conc. H2SO4. The
(b) 500 g of solvent evolved gaseous mixture is passed through
(c) 100 mL of solvent KOH pellets. Weight (in g) of the remaining
(d) 1000 g of solvent product at STP will be : (2018)
6. What mass of 95% pure CaCO2 will be (a) 1.4
required to neutralise 50 mL of 0.5 M HCl (b) 3.0
solution according to the following reaction? (c) 4.4
CaCO3(s) + 2HCl(aq) ⟶ CaCl2(aq) + CO2(g) + (d) 2.8
2H2O(𝑙) 12. In which case is number of molecules of
[Calculate upto second place of decimal water maximum? (2018)
point] (2022) (a) 18 mL of water
1
(b) 0.18 g of water (Atomic weight of Mg = 24) (2015 Re)
(c) 10–3 mole of water (a) 96
(d) 0.00224 L of water vapours at 1 atom (b) 60
and 273 K (c) 84
13. A hydrocarbon contains 85.7% of Carbon (d) 75
and 14.3% of Hydrogen. If 42 mg of the 19. When 22.4 litres of H2(g) is mixed with 11.2
compound contains 3.01 × 1020 molecules, litres of Cl2(g), each at STP, the moles of
the molecular formula of the compound will HCl(g) formed is equal to : (2014)
be : (2017-Gujarat) (a) 2 mole of HCl(g)
(a) C2H4 (b) 0.5 mol of HCl(g)
(b) C3H6 (c) 1.2 mole of HCl(g)
(c) C6H12 (d) 1 mol of HCl(g)
(d) C12H24
20. 1.0 g of magnesium is burnt with 0.56 g O2
14. Suppose the elements X and Y combine to in a closed vessel. Which reactant is left in
form two compounds XY2 and X3Y2. When excess and how much? (2014)
0.1 mole of XY2 weighs 10 g and 0.05 mole (a) O2, 0.16 g
of X3Y2 weighs 9 g, the atomic weights of X (b) Mg, 0.44 g
and Y are : (2016-II) (c) O2, 0.28 g
(a) 20, 30 (d) Mg, 0.16 g
(b) 30, 20
(c) 40, 30 21. Equal masses of H2, O2 and methane have
(d) 60, 40 been taken in a container of volume V at
temperature 27°C in identical conditions.
15. The number of water molecules is The ratio of the volumes of gases H2 : O2 :
maximum in : (2015 Re) methane would be : (2014)
(a) 18 moles of water (a) 8 : 16 : 1
(b) 18 molecules of water (b) 16 : 8 : 1
(c) 1.8 gram of water (c) 16 : 1 : 2
(d) 18 gram of water (d) 8 : 1 : 2
16. If Avogadro number NA, is changed from 22. 6.02 × 1020 molecules of urea are present in
6.022 × 1023 mol–1, this would change : 100 mL of its solution. The concentration of
(2015 Re) solution is : (2013)
(a) The ratio of elements to each other in a (a) 0.02 M
compound (b) 0.01 M
(b) The definition of mass in units of grams (c) 0.001 M
(c) The mass of one mole of carbon (d) 0.1 M
(d) The ratio of chemical species to each
23. An excess of AgNO3 is added to 100 mL of a
other in a balanced equation
0.01 M solution of dichlorotetraaqua-
17. What is the mass of the precipitate formed
chromium (III) chloride. The number of
when 50 mL of 16.9% solution of AgNO3 is
moles of AgCl precipitated would be :
mixed with 50 mL of 5.8% NaCl solution?
(2013)
(Ag = 107.8, N = 14, O = 16, a = 23, Cl =
(a) 0.001
35.5) (2015 Re)
(b) 0.002
(a) 3.5 g
(c) 0.003
(b) 7 g
(d) 0.01
(c) 14 g
(d) 28 g
18. 20.0 g of a magnesium carbonate sample
decomposes on heating to give carbon
dioxide and 8.0 g magnesium oxide. What
will be the percentage purity of magnesium
carbonate in the sample?

2
Answer Key
S1. Ans. (d) S13. Ans. (c)

S2. Ans. (a) S14. Ans. (c)

S3. Ans. (c) S15. Ans. (a)

S4. Ans. (b) S16. Ans. (c)

S5. Ans. (b) S17. Ans. (b)

S6. Ans. (b) S18. Ans. (c)

S7. Ans. (b) S19. Ans. (d)

S8. Ans. (c) S20. Ans. (d)

S9. Ans. (c) S21. Ans. (c)

S10. Ans. (c) S22. Ans. (b)

S11. Ans. (d) S23. Ans. (a)

S12. Ans. (a)

3
Solutions
S1. Ans.(d) m=
Moles of solute
Mass of solvent (in kg)
1000×M
m= 0.5
1000×d−MMw l= Mass of solvent (in kg)
1000×1
m= 1000×1.25−1×85 ∴ Mass of solvent (in kg) = 0.5
1000
m= = 0.858 = 500 g
1165

S2. Ans.(a) S6. Ans.(b)

Weight of impure limestone = 20 g Let m gram mass of CaCO3 is required


95
Weight of pure limestone (CaCO3) = 20% Pure CaCO3 in m gram = 100
× m
of 20 g 95 m
Moles of CaCO3 = 100
× 100
20
= 100
× 20
Moles of HCl required = 2 × moles of
=4g CaCO3
4
nCaCO3 = 100
= 0.04 =2×
95 m
× 100
100
nCO2 = 0.04 95 m 50
2× × = × 0.5
100 100 1000
WCO2 = 0.04 × 44
m = 1.315 g ≈ 1.32 g
= 1.76 g S7. Ans.(b)
S3. Ans.(c)
Mass = Volume × Density Element % At % Simplest
Weight _______ Ratio
= 2.5 × 2.15 At
weight
= 5.375 g
Since 2.5 has two significant figures, so C 78 12 6.5 1
the mas of solution is correct significant H 22 1 22 3
figures will be 5.4 g. Empirical formula of this compound is
S4. Ans.(b) CH3.
Fe0.96O S8. Ans.(c)
Let Fe(II) present in Fe0.96O = x 1
(a) Number of Mg atoms = × 𝑁𝐴
Fe(III) present = (0.96 – x) 1
24
= × 6.022 × 1023 atm
Total charge on Fe = 2x + (0.96 – x)3 24
1
(b) Number of O atoms = × 𝑁𝐴
Total charge on O = −2 32
1
2x + (0.96 – x)3 = 2 = 32
× 2 × 6.022 × 1023 atm
1
2x + 2.88 – 3x = 2 (c) Number of Li atoms = 7 × 𝑁𝐴
1
−x = −0.88 = 7 × 6.022 × 1023 atm
1
x = 0.88 (d) Number of Ag atoms = 108 × 𝑁𝐴
1
Fe2+ = 0.88, Fe3+ = 0.08 = 108 × 6.022 × 1023 atm

Fraction of Fe3+ =
0.08
= 1/12 Hence, 1 g lithium has the largest
0.96

S5. Ans.(b) number of atoms.

Molality is the moles of solute dissolved S9. Ans.(c)


per kg of solvent therefore 500 g, 1 No of atom in 12 g carbon
molal solution contains 0.5 of solute, as
4
= 12 ÷ (1.9926 × 10−23 ) empirical weight = 14
Thus Number of atoms in 1 mole carbon 3.01×1020
= 𝑁𝑜. 𝑜𝑓 𝑚𝑜𝑙𝑒𝑠 =
42×10−3
6.022×1023 𝑀
= 6.022 × 1023 atoms 1 42×10−3
× 10−3 =
2 𝑀
S10. Ans.(c)
M = 84
Haber’s process 84
∴ Atomicity = =6
14
𝑁2 (𝑔) + 3𝐻2 (𝑔) ⇌ 2𝑁𝐻3 (𝑔)
Molecular formula = C6H12.
20 moles need to be produced
2 moles of 𝑁𝐻3 → 3 𝑚𝑜𝑙𝑒𝑠 𝑜𝑓 𝐻2 S14. Ans.(c)
3×20
Hence 20 moles of 𝑁𝐻3 → = 30 moles For XY2, let atomic weight of X = Ax and
2
of Y = Ay
of H2 10
So, 𝑛𝑥𝑦 2 = 0.1 = 𝐴
𝑥 +2𝐴𝑦
S11. Ans.(d)
Ax + 2Ay = 100 …(i)
𝐶𝑜𝑛𝑐.𝐻2 𝑆𝑂4
𝐻𝐶𝑂𝑂𝐻 → 𝐶𝑂(𝑔) + 𝐻2 𝑂(𝑙) Similarly for X3 Y2,
1 1
2.3g or (20 𝑚𝑜𝑙) 20 𝑚𝑜𝑙 3Ax + 2Ay = 180 …(ii)
On solving (i) and (ii)
𝐶𝑜𝑛𝑐.𝐻2 𝑆𝑂4 Ax = 40 and Ay = 30
𝐶𝑂𝑂𝐻 → 𝐶𝑂(𝑔) + 𝐶𝑂2 (𝑔) + 𝐻2 𝑂(𝑙)
| S15. Ans.(a)

𝐶𝑂𝑂𝐻
1
𝑚𝑜𝑙
1
𝑚𝑜𝑙 (a) 18 mols of water will contain
20 20 = 18 × 6.022 × 1023 molecules of H2O
1
4.5 𝑜𝑟 [20 𝑚𝑜𝑙] 18 molecules
1.8
(c) 18 = 0.1 mole will contain
Gaseous mixture formed is CO and CO .
When it is passed through KOH, only = 0.1 × 5.022 × 1023 molecules of
CO2 is absorbed. So the remaining gas H 2O
18
is CO. KOH pellets absorbs all CO2, H2O (d) 𝑔 = 1 mole = 1 × 6.022 × 1023
18
is absorbed by H2SO4 thus CO is molecules of H2O
remaining product.
So, maximum number of molecules is
So, weight of remaining gaseous product
2 present in 18 moles of H2O.
CO is 20 × 28 = 2.8𝑔
So, the correct option is (d) S16. Ans.(c)
Avogadro’s number 6.022 × 1023 is
S12. Ans.(a)
ideally the mass of number of atoms
(a) Mass of water = 18 × 1 = 18 g
Molecules of water = 𝑚𝑜𝑙𝑒 × 𝑁𝐴 = present in 1 mole that is 12 grams of C.
18
𝑁
18 𝐴
= 1 𝑁𝐴 If we change the Avogadro’s number it
0.18
(b) Molecules of water 𝑚𝑜𝑙𝑒 × 𝑁𝐴 = 𝑁 will directly change the mass of 1 mole
18 𝐴
−2
= 10 𝑁𝐴 that is 12 g of C.
(c) Molecules of water 𝑚𝑜𝑙𝑒 × 𝑁𝐴 =
10−3 𝑁𝐴 S17. Ans.(b)
(d) Moles of water = 𝑚𝑜𝑙𝑒 × 𝑁𝐴 = 10−4 𝑁𝐴 Molecular weight of AgNO3 = 170
Molecular weight of NaCl = 58.5
S13. Ans.(c)
1. 16.9% solution of AgNO3 means 16.9
% Moles Relative moles
g of AgNO3 in 100 mL of solution
C 85.7 85.7
= 7.14 1 So, 8.45 g of AgNO3 in 50 mL of
12
solution.
H 14.3 14.3
= 14.3 2
1
Hence, empirical formula = CH2.
5
2. 5.8% solution of NaCl means 5.8 g of Final (0.0416–2×0.0175) 0 2×0.0175
NaCl is in 100 mL solution. So, in 0
mL = 2.9 g NaCl = 0.0066 moles (O2 is limiting reagent)
𝐴𝑔𝑁𝑂3 + 𝑁𝑎𝐶𝑙 → 𝐴𝑔𝐶𝑙 + 𝑁𝑎𝑁𝑂3
8.45 2.9
∴ Mass of Mg left in excess
Initial Mole : 0 0
170 58.5
= 0.0066 × 24 = 0.16 g
= 0.049 = 0.049 0 0
Final mole : 0 0 0.049 0.049 S21. Ans.(c)
Mass of AgCl precipitated = 0.049 mole According to Avogadro’s principle, ratio
= 0.049 × 143.3 of volume of gases will be equal to the
= 7.02 gm ≃ 7 gm
ratio of their number of moles
S18. Ans.(c) 𝑤
mole = 𝑀𝑤
𝑀𝑔𝐶𝑂3 → 𝑀𝑔𝑂 + 𝐶𝑂2 (𝑔)
↓ ↓ 𝑛𝐻 2 ∶ 𝑛𝑂 2 ∶ 𝑛𝐶𝐻 4
Mg→ 84 g 40 g
𝑤 𝑤 𝑤
∶ ∶ ⇒ 16 : 1 : 2
According to question 2 32 16
84 g MgCO3 gives = 40 g MgO S22. Ans.(b)
40
1 g MgCO3 gives = 6.02 × 1023 number of molecules
84
40
20 g MgCO3 gives = 84 × 20 = 1 mole
= 9.52 g of MgO
6.02 × 1020 = 0.001 mole
But according to question yield of MgO
is = 8 g × 1000
8 𝑉(𝑚𝐿)
% of purity = 9.52
× 100 = 84%
0.001
= 100
× 1000 ⇒ 0.01 M
S19. Ans.(d)
1 mole = 22.4 litres at S.T.P. S23. Ans.(a)

22.4 11.2
Molarity of solution of dichlorotetra-
𝑛𝐻 2 = = 1 𝑚𝑜𝑙𝑒; 𝑛𝐶𝑙 2 = = 0.5 𝑚𝑜𝑙 aquachromium (III) chloride = 0.01 M.
22.4 22.4

Reaction is as, Volume of solution of dichlorotetra-


aquachromium (III) chloride = 100 ml.
𝐻2 (𝑔) + 𝐶𝑙2 (𝑔) → 2𝐻𝐶𝑙(𝑔)
The formula of dichlorotetra-
Initial 1 mol 0.5 mol 0
aquachromium (III) chloride is
Final (1– 0.5) (0.5 – 0.05) 2 × 0.5 [𝐶𝑟(𝐻2 𝑂)𝐶𝑙2 ]𝐶𝑙.
= 0.5 mol = 0 mol 1 mol On ionisation,
Here, Cl2 is limiting reagent. So, 1 mole [𝐶𝑟(𝐻2 𝑂)𝐶𝑙2 ]𝐶𝑙 → [𝐶𝑟(𝐻2 𝑂)𝐶𝑙2 ]+ + 𝐶𝑙 −
of HCl (g) is formed. Initial 100 × 0.01 0 0
S20. Ans.(d) Final 0 1 mol 1 mol
𝑛𝑀𝑔 =
1
= 0.0416 𝑚𝑜𝑙𝑒𝑠 So, 1 mol of Cl– ions will react with 1
24
mol of AgNO3 mole of [𝐶𝑟(𝐻2 𝑂)𝐶𝑙2 ]𝐶𝑙 0.1
0.56 M 100 ml solution is,
𝑛𝑂 2 = 32
= 0.0175 𝑚𝑜𝑙𝑒𝑠
No. of moles = Molarity × Volume
The balanced chemical equation : = 0.01 × 0.1 = 0.001 mol
1 Hence, 0.001 mol of Cl– ions will react
𝑀𝑔 + 𝑂
2 2
→ 𝑀𝑔𝑂 with 0.001 mole of AgNO3.
Initial 0.0416 0.0175 0 So, number of moles of AgCl formed is
0.001 mol.
moles moles
6
Chapter 3
Classification of Elements & Periodicity in Properties

1. Which of the following is correctly matched? enthalpies of X and Y are 3069 kJ mol-1 and
(2023) 1145 kJ mol-1, respectively. Then correct
(a) Basic oxides – In2O3, K2O, SnO2 statement is: (2022)
(b) Neutral oxides – CO, NO2, N2O (a) Both X and Y are alkaline earth metals
(c) Acidic oxides – Mn2O7, SO2, TeO3 (b) X is an alkali metal and Y is an alkaline
(d) Amphoteric oxides – BeO, Ga2O3, GeO earth metal
2. The correct sequence given below (c) X is an alkaline earth metal and Y is an
containing neutral, acidic, basic and alkali metal
amphoteric oxide each, respectively, is: (d) Both X and Y are alkali metals
(2023) 7. The IUPAC name of an element with atomic
(a) NO, ZnO, CO2, CaO number 119 is (2022)
(b) ZnO, NO, CaO, CO2 (a) ununennium
(c) NO, CO2, ZnO, CaO (b) unnilennium
(d) NO, CO2, CaO, ZnO (c) unununnium
3. The correct order of first ionization enthalpy (d) ununoctium
for the given four elements is: (2022) 8. Gadolinium has a low value of third
(a) C < F < N < O ionisation enthalpy because of (2022)
(b) C < N < F < O
(c) C < N < O < F (b) high exchange enthalpy
(d) C < O < N < F (c) high electronegativity
4. Decreases in size from left to right in (d) high basic character
actinoid series is greater and gradual than 9. From the following pairs of ions which one
that in lanthanoid series due to (2022) is not an iso-electronic pair? (2021)
(a) 5f orbitals have greater shielding effect (a) Na , Mg
+ 2+

(b) 4f orbitals are penultimate (b) Mn2+, Fe3+


(c) 4f orbitals have greater shielding effect (c) Fe2+, Mn2+
(d) 5f orbitals have poor shielding effect (d) O2–, F–
5. Fluorine is a stronger oxidising agent than 10. Identify the incorrect match (2020)
chorine because: Name IUPAC Official Name
(A) F-F bond has a low enthalpy of Unnilunium (i) Mendelevium
A.
dissociation.
B. Unniltrium (ii) Lawrencium
(B) Fluoride ion (F − ) has high hydration
enthalpy. C. Unnilhexium (iii) Seaborgium
(C) Electron gain enthalpy of fluorine is less D. Unununnium (iv) Darmstadtium
negative than chlorine.
(a) B-(ii)
(D) Fluorine has a very small size.
(b) C-(iii)
Choose the most appropriate answer from
(c) D-(iv)
the options given: (2022) (d) A-(i)
(a) B and C only
11. For the second period elements the correct
(b) A and B only
increasing order of first ionization enthalpy
(c) A and C only
is : (2019)
(d) A and D only
(a) Li < Be < B < C < N < C < F < Ne
6. If first ionization enthalpies of elements X (b) Li < B < Be < C < O < N > F < Ne
and Y are 419 kJ mol-1 and 590 kJ mol-1, (c) Li < B < Be < C < N < O < F < Ne
respectively and second ionization (d) Li < Be < B < C < O < N < F < Ne
7
12. The element Z = 114 has been discovered (b) d-electrons in Fe (Z = 26)
recently. It will belong to which of the (c) p-electrons in Ne (Z = 10)
following family group and electronic (d) s-electrons in Mg (Z = 12)
configuration? (2017-Delhi) 16. The species Ar, K+, Ca2+ contain the same
(a) Nitrogen family, [𝑅𝑛]5𝑓 6𝑑10 7𝑠 2 7𝑝6
14
number of electrons. In which order do their
(b) Halogen family, [𝑅𝑛]5𝑓 14 6𝑑10 7𝑠 2 7𝑝5 radii increase? (2015)
(c) Carbo family, [𝑅𝑛]5𝑓 14 6𝑑10 7𝑠 2 7𝑝2 (a) Ca2+ < Ar < K+
(d) Oxygen family, [𝑅𝑛]5𝑓 14 6𝑑10 7𝑠 2 7𝑝4 (b) Ca2+ < K+ < Ar
13. In which of the following options the order (c) K+ < Ar < Ca2+
of arrangement does not agree with the (d) Ar < K+ < Ca2+
variation of property indicated against it? 17. Be2+ is isoelectronic with which of the
(2016-I) following ions? (2014)
(a) Li < Na < K < Rb (increasing metallic (a) Li+
radius) (b) Na+
(b) Al3+ < Mg2+ < Na+ < F– (increasing ionic (c) Mg2+
size) (d) H+
(c) B < C < N < O (increasing first ionization
enthalpy) 18. Which of the following orders of ionic radii
(d) I < Br < Cl < F (increasing electron gain is correctly represented? (2014)
(a) Na > F > O
+ – 2–
enthalpy) (b) O2– > F– > Na+
14. The formation of the oxide ion, O2– (g) from (c) Al3+ > Mg2+ > N3–
oxygen atom requires first and exothermic (d) H– > H+ > H
and then an endothermic step as shown 19. Identify the wrong statement in the
below: following: (2012 Pre)
𝑂(𝑔) + 𝑒 − → 𝑂 (a) Atomic radius of the elements decreases
𝑂− (𝑔) + 𝑒 − → 𝑂2− (𝑔); ∆𝑓 𝐻° = +780 𝑘𝐽 𝑚𝑜𝑙 −1 as one moves across from left to right in
Thus, process of formation of O2– in gas the 2nd period of the periodic table
phase is unfavorable even through O2– is (b) Amongst isoelectronic species, smaller
isoelectronic with neon. It is due to the fact the positive charge on the carbon,
that, smaller is the ionic radius
(a) O– ion has comparatively smaller size (c) Amongst isoelectronic species, greater
than oxygen atom the negative charge on the anion, larger
(b) Oxygen is more electronegative is the ionic radius
(c) Addition of electron in oxygen results in (d) Atomic radius of the elements increases
larger size of the ion as one moves down the first group of the
(d) Electron repulsion outweighs the periodic table
stability gained by achieving noble gas
configuration
15. The number of d-electrons in Fe2+ (Z = 26)
is not equal to the number of electrons in
which one of the following? (2015)
(a) p-electrons in Cl (Z = 17)

8
Answer Key

S1. Ans. (c) S11. Ans. (b)

S2. Ans. (d) S12. Ans. (c)

S3. Ans. (d) S13. Ans. (c/d)

S4. Ans. (d) S14. Ans. (d)

S5. Ans. (b) S15. Ans. (a)

S6. Ans. (b) S16. Ans. (b)

S7. Ans. (a) S17. Ans. (a)

S8. Ans. (b) S18. Ans. (b)

S9. Ans. (c) S19. Ans. (b)

S10. Ans. (c)

9
Solutions
S1. Ans. (c) configuration will be [Xe] 4f7 that is
Mn2O7, SO2, TeO3 are acidic oxides. stable electronic configuration as it will
have high exchange energy, hence less
S2. Ans. (d)
energy will be required to remove 3rd
NO ⟶ Neutral electron.
CaO ⟶ Basic S9. Ans.(c)
CO2 ⟶ Acidic 26 𝐹 → 3𝑑6 4𝑠 2 , 𝐹𝑒 +2 → 3𝑑6 24
ZnO ⟶ Amphoteric
25 𝑀𝑛 → 3𝑑5 4𝑠 2 , 𝑀𝑛+2 → 3𝑑5 23
S3. Ans. (d)
• Generally, on moving left to right in a S10. Ans.(c)
period. First ionization enthalpy of Unununnium is the element that has
elements increases due to increase in Atomic number = 111
effective nuclear charge. IUPAC official name of Unununium:
• Due to more stable half-filled outer Roentgenium
electronic configuration (2s22p3) of N, Thus option (c) is correct.
its first ionization enthalpy is more S11. Ans.(b)
than O.
‘Be’ and ‘N’ have comparatively more
So, correct order of IP is: C < O < N < F
stable valence sub-shell than ‘B’ and ‘O’.
S4. Ans. (d)
Generally Ionisation energies increases
Due to more diffused nature of 5f orbitals across a period.
as compared to 4f orbitals the shielding
effect of 5f is poor, resulting in the Thus, correct increasing order of first
decrease in size from left to right in ionization enthalpy is :
actinoid series which is greater and Li < B < Be < C < O < N < F < Ne
gradual than that in lanthanoid series. S12. Ans.(c)
S5. Ans. (b)
Carbon family: [𝑅𝑛]5𝑓 14 6𝑑10 7𝑠 2 7𝑝2
Fluorine is a stronger oxidising agent
S13. Ans.(c, d)
than chlorine due to
Increasing first ionization enthalpy will
(i) Low dissociation enthalpy of F-F bond
be B < C < O < N.
(ii) High hydration enthalpy of F − ion
Electron gain enthalpy: I < Br < F < Cl
S6. Ans. (b)
S14. Ans.d()
As it can be observed from given data of
question, in case of element 'X' there is There is a lot of repulsion when similar
huge difference between IP1 and IP2 charges approach each other as O– (g)
hence, it will have one electron in and e– are both negatively charged. To
outermost shell and will be alkali metal. add an electron under such situation,
While for 'Y' difference is not that high the force of repulsion is to be overcome
hence, it will be alkaline earth metal. by applying external energy.

S7. Ans. (a) S15. Ans.(a)


IUPAC name of element : 119 : Number of d electrons in Fe2+ (26) = 6
ununennium Number of p electron in Cl (Z = 17) = 11
S8. Ans. (b) Number of s electron in Mg (Z = 12) = 6
Electronic configuration of Gadolinium Number of p electron in Ne = 6
Gd: [Xe] 4f7 5d1 6s2
In case of 3rd ionization enthalpy electron
will be removed from 5d and resultant
10
S16. Ans.(b)
In case of isoelectronic species, radius
decreases with increase in nuclear
charge.
S17. Ans.(a)
Be2+ = 2e–
Li+ = 2e–
Isoelectronic species means ions with
same number of electron.
S18. Ans.(b)
Cations lose electrons and are smaller in
size than the parent atom, whereas
anions add electrons and are larger in
size than the parent atom.
Hence, the order is H– > H > H+.
For isoelectronic species, the ionic radii
decreases with increase in atomic
number i.e., nuclear charge.
Hence, the correct orders are :
O2– > F– > Na and N > Mg > Al
S19. Ans.(b)
Among isoelectronic species the ion with
the maximum positive charge will have
the smallest radius.

11
Chapter 4
Chemical Bonding and Molecular Structure

1. The correct order of dipole moments for 5. The correct order of energies of molecular
molecules NH3, H2S, CH4 and HF is: (2023) orbitals of N2 molecule, is (2023)
(a) CH4 > H2S > NH3 > HF (a) 𝜎1𝑠 < 𝜎 1𝑠 < 𝜎2𝑠 < 𝜎 2𝑠 < 𝜎2𝑝𝑧 <
∗ ∗

(b) H2S > NH3 > HF > CH4 (𝜋2𝑝𝑥 = 𝜋2𝑝𝑦 ) < (𝜋 ∗ 2𝑝𝑥 = 𝜋 ∗ 2𝑝𝑦 ) < 𝜎 ∗ 2𝑝𝑧
(c) NH3 > HF > CH4 > H2S (b) 𝜎1𝑠 < 𝜎 ∗ 1𝑠 < 𝜎2𝑠 < 𝜎 ∗ 2𝑠 < 𝜎2𝑝𝑧 <
(d) HF > NH3 > H2S > CH4 𝜎 ∗ 2𝑝𝑧 < (𝜋2𝑝𝑥 = 𝜋2𝑝𝑦 ) < (𝜋 ∗ 2𝑝𝑥 = 𝜋 ∗ 2𝑝𝑦 )
2. Which one of the following represents all (c) 𝜎1𝑠 < 𝜎 ∗ 1𝑠 < 𝜎2𝑠 < 𝜎 ∗ 2𝑠 < (𝜋2𝑝𝑥 =
isoelectronic species? (2023) 𝜋2𝑝𝑦 ) < (𝜋 ∗ 2𝑝𝑥 = 𝜋 ∗ 2𝑝𝑦 ) < 𝜎2𝑝𝑧 < 𝜎 ∗ 2𝑝𝑧
(a) Na , Cl , O , NO
+ − − +
(d) 𝜎1𝑠 < 𝜎 ∗ 1𝑠 < 𝜎2𝑠 < 𝜎 ∗ 2𝑠 < (π2p𝑥 =
(b) N2O, N2O4, NO+, NO
(c) Na+, Mg2+, O− , F − 𝜋2𝑝𝑦 ) < 𝜎2𝑝𝑧 < (𝜋 ∗ 2𝑝𝑥 = 𝜋 ∗ 2𝑝𝑦 ) < 𝜎 ∗ 2𝑝𝑧
(d) Ca2+, Ar, K+, Cl− 6. Talking stability as the factor, which one of
3. Which one of the following statements is the following represents correct
incorrect related to Molecular Orbital relationship? (2023)
Theory? (2023) (a) Inl3 > Inl
(a) The π antibonding molecular orbital
∗ (b) AlCl > AlCl3
has a node between the nuclei. (c) Tll > Tll3
(b) In the formation of bonding molecular
orbital, the two electron waves of the 7. Intermolecular forces are forces of
bonding atoms reinforce each other. attraction and repulsion between
(c) Molecular orbitals obtained from 2Px interacting particles that will include:
and 2Py orbitals are symmetrical around A. dipole-dipole forces
the bond axis. B. dipole-induced dipole forces
(d) A π-bonding molecular orbital has larger C. hydrogen bonding
electron density above and below the D. covalent bonding
E. dispersion forces
internuclear axis.
Choose the most appropriate answer from
4. Given below are two statements: the options given below: (2023)
Statement I: Hydrated chlorides and (a) A, B, C, D are correct
bromides of Ca, Sr and Ba on heating (b) A, B, C, E are correct
undergo hydrolysis. (c) A, C, D, E are correct
Statement II: Hydrate chlorides and (d) B, C, D, E are correct
bromides of Be and Mg on heating undergo
8. Match List I with List II:
dehydration.
List I List II
In the light of the above statements, choose
(Molecules) (Shape)
the correct answer from the options given
below: (2023) A. NH3 i. Square pyramidal
(a) Statement I is correct but Statement II ClF3 ii. Trigonal bipyramidal
B.
is false.
C. PCl5 iii. Trigonal pyramidal
(b) Statement II is incorrect but Statement
II is true. D. BrF5 iv. T-shape
(c) Both Statement I and Statement II are
Choose the correct answer from the options
true.
given below: (2022)
(d) Both Statement I and Statement II are
(a) A-iii, B-iv, C-i, D-ii
false. (b) A-ii, B-iii, C-iv, D-i
(c) A-iii, B-iv, C-ii, D-i
12
(d) A-iv, B-iii, C-i, D-ii B. SF6 (ii) Trigonal planar
9. The correct order of bond angles in the C. BrF5 (iii) Octahedral
following compounds/species is: (2022)
+ D. BF3 (iv) Trigonal bipyramidal
(a) CO2 < NH3 < H2O < NH4
+ Choose the correct answer from the options
(b) H2O < NH3 < NH4 < CO2 given below.
+
(c) H2O < NH4 < NH3 < CO2 (a) A-ii, B-iii, C-iv, D-i
+ (b) A-iii, B-I, C-iv, D-ii
(d) H2O < NH4 = NH3 < CO2 (c) A-iv, B-iii, C-ii, D-i
10. Given below are two statements: one is (d) A-iv, B-iii, C-I, D-ii
labelled as Assertion (A) and the other is 15. Which of the following molecules is non-
labelled as Reason (R). polar in nature? (2021)
Assertion (A): ICl is more reactive than I2. (a) CH2O
Reason (R): I-Cl bond is weaker than l-l (b) SbCl5
bond. (c) NO2
In the light of the above statements, choose (d) POCl3
the most appropriate answer from the
16. Which of the following set of molecules will
options given below: (2022)
have zero dipole moment? (2020)
(a) Both (A) and (R) are correct and (R) is
(a) Boron trifluoride, hydrogen fluoride,
the correct explanation of (A).
carbon dioxide, 1,3-dichlorobenzene
(b) Both (A) and (R) are correct but (R) is not
(b) Nitrogen trifluoride, beryllium
the correct explanation of (A).
difluoride, water 1, 3-dichlorobenzzene
(c) (A) is correct but (R) is not correct.
(c) Boron trifluoride, beryllium difluoride,
(d) (A) is not correct but (R) is correct.
carbon dioxide, 1,4-dichlorobenzene
11. Amongst the following which one will have (d) Ammonia, beryllium difluoride, water,
maximum 'lone pair – lone pair' electron 1,4-dichlorobenzene
repulsions? (2022) 17. Identify a molecule which does not exist.
(a) ClF3 (2020)
(b) IF5 (a) Li2
(c) SF4 (b) C2
(d) XeF2 (c) O2
12. Which amongst the following is incorrect (d) He2
statement? (2022) 18. How many (i) sp2 hybridised carbon atoms
(a) The bond orders of O+ 2 , O 2 , O−
2 and O2−
2 and (ii)  bonds are present in the following
are 2.5, 2, 1.5 and 1, respectively compound? (2020 Covid Re-NEET)
(b) C2 molecule has four electrons in its two
degenerate π molecular orbitals
(c) H2+ ion has one electron
(d) O+
2 ion is diamagnetic (a) 8, 6
13. BF3 is planar and electron compound. (b) 7, 6
Hybridization and number of electrons (c) 8, 5
around the central atom, respectively are : (d) 7, 5
(2021)
19. The potential energy (y) curve for H2
(a) sp3 and 6
formation as a function of internuclear
(b) sp2 and 6
distance (x) of the H atoms is shown below.
(c) sp2 and 8
(d) sp3 and 4
14. Match List-I with List-II. (2021)
List-I List-II

A. PCl5 (i) Square pyramidal

13
The bond energy of H2 is 26. Which one of the following ions is not
(2020 Covid Re-NEET) tetrahedral in shape? (2017-Gujarat)
(a)
(𝑐−𝑎) (a) [𝑁𝑖𝐶𝑙4 ]2−
2
(𝑏−𝑎) (b) 𝑁𝐻4+
(b) (c) 𝐵𝐹4−
2
(c) (𝑐 − 𝑎) (d) [𝐶𝑢(𝑁𝐻3 )4 ]2+
(d) (𝑏 − 𝑎)
27. Which of the following pair of species is not
20. Identify the wrongly matched pair. iso-structural? (2017-Gujarat)
(2020 Covid Re-NEET) (a) 𝐵𝑟𝑂3− , 𝑋𝑒𝑂3
Molecule Shape or geometry of (b) 𝐼𝐶𝑙4− , 𝑋𝑒𝐹4
molecule (c) 𝐶𝑙𝑂3− , 𝐶𝑂32−
(a) SF6 Octahedral (d) 𝐼𝐵𝑟2− , 𝑋𝑒𝐹2
(b) BeCl2 Linear
28. Which of the following hydrides has the
(c) NH3 Trigonal pyramidal
largest bond angle? (2017-Gujarat)
(d) PCl5 Trigonal planar
(a) H2Se
21. Which of the following diatomic molecular (b) H2S
species has only  bonds according to (c) H2Te
Molecular Orbital Theory? (2019) (d) H2O
(a) O2
29. Which one of the following compounds
(b) N2
shows the presence of intramolecular
(c) C2
hydrogen bond? (2016-II)
(d) Be2
(a) Cellulose
22. Identify the incorrect statement related to (b) Concentrated acid
PCl5 from the following: (2019)
(a) Three equational P–Cl bonds make an (d) HCN
angle of 120° with each other
30. In which of the following molecules, all
(b) Two axial P–Cl bonds make an angle of
atoms are coplanar? (2016-II)
180° with each other
(c) Axial P–Cl bonds are longer than
(a)
equatorial P–Cl bonds
(d) PCl5 molecule is non-reactive
23. Consider the following species: (2018) (b)
CN , CN , NO and CN
+ –

Which one of these will have the highest


bond order?
(a) NO
(b) CN– (c)
(c) CN
(d) CN+
24. Which one of the following pair of species
have the same bond order? (2017-Delhi)
(a) 𝑁2 , 𝑂2− (d)
(b) 𝐶𝑂, 𝑁𝑂
(c) 𝑂2 , 𝑁𝑂+
31. Among the following which one is a wrong
(d) 𝐶𝑁 − , 𝐶𝑂
statement? (2016-II)
25. The species, having bond angles of 120° is (a) SeF4 and CH4 have same shape
(2017-Delhi) (b) I3+ has bent geometry
(a) BCl3 (c) PH5 and BiCl5 do not exist
(b) PH3 (d) p – d bonds are present in SO2
(c) ClF3
(d) NCl3
14
32. The hybridisations of atomic orbitals of (b) 12
nitrogen in 𝑁𝑂2+ , 𝑁𝑂3− and 𝑁𝐻4+ respectively (c) 16
are : (2016-II) (d) 4
(a) sp, sp3 and sp2 38. Which of the following options represents
(b) sp2, sp3 and sp the correct bond order? (2015)
(c) sp, sp2 and sp3 (a) 𝑂2− < 𝑂2 < 𝑂2+
(d) sp2, sp and sp3 (b) 𝑂2− > 𝑂2 < 𝑂2+
33. Which of the following pairs of ions is (c) 𝑂2− < 𝑂2 > 𝑂2+
isoelectronic and isostructural? (d) 𝑂2− > 𝑂2 > 𝑂2+
(2016-II) 39. The correct bond order in the following
(a) 𝐶𝑂32− , 𝑁𝑂3− species is: (2015)
2+ − +
(b) 𝐶𝑙𝑂3− , 𝐶𝑂32− (a) 𝑂2 < 𝑂2 < 𝑂2
(c) 𝑆𝑂32− , 𝑁𝑂3− (b) 𝑂22+ < 𝑂2− < 𝑂2+
(d) 𝐶𝑙𝑂3− , 𝑆𝑂32− (c) 𝑂2− < 𝑂2+ < 𝑂22+
34. Consider the molecules CH4, NH3 and H2O. (d) 𝑂22+ < 𝑂2+ < 𝑂2−
Which of the given statement is false? 40. Which of the following pairs of ions are
(2016-I) isoelectronic and isostructural? (2015)
(a) The H–C–H bond angle in CH4 is larger 2−
(a) 𝑆𝑂3 , 𝑁𝑂3 −

than the H–N–H bond angle in NH3 (b) 𝐶𝑙𝑂3− , 𝑆𝑂32−


(b) The H–C–H bond angle in CH4, the H–N– (c) 𝐶𝑂32− , 𝑆𝑂32−
H bond angle in NH3, and the H–O–H (d) 𝐶𝑙𝑂3− , 𝐶𝑂32−
bond angle in H2O are all greater than
41. Maximum bond angle at nitrogen is present
90°
in which of the following? (2015)
(c) Then H–O–H bond angle in H2O is larger
(a)
than the H–C–H bond angle in CH4
(d) The H–O–H bond angle in H2O is smaller
(c) 𝑁𝑂2
than the H–N–H bond angle in NH3
(d) 𝑁𝑂2−
35. Predict the correct order among the
following: (2016-I) 42. Which of the following molecules has the
(a) Lone pair – bond pair > bond pair – bond maximum dipole moment? (2014)
pair > lone pair – lone pair (a) CH4
(b) Lone pair – lone pair > lone pair – bond (b) NH3
pair > bond pair – bond pair (c) NF3
(c) Lone pair – lone pair > bond pair – bond (d) CO2
pair > lone pair – bond pair 43. Which one of the following species has
(d) Bond pair – bond pair > lone pair – bond planar triangular shape? (2014)
pair > lone pair – lone pair (a) 𝑁𝑂3−

36. Decreasing order of stability of 𝑂2 , 𝑂2− , 𝑂2+ (b) 𝑁𝑂2−


and 𝑂22− is : (2015 RE) (c) 𝐶𝑂2
(a) 𝑂2− > 𝑂22− > 𝑂2+ > 𝑂2 (d) 𝑁3
(b) 𝑂2+ > 𝑂2 > 𝑂2− > 𝑂22−
44. Which of the following organic compounds
(c) 𝑂22− > 𝑂2− > 𝑂2 > 𝑂2+ has same hybridization as its combustion
(d) 𝑂2 > 𝑂2+ > 𝑂22− > 𝑂2− product (CO2)? (2014)
37. The total number of -bond electrons in the (a) Ethyne
following structure is: (2015) (b) Ethene
(c) Ethanol
(d) Ethane
45. Which of the following is a polar molecule?
(2013)
(a) BF3
(a) 8 (b) SF4

15
(c) SiF4
(d) XeF4
46. Which of the following is electron-deficient?
(2013)
(a) (CH3)2
(b) (SiH3)2
(c) (NH3)2
(d) PH3
47. Which of the following is paramagnetic?
(2013)
(a) CO
(b) O−
2
(c) CN
(d) NO+

16
Answer Key

S1. Ans. (d) S25. Ans. (a)

S2. Ans. (d) S26. Ans. (d)

S3. Ans. (c) S27. Ans. (c)

S4. Ans. (d) S28. Ans. (d)

S5. Ans. (d) S29. Ans. (a)

S6. Ans. (c) S30. Ans. (c)

S7. Ans. (b) S31. Ans. (a)

S8. Ans. (c) S32. Ans. (c)

S9. Ans. (b) S33. Ans. (a/d)

S10. Ans. (a) S34. Ans. (c)

S11. Ans. (d) S35. Ans. (b)

S12. Ans. (d) S36. Ans. (b)

S13. Ans. (b) S37. Ans. (a)

S14. Ans. (d) S38. Ans. (a)

S15. Ans. (b) S39. Ans. (c)

S16. Ans. (c) S40. Ans. (b)

S17. Ans. (d) S41. Ans. (a)

S18. Ans. (b) S42. Ans. (b)

S19. Ans. (d) S43. Ans. (a)

S20. Ans. (d) S44. Ans. (a)

S21. Ans. (c) S45. Ans. (b)

S22. Ans. (d) S46. Ans. (c)

S23. Ans. (b) S47. Ans. (b)

S24. Ans. (d)

17
Solutions
S1. Ans.(d) S7. Ans.(b)
HF > NH3 > H2S > CH4 (non-polar) Intermolecular forces means force of
S2. Ans.(d) attraction between two or more
molecules dipole-dipole (attraction
Total numbers electrons are same
between two or more polar molecules).
Cr+2, Ar, K+, Cl− → 20 electrons
Dipole induced dipole (attraction
S3. Ans.(c) between polar and non-polar molecules)
In the formation of BMO, the two electron Hydrogen bonding (it is a special type of
waves of the bonding atoms reinforce dipole-dipole and ion-dipole attraction)
each other due to constructive
Dispersion forces (mainly acts between
interference. Molecular orbitals obtained
non-polar molecules).
from 2Px and 2Py orbitals are
'unsymmetrical' around bond axis. Covalent bonding (acts between atom not
between molecules).
S4. Ans.(d)
S8. Ans.(c)
Hydrated chlorides and Bromides of Ca,
Sr and Ba are ionic so undergo
dehydration after heating. Hydrated
chlorides and Bromides of Be and Mg are
covalent so undergo hydrolysis on
Heating.
NH3, AlCl3, BeCl , CCl PCl
S5. Ans.(d)
Molecular orbital (energy) diagram
sequence of N2 A-iii, B-iv, C-ii, D-i
S9. Ans.(b)
CO2 ⟹ sp2 hybridisation, bond angle =
180°
NH4+ ⟹ sp3 hybridisation, bond angle =
109° 28'
NH3 ⟹ sp3 hybridisation with one lone
pair on central atom, bond angle ≃ 107°
H2O ⟹ sp3 hybridisation with two lone
pairs on central atom, bond angle ≃
104.5°
S10. Ans.(a)
In general, interhalogen compounds are
more reactive than halogens (except
fluorine). This is because X-X' bond in
interhalogens is weaker than X-X bond in
S6. Ans.(c) halogens excepts F-F bond. Therefore I-
Tl+ & I − > Tl+3 & 3I − Cl is more reactive than I2 because of
weaker I-Cl bond then I-I bond.
due to inert pair effect Tl+ is more stable
than Tl+3.

18
S11. Ans.(d) S15. Ans.(b)

sp3d Dipole moment () = 0


Trigonal Non-polar
bipyramidal
S16. Ans.(c)
The given set of molecules have dipole
moment zero. Because dipoles of the
bond cancel each other.

XeF2 having maximum lone pairs, so, it


has maximum 'lone pair – lone pair'
electron repulsions.
S12. Ans.(d)

Due to one unpaired electron in π∗ 2p


molecular orbital, O2 is a paramagnetic S17. Ans.(d)
ion. For He2 molecule
S13. Ans.(b) Electronic configuration is  1s2, *1s2
1
𝐵. 𝑂. = (𝑁𝑏 − 𝑁𝑎 )
2
1
= (2 − 2) = 0
2

The bond order comes out to be zero.


This indicates that there is no bond
formation between 2 He atoms and
sp2, Trigonal planar 6e– around central
hence the He2 molecule does not exist.
atom.
S18. Ans.(b)
S14. Ans.(d)
(i) Number of sp2 hybridised carbon
Hybridisa- L.P. Shape atoms is 7
tion (i) Number of pi bonds is 6

A PCl5 sp3d 0 Trigonal


bipyramidal

B SF6 sp3d2 0 Octahedral

C BrF5 sp3d2 1 Square S19. Ans.(d)


pyramidal Potential energy of two H atoms at
infinite distance = a
D BF3 sp2 0 Trigonal
Potential energy of two H atoms at
planar distance equal to bond length = b
So, the bond energy of H2 = (b – a)

19
S20. Ans.(d) Hence, option (b) should be the right
answer.
S24. Ans.(d)
CN– = 6 + 7 + 1 = 14
CO = 6 + 8 = 14
These two species are isoelectronic and
iso structural in nature. Therefore, they
have both have same bond order.
S25. Ans.(a)
BCl3 having bond angles of 120°.

S21. Ans.(c)
σ1s 2 , σ ∗ 1s 2 , σ2s 2 , σ ∗ 2s 2 , π2p2x = π2p2y S26. Ans.(d)
Double bond in C2 consists of both  [Cu(NH3 )4 ]2+ is not tetrahedral. It is a
bonds because of the presence of four square planar complex.
electrons in two  molecular orbital. In
other molecule a double bond is made
up of a sigma bond and a pi bond.
S22. Ans.(d)
S27. Ans.(c)

(d) False Hence, they both are not isostructural.


Due to longer and thus weaker axial S28. Ans.(d)
bonds, PCl5 is a reactive molecule. O, Se, Se, Te belong to Group 16.
S23. Ans.(b) On moving down the group, size of atom
NO : , (𝜎 ∗ 1𝑠)2 , (𝜎2𝑠)2 , (𝜎 ∗ 2𝑠)2 , (𝜎2𝑝𝑧 )2 , increases.
2 0
(𝜋2𝑝𝑥 )2 = (𝜋2𝑝𝑦 ) , (𝜋 ∗ 2𝑝𝑥 )1 = (𝜋 ∗ 2𝑝𝑦 )
10−5
𝐵𝑂 = = 2.5
2
CN– : (𝜎1𝑠)2 , (𝜎 ∗ 1𝑠)2 , (𝜎2𝑠)2 , (𝜎 ∗ 2𝑠)2 ,
2
(𝜋2𝑝𝑥 )2 = (𝜋2𝑝𝑦 ) , (𝜎2𝑝𝑧 )2 ‘O’ is most electronegative and lone
10−4 pairs lie close to the atom electron
𝐵𝑂 = =3
2 cloud. This causes repulsion in lone
CN : (𝜎1𝑠)2 , (𝜎 ∗ 1𝑠)2 , (𝜎2𝑠)2 , (𝜎 ∗ 2𝑠)2 , pairs of oxygen and bond pairs of
2 hydrogen.
(𝜋2𝑝𝑥 )2 = (𝜋2𝑝𝑦 ) , (𝜎2𝑝𝑧 )1
∴ Angle maximum due to l.p. – b.p.
9−4
𝐵𝑂 = 2
= 2.5 repulsion.
CN+ : (𝜎1𝑠)2 , (𝜎 ∗ 2
1𝑠) , (𝜎2𝑠)2 , (𝜎 ∗ 2
2𝑠) ,
2
(𝜋2𝑝𝑥 )2 = (𝜋2𝑝𝑦 )
8−4
𝐵𝑂 = =2
2

20
Te has maximum size: lone pair lie far 1
𝑁𝐻4+ = 2 [5 + 4 − 1]
away from the atom electron cloud.
Lone pair – bond pair repulsion is the = 4 → 𝑠𝑝3
least. S33. Ans.(a, d)
S29. Ans.(a)
Intramolecular hydrogen bonding takes
place within the same molecule i.e.,
between the atoms of the same
molecule. Since, cellulose is a complex
structure containing oxygen and
hydrogen the bonding occurs between
them easily.
Whereas, in HCN, H2O2 and S34. Ans.(c)
concentrated acetic acid intermolecular
hydrogen bonding occurs.
Eg.:

S30. Ans.(c) CH4 – sp3 hybridised, tetrahedral bond


angle 109°28’
NH3 – sp3 hybridised, bond angle – 107°

Coplanar are in a plane or where all C


atoms are sp hybridised.
S31. Ans.(a)
SeF4 and CH4 do not have same shape.
SeF4 is AB4L type molecule with 4 bond H2O – sp3 – bent shaped bond angle –
pair and 1 lone pair with shape see-saw. 104°5’
CH4 is AB4 type molecule with no lone So, bond angle of H2O is less than that
pair and tetrahedral shape.
of NH3 & CH4
I3+ have 2 lone pairs with bent/angular
shape. BiCl5 does not exists because of S35. Ans.(b)
inert pair effect. SO2 type molecule have Order of repulsing force according to
both p – p & d – p bonds. VSEPR theory is lone pair – lone pair >
S32. Ans.(c) lone pair – bond pair > bond pair – bond
Hybridisation state = Number of  bond pair.
+ number of lone pair S36. Ans.(b)
Or O2 (atomic number) = 16
Hybridisation state → from steric Molecular orbital Diagram :
number rule Bond order = ½ (No. of bonding orbital –
no. of anti bonding orbitals)
Hybridisation state = ½ (10 – 6) = 2
1
= 2 (V.E + MA – C + a) Similarly, For 𝑂2− = 1.5
1
𝑂22− = 1
For, 𝑁𝑂2+ = 2 (5 + 0 − 1) 1
and Bond order ∝ Bond length ∝ stability
= 2 → 𝑠𝑝 ∴ Order of Bond length
1
𝑁𝑂3− = [5 + 0 + 1] = 𝑂22− > 𝑂2− > 𝑂2 > 𝑂2+
2

= 3 → 𝑠𝑝2

21
= 0.23 D

S43. Ans.(a)
𝑁𝑂3− resonating structure

Planar structure
S44. Ans.(a)
Hybridisation of CO2 is sp O = C = O
and of ethyne C2H2 is sp H – C ≡ C – H
S45. Ans.(b)
S37. Ans.(a)
Each pi bond have 2e– involved so 4 pi
bonds = 8e–.
S38. Ans.(a)
Bond order
𝑂2+ > 𝑂2 > 𝑂
S39. Ans.(c)
Indentation Bond order :
𝑂22+ = 3, 𝑂2− = 1.5, 𝑂2+ = 2.5
→ 𝑂22+ > 𝑂2+ > 𝑂2−
S40. Ans.(b)
S41. Ans.(a) S46. Ans.(c)

Among the 4 optuions (CH3)2, (SiH3)2


and PH3 all have complete octets with
8e–. But BH3 has only 6e– and act as a
Lewis acid.
S47. Ans.(b)

In paramagnetic species there is a


presence of unpaired electrons.
Except O−
2 all contains paired electrons.
S42. Ans.(b)

22
CHEMISTRY
Structure of Atom

1. Incorrect set of quantum numbers from the 4. The relation between nm, (nm = the number
following is: (2023) of permissible values of magnetic quantum
(a) n = 4, l = 3, m1 = −3, −2, −1, 0, +1, +2, number (m)) for a given value of azimuthal
+3, ms = −1/2 quantum number (𝑙) is (2023)
(b) n = 5, l = 2, m1 = −2, −1, +1, +2, ms = (a) 𝑙 = 2nm + 1
+1/2 (b) nm = 2𝑙 2 + 1
(c) n = 4, l = 2, m1 = −2, −1, 0, +1, +2, ms (c) nm = 𝑙 + 2
= −1/2 (d) 𝑙 =
nm −1
2
(d) n = 5, l = 3, m1 = −3, −2, −1, 0, +1, +2,
5. Match List I with List II:
+3, ms = +1/2
List I List II
2. Given below are two statements: (quantum number) (orbital)
Statement I: The value of wave function, ψ
(A) n = 2, l = 1 (i) 2s
depends upon the coordinates of the
electron in the atom. (B) n = 3, l = 2 (ii) 3s
Statement II: The probability of finding an n = 3, l = 0 (iii) 2p
(C)
electron at a point within an atom is
(D) n = 2, l = 0 (iv) 3d
proportional to the orbital wave function.
In the light of the above statements, choose Choose the correct answer from the options
the correct answer from the option given given below: (2022)
below: (2023) (a) A-iii, B-iv, C-ii, D-i
(a) Statement I is true but Statement II is (b) A-iii, B-iv, C-i, D-ii
false. (c) A-iv, B-iii, C-i, D-ii
(b) Statement II is false but Statement II is (d) A-iv, B-iii, C-ii, D-i
true.
6. When electromagnetic radiation of
(c) Both Statement I and Statement II are
wavelength 300 nm falls on the surface of a
true.
metal, electrons are emitted with the kinetic
(d) Both Statement I and Statement II are
energy of 1.68 × 105 J mol-1. What is the
false. minimum energy needed to remove an
3. Select the correct Statements from the electron from the metal? (2022)
following: (h = 6.626 × 10 Js, c = 3 × 10 ms-1, NA =
-34 8

A. Atoms of all elements are composed of 6.022 × 1023 mol-1)


two fundamental particles. (a) 2.31 × 105 J mol-1
B. The mass of the electron is 9.10939 × 10- (b) 2.31 × 106 J mol-1
31 kg. (c) 3.84 × 104 J mol-1
C. All the isotopes of a given elements show (d) 3.84 × 10-19 J mol-1
same chemical properties. 7. Identify the incorrect statement from the
D. Protons and electrons are collectively following. (2022)
known as nucleons. (a) All the five 5d orbitals are different in
E. Dalton's atomic theory, regarded the size when compared to the respective 4d
atom as an ultimate particle of matter. orbitals.
Choose the correct answer from the option (b) All the five 4d orbitals have shapes
given below. (2023) similar to the respective 3d orbitals.
(a) C, D and E only (c) In an atom, all the five 3d orbitals are
(b) A and E only equal in energy in free state.
(c) B, C and E only
(d) A, B and C only
23
(d) The shapes of dxy, dyz and dzx orbitals are state electronic configuration of (X) is 1s2
similar to each other and dx2 −y2 and dz2 2s2 2p3, the simplest formula for this
compound is (2018)
are similar to each other. (a) Mg2X3
8. If radius of second Bohr orbit of the He+ ion (b) MgX2
is 105.8 pm, what is the radius of third (c) Mg3X2
Bohr orbit of Li2+ ion? (2022) (d) Mg2X
(a) 158.7 pm 15. Which one is a wrong statement? (2018)
(b) 15.87 pm (a) Total orbital angular momentum of
(c) 1.587 pm electron in ‘s’ orbital is equal to zero
(d) 158.7 Ȧ (b) An orbital is designated by three
9. A particular station of All India Radio, New quantum numbers while an electron in
Delhi, broadcasts on a frequency of 1,368 an atom is designated by four quantum
kHz (kilohertz). The wavelength of the numbers
electromagnetic radiation emitted by the (c) The value of m for 𝑑𝑧2 is zero
transmitter is : (d) The electronic configuration of N atom is
[speed of light, c = 3.0 × 108 ms–1] (2021)
(a) 219.2 m
(b) 2192 m
(c) 21.92 m 16. Which one is the wrong statement?
(d) 219.3 m (2017-Delhi)
(a) The energy of 2s orbital is less than the
10. The number of protons, neutrons and
energy of 2p orbital in case of hydrogen
electrons in 175
71𝐿𝑢, respectively, are :
like atoms
(b) de-Broglie’s wavelength is given by 𝜆 =
(a) 104, 71 ad 71 ℎ
(b) 71, 71 and 104 𝑚𝑣
, where m = mass of the particle, v =
(c) 175, 104 and 71 group velocity of the particle

(d) 71, 104 and 71 (c) The uncertainty principle is ∆𝐸 × ∆𝑡 ≥ 4𝜋
11. The number of angular nodes and radial (d) Half-filled and fully filled orbitals have
nodes in 3s orbital are greater stability due to greater exchange
(2020 Covid Re-NEET)
(a) 1 and 0, respectively energy, greater symmetry and more
(b) 3 and 0, respectively balanced arrangement
(c) 0 and 1, respectively
17. The total number of orbitals present for
(d) 0 and 2, respectively
principal quantum number, n = 4 is :
12. 4d, 5p, 5f and 6p orbitals are arranged in (2017-Gujarat)
the order of decreasing energy. The correct (a) 30
option is (2019) (b) 12
(a) 5f > 6p > 5p > 4d (c) 15
(b) 6p > 5f > 5p > 4d (d) 16
(c) 6p > 5f > 4d > 5p
18. Which of the following pairs of d-orbitals
(d) 5f > 6p > 4d > 5p
will have electron density along the axes?
13. Which of the following series of transitions (2016-II)
in the spectrum of hydrogen atom fall in (a) 𝑑𝑧2 , 𝑑𝑥 2 −𝑦2
visible region? (2019) (b) 𝑑𝑥𝑦 , 𝑑𝑥 2 −𝑦2
(a) Lyman series
(c) 𝑑𝑧2 , 𝑑𝑥𝑧
(b) Balmer series
(d) 𝑑𝑥𝑧 , 𝑑𝑦𝑧
(c) Paschen series
(d) bracket series 19. How many electrons can fit in the orbital for
which : n = 3 and l = 1? (2016-II)
14. Magnesium reacts with an element (X) to
(a) 10
form an ionic compoun(d). If the ground
24
(b) 14
(c) 2 26. The value of Planck’s constant is 6.63 ×
(d) 6
10−34 𝐽𝑠. The speed of light is 3 × 108 𝑚𝑠 −1 .
20. Two electrons occupying the same orbital Which value is closest to the wavelength in
are distinguished by : (2016-I) nanometer of a quantum of light with
(a) Spin quantum number frequency of 6 × 1015 𝑠 −1 ? (2013)
(b) Principal quantum number (a) 10 nm
(c) Magnetic quantum number (b) 25 nm
(d) Azimuthal quantum number (c) 50 nm
21. Which is the correct order of increasing (d) 75 nm
energy of the listed orbitals in the atom of 𝑍2
27. Based on equation, 𝐸 = −2.178 × 10−18 𝐽 (𝑛2 )
titanium? (Atomic number Z = 22)
(2015 Re) certain conclusions are written. Which of
(a) 3s 3p 4s 3d them is not correct? (2013)
(b) 3s 4s 3p 3d (a) The negative sign in equation simply
(c) 4s 3s 3p 3d means that the energy of electron bound
(d) 3s 3p 3d 4s to the nucleus is lower than it would be
if the electrons were at the infinite
22. The angular momentum of electron in ‘d’ distance from the nucleus.
orbital is equal to : (2015) (b) Larger the value of n, the larger is the
(a) 2√3ℎ orbit radius.
(b) 0ℎ (c) Equation can be used to calculate the
(c) √6ℎ change in energy when the electron
(d) √2ℎ changes orbit.
(d) For n = 1, the electron has a more
23. What is the maximum number of orbitals
negative energy than it does for n = 6
that can be identified with the following
quantum numbers: (2014) which means that the electron is more
(a) 2 loosely bound in the smallest allowed
(b) 3 orbit.
(c) 4
(d) 1
24. Calculate the energy in joule corresponding
to light of wavelength 45 nm (Planck’s
constant h = 6.63 × 10–34 Js; speed of light
c = 3 × 108 ms–1) (2014)
(a) 6.67 × 1011
(b) 4.42 × 10–15
(c) 4.42 × 10–18
(d) 6.67 × 1015
25. What is the maximum number of electrons
that can be associated with the following set
of quantum numbers? (n = 3, l = 1 and m =
–1). (2013)
(a) 10
(b) 6
(c) 4
(d) 2

25
Answer Key
S1. Ans. (b) S16. Ans. (a)

S2. Ans. (a) S17. Ans. (d)

S3. Ans. (c) S18. Ans. (a)

S4. Ans. (d) S19. Ans. (c)

S5. Ans. (a) S20. Ans. (a)

S6. Ans. (a) S21. Ans. (a)

S7. Ans. (d) S22. Ans. (c)

S8. Ans. (a) S23. Ans. (d)

S9. Ans. (d) S24. Ans. (c)

S10. Ans. (d) S25. Ans. (d)

S11. Ans. (d) S26. Ans. (c)

S12. Ans. (a) S27. Ans. (d)

S13. Ans. (b)

S14. Ans. (c)

S15. Ans. (d)

26
Solutions
S1. Ans.(b) = 2.31 × 105 J mol-1
n = 5, l = 2, m = −2, −1, +1, +2, ms = S7. Ans.(d)
+1/2 In an atom, all the five 3d orbitals are
S2. Ans.(a) equal in energy in free state i.e.,
Statement I is true and Statement II is degenerate.
false. The shape of dx2 −y2 is different then
S3. Ans.(c) shape of dz2 .
It is statement based question.
Statements B, C and E are correct.
(B) Mass of the electron is 9.10939 ×
10-31 kg
(C) All the isotopes of given elements
show same chemical properties.
(E) Dalton's atomic theory, regarded the The size of orbital depends on principal
atom as an ultimate particle of quantum number 'n' therefore all the five
matter. 3d orbitals are different in size when
S4. Ans.(d) compared to the respective 4d orbitals.
Sol. Number of permissible values of Shape of orbitals depends on azimuthal
magnetic quantum number for a given quantum number 'l' therefore shapes of
value of azimuthal quantum (𝑙) 4d orbitals are similar to the respective
3d orbitals.
⟹ nm = 2𝑙 + 1
nm −1 S8. Ans.(a)
⟹𝑙= 2
rn ∝ n2/Z
S5. Ans.(a) r3 (Li2+ ) (n3 )2 Z(He+ )
= ×
n l Subshell notation r2 (He+ ) Z(Li2+ ) (n2 )2

r3 (Li2+ ) (3)2 2
2 0 2s = × (2)2
105.8 3
2 1 2p
= 105.8 × 3/2
3 0 3s r3 (Li2+ ) = 158.7 pm
3 1 3p S9. Ans.(d)
3 2 3d 𝜆=𝑣
𝑐

3×108
𝜆= = 219.298 𝑚 = 219.3 𝑚
S6. Ans.(a) 1368×103

hc S10. Ans.(d)
Energy of one photon = λ
(λ = 300 nm)
17
71𝐿𝑢
hc
For one mole photons, E = λ
× NA Z = atomic number
6.626×10−34 ×3×108 ×6.023×1023 Z = No. of Protons = 71 = No. of
E= 300×10−9
Electrons
E = 3.99 × 105 J mol-1
No. of Neutrons = Mass no. – No. of
Kinetic energy = 1.68 × 105 J mol-1 Protons
W0 = E – K.E. = 175 – 71
= 3.99 × 105 – 1.68 × 105 =104

27
S11. Ans.(d) S16. Ans.(a)
Number of radial nodes = n – l – 1 The energy of 2s orbital is less than the
Number of angular nodes = l energy of 2p orbital in case of hydrogen
like atoms is a wrong statement because
For 3s orbital,
1s > 2s = 2p > 3s = 3p = 3d ….. etc.
l=0
S17. Ans.(d)
– Number of radial nodes = 3 – 0 – 1 = 2
The total number of orbital present in n
– Number of angular nodes = 0
= 4 is n2.
S12. Ans.(a)
= (4)2 = 16
n l
Shell No. of orbital
(𝑛 + 1) value for, 4d = 4 + 2 = 6 s 1
5p = 5 + 1 = 6 p 3
5f = 5 + 3 = 8 d 5
6p = 6 + 1 = 7 f 7
Lower value of (n + l) signifies lower S18. Ans.(a)
energy Among d-orbitals 𝑑𝑧2 and 𝑑𝑥 2 −𝑦2 have
In case of 4d and 5p, lower value of n in their electron densities oriented towards
4d has compare to 5p. axes.
So, 4d has less energy in comparison to S19. Ans.(c)
5p. When n = 3 and l = 1 orbital is 3p, so
∴ Correct order of energy will be 5f > 6p total number of electron that can be
> 5p > 4d filled are 6 but in any orbital only 2
S13. Ans.(b) electron can accumulate.
In visible region Balmer series S20. Ans.(a)
transitions fall in H-spectrum. 2 electron occupying the same orbital
S14. Ans.(c) can be distinguished using their spin
quantum number (ms) where one is
Element (X) electronic configuration
clockwise (↑) and other is anti-clockwise
1s2 2s2 2p3 (↓). In such case, the value of spin
So, valency of X will be 3. quantum number changes rest all
Valency of Mg is 2. quantum number remains the same.

Formula of compound formed by Mg S21. Ans.(a)


and X will be Mg3X2. According to Aufbau’s rule of increasing
S15. Ans.(d) order of energy for filling up of electron.

According to Hund’s Rule of maximum 3s < 3p < 4s < 3d


multiplicity, the correct electronic S22. Ans.(c)
configuration of N-atom is ℎ
Angular momentum= √𝑙(𝑙 + 1)
2𝜋

For d orbital l = 2

So, angular momentum= √2(2 + 1) 2𝜋
Or

= √6
2𝜋

= ℏ → called as Planck’s reducing
∵ Option (d) violates Hund’s Rule. 2𝜋
constant or Dirac constant.

28
S23. Ans.(d)
n =3 l=1 ml = 0
l=p 3p = 1 orbital as ml = 0 sp 3𝑝𝑧 .

Thus, maximum no. of orbitals


identified in 3pz is 1.
S24. Ans.(c)
ℎ𝑐
𝐸= 𝜆
6.63×10−34 ×3×108
𝐸= 45×10−9
= 4.42 × 10−18 𝐽
S25. Ans.(d)
n=3 l=1 ml = –1
for p orbital l = 1
So orbital must be 3px or 3py, where
number of electron in each orbital will
be 2.
S26. Ans.(c)
According to formula :
ℎ𝑐
𝐸= 𝜆
or hv
𝑐 3×108
Where, =𝑣 ⇒ 𝜆= = 50 𝑛𝑚
𝜆 6×1015

S27. Ans.(d)
The orbitals which are closer to nucleus
are more strongly affected by the
positive charged field of protons in
nucleus. Whereas when we move away
from nucleus and n (principal quantum
number) increases, the effective
influence of positive charged field
decreases & electron in later (n) orbitals
become loosely bounded.

29
Chapter 5
Thermodynamics

1. Consider the following reaction: 6. Which of the following p-V curve represents
2H2(g) + O2(g) ⟶ 2H2O(g) ∆rH° = −483.64 kJ maximum work done? (2022)
What is the enthalpy change for
decomposition of one mole of water?
(Choose the right option). (2023)
(a) 120.9 kJ
(b) 241.82 kJ
(c) 18 kJ
(d) 100 kJ (a)

2. The equilibrium concentrations of the


species in the reaction A + B ⇌ C + D are 2,
3, 10 and 6 mol L-1, respectively at 300 K.
∆G° for the reaction is: (R = 2 cal/mol K)
(2023)
(a) −137.26 cal (b)
(b) −1381.80 cal
(c) −13.73 cal
(d) 1372.60 cal
3. Which amongst the following options is the
correct relation between change in enthalpy
and change in internal energy? (2023)
(c)
(a) ∆H = ∆U + ∆ngRT
(b) ∆H – ∆U = −∆nRT
(c) ∆H + ∆U = ∆nR
(d) ∆H = ∆U – ∆ngRT
4. One mole of an ideal gas at 300 K is
expanded isothermally from 1 L to 10 L
volume. ∆U for this process is: (2022) (d)
(Use R = 8.314 J k-1 mol-1) 7. Which one among the following is the
(a) 0 J correct option for right relationship between
(b) 1260 J CP and CV for one mole of ideal gas? (2021)
(c) 2520 J (a) CP – CV = R
(d) 5040 J (b) CP = RCV
5. A vessel contains 3.2 g of dioxygen gas at (c) CV = RCP
STP (273.15 K and 1 atm pressure). The gas (d) CP + CV = R
is now transferred to another vessel at 8. For irreversible expansion of an ideal gas
constant temperature, where pressure under isothermal condition, the correct
becomes one third of the original pressure. option is: (2021)
The volume of new vessel in L is: (2022) (a) ∆𝑈 ≠ 0, ∆𝑆𝑡𝑜𝑡𝑎𝑙 ≠ 0
(Given: molar volume at STP is 22.4 L) (b) ∆𝑈 = 0, ∆𝑆𝑡𝑜𝑡𝑎𝑙 ≠ 0
(a) 67.2 (c) ∆𝑈 ≠ 0, ∆𝑆𝑡𝑜𝑡𝑎𝑙 = 0
(b) 6.72 (d) ∆𝑈 = 0, ∆𝑆𝑡𝑜𝑡𝑎𝑙 = 0
(c) 2.24
9. For the reaction, 2𝐶𝑙(𝑔) → 𝐶𝑙2 (𝑔), the correct
(d) 22.4
option is: (2020)

30
(a) ∆𝑟 𝐻 > 0 𝑎𝑛𝑑 ∆𝑟 𝑆 < 0 The bond dissociation energy of X2 will be
(b) ∆𝑟 𝐻 < 0 𝑎𝑛𝑑 ∆𝑟 𝑆 < 0 (2018)
(c) ∆𝑟 𝐻 < 0 𝑎𝑛𝑑 ∆𝑟 𝑆 < 0 (a) 200 kJ mol –1

(d) ∆𝑟 𝐻 > 0 𝑎𝑛𝑑 ∆𝑟 𝑆 > 0 (b) 100 kJ mol–1


(c) 400 kJ mol–1
10. The correct option for free expansion of an
(d) 800 kJ mol–1
ideal gas under adiabatic condition is:
(2020) 16. A gas is allowed to expand in a well
(a) 𝑞 = 0, ∆𝑇 < 0 𝑎𝑛𝑑 𝑤 > 0 insulated container against a constant
(b) 𝑞 < 0, ∆𝑇 = 0 𝑎𝑛𝑑 𝑤 = 0 external pressure of 2.5 atm from an initial
(c) 𝑞 > 0, ∆𝑇 > 0 𝑎𝑛𝑑 𝑤 > 0 volume of 2.50 L to a final volume of 4.50 L.
(d) 𝑞 = 0, ∆𝑇 = 0 𝑎𝑛𝑑 𝑤 = 0 The change in internal energy ∆U of the gas
in joules will be: (2017-Delhi)
11. If for a certain reaction ∆𝑟 𝐻 is 30 kJ mol–1 at
(a) +505 J
450 K, the value of ∆𝑟 𝑆 (in JK–1 mol–1) for
(b) 1136.25 J
which the same reaction will be
(c) –500 J
spontaneous at the same temperature is
(d) –505 J
(2020 Covid Re-NEET)
(a) –33 17. For a given reaction, ∆H = 35.5 KJ mol−1 and
(b) 33 ∆S = 83.6 JK −1 mol−1 . The reaction is
(c) –70 spontaneous at: (Assume that ∆H and ∆S do
(d) 70 not vary with temperature) (2017-Delhi)
(a) T > 298 K
12. At standard conditions, if the change in the
enthalpy for the following reaction is –109 (b) T < 425 K
kJ mol–1 (c) T > 425 K
𝐻2(𝑔) + 𝐵𝑟2(𝑔) → 2𝐻𝐵𝑟
Given that bond energy of H and Br is 435 18. Of the following, the largest value of entropy
kJ mol–1 and 192 kJ mol–1, respectively, at 25°C and 1 atm is that of :
what is the bond energy (in kJ mol–1) of (2017-Gujarat)
HBr? (2020 Covid Re-NEET) (a) CH4
(a) 736 (b) H2
(b) 518 (c) C2H6
(c) 259 (d) C2H2
(d) 368 19. Under isothermal and reversible conditions,
13. Under isothermal condition, a gas at 300 K the term “free energy” in thermodynamics
expands from 0.1 L to 0.25 L against a signifies : (2017-Gujarat)
constant external pressure of 2 bar. The (a) Expansion work done on the system
work done by the gas is (Give that 1 L bar = (b) Non-expansion work done by the system
100 J) (2019) (c) Expansion work done by the system
(a) –30 J (d) Non expansion work done on the system
(b) 5 kJ 20. For a sample of perfect gas when its
(c) 25 J pressure is changed isothermally from Pi to
(d) 30 J Pf, the entropy change is given by:
14. In which case change in entropy is (2016-II)
𝑃
negative? (2019) (a) ∆𝑆 = 𝑛𝑅𝑇 𝑙𝑛 ( 𝑃𝑓 )
(a) Evaporation of water 𝑃
𝑖

(b) Expansion of a gas at constant (b) ∆𝑆 = 𝑛𝑅𝑇 𝑙𝑛 (𝑃 𝑖 )


𝑓
temperature 𝑃𝑓
(c) ∆𝑆 = 𝑛𝑅 𝑙𝑛 ( 𝑃 )
(c) Sublimation of solid to gas 𝑖

(d) 2H(g) → H2 (g) (d) ∆𝑆 = 𝑛𝑅


𝑃
𝑙𝑛 (𝑃 𝑖 )
𝑓
15. The bond dissociation energies of X2, Y2 and
XY are in the ratio of 1 : 0.5 : 1. ∆H for the
formation of XY is –200 kJ mol–1.
31
21. The correct thermodynamic conditions for
the spontaneous reaction at all
temperatures is: (2016-II)
(a) H < 0 and S < 0
(b) H < 0 and S = 0
(c) H > 0 and S < 0
(d) H < 0 and S > 0
22. Consider the following liquid-vapour
equilibrium.
Liquid ⇌ Vapour . Which of the following
relations is correct? (2016-I)
𝑑𝑙𝑛𝑃 −∆𝐻𝑣
(a) 𝑑𝑇 2
= 𝑇2
𝑑𝑙𝑛𝑃 −∆𝐻𝑣
(b) 𝑑𝑇
= 𝑅𝑇 2
𝑑𝑙𝑛𝐺 ∆𝐻𝑣
(c) =
𝑑𝑇 2 𝑅𝑇 2
𝑑𝑙𝑛𝑃 −∆𝐻𝑣
(d) =
𝑑𝑇 𝑅𝑇

23. The heat of combustion of carbon to CO2 is


–393.5 kJ/mol. The heat released upon
formation of 35.2 g of CO2 from carbon and
oxygen gas is: (2015)
(a) +315 kJ
(b) –630 kJ
(c) –3.15 kJ
(d) –315 kJ
24. For the reac 𝑋2 𝑂4 (𝑙) → 2𝑋𝑂2 (𝑔) ∆𝑈 =
2.1 𝑘𝑐𝑎𝑙, ∆𝑆 = 20 𝑐𝑎𝑙 𝐾 −1 at 300 K. Hence, ∆𝐺
is : (2014)
(a) –2.7 kcal
(b) 9.3 kcal
(c) –9.3 kcal
(d) 2.7 kcal

32
Answer Key

S1. Ans. (b)

S2. Ans. (b)

S3. Ans. (a)

S4. Ans. (a)

S5. Ans. (b)

S6. Ans. (b)

S7. Ans. (a)

S8. Ans. (b)

S9. Ans. (c)

S10. Ans. (d)

S11. Ans. (d)

S12. Ans. (d)

S13. Ans. (a)

S14. Ans. (d)

S15. Ans. (d)

S16. Ans. (d)

S17. Ans. (c)

S18. Ans. (c)

S19. Ans. (b)

S20. Ans. (d)

S21. Ans. (d)

S22. Ans. (b)

S23. Ans. (a)

S24. Ans. (a)

33
Solutions
S1. Ans.(b) S6. Ans.(b)
Decomposition for 1 mole of water Work done under any thermodynamic
1 483.64 process can be determined by area under
H2O(g) ⟶ H2(g) + 2O2(g); ∆H = + 2
the 'p-V' graph. As it can be observed
∆H = +241.82 kJ maximum area is covered in option '2'.
S2. Ans.(b)
A+B⇌C+D

[A] = 2 mol L-1

[B] = 3 mol L-1


1.
[C] = 10 mol L-1

[D] = 6 mol L-1

∆G° = −2.303 RT log Keq


[C][D]
= −2.303 RT log [A][B]
2.
10×6
= −2.303 × 2×3

= −2.303 × 2 × 300 × log 10

= −1381.8 cal

S3. Ans.(a)
∆H = ∆U + ∆ngRT 3.

S4. Ans.(a)
∆U = nCv∆T

For isothermal condition; ∆T = 0

∴ ∆U = 0 4.

S5. Ans.(b)
At constant temperature and amount S7. Ans.(a)
For one mole of an ideal gas CP − CV = R
P1V1 = P2V2
S8. Ans.(b)
P1 P1
P1V1 = 3
V2 [∴ P2 = 3
] For Irreversible expansion of an ideal
gas under Isothermal condition
V2 = 3V1
∆𝑈 = 0
mole of O2(g) = 3.2/32 = 0.1 mole
∆𝑆𝑇𝑜𝑡𝑎𝑙 ≠ 0
Volume of O2(g) = (0.1 × 22.4)L = 2.24 L S9. Ans.(c)
At STP (V1) Given reaction, 2𝐶𝑙(𝑔) → 𝐶𝑙2 (𝑔)
The reaction is exothermic since a bond
V2 = 3V1 = 3 × 2.24 = 6.72 L
is being formed in the given reaction,
i.e., ∆𝐻 is also –ve. So ∆𝑟 𝐻 < 0 and

34
Entropy is decreasing (–ve) in the S14. Ans.(d)
reaction as no. of molecules are (i) For evaporation of water S > 0;
decreasing which reduces the
S = +ve
randomness of the system.
(ii) In expansion of gas at constant T
Thus, ∆𝑟 𝑆 < 0 S > 0 ; S = +ve
So, (c) option is correct. (iii) S→g
S = +ve
S10. Ans.(d)
(iv) 2Hg → H2 (g), ∆S < 0 (∵ ng < 0)
Free expansion, since 𝑃𝑒𝑥𝑡 = 0 No. of particle decreases from reactant
𝑤 = −𝑃𝑒𝑥 ∆𝑉 = 0 ; So, work done = 0 to product side.
For adiabatic condition, q = 0 S15. Ans.(d)

From first law of thermodynamics The reaction for ∆𝑓 𝐻°(𝑋𝑌)


1 1
∆𝐸 = 𝑞 + 𝑤 𝑋 (𝑔) + 2 𝑌2 (𝑔)
2 2
→ 𝑋𝑌(𝑔)
∴ ∆𝐸 = 0 Bond energies of X2, Y2 and XY are x, 2,
𝑥

Internal energy of an ideal gas is a x respectively


function of temperature 𝑥 𝑥 𝑥
∴ ∆𝐻 = (2 + 4) − 2 = −200
∴ If internal energy remains constant
∴ ∆𝑇 = 0 On solving, we get
𝑥
Thus (d) option is correct. ⇒ 4
= −200
S11. Ans.(d) ⇒ x = 800 kJ/mole
For spontaneous reaction, ∆ 𝐺 must be S16. Ans.(d)
less than zero U = q + W
So, ∆𝑟 𝐺 = ∆𝑟 𝐻 − 𝑇∆𝑟 𝑆 < 0 For adiabatic process, q = 0
∆𝑟 𝐻
or, ∆𝑟 𝑆 > 𝑇 ∴ U = W = –P. V

>
30,000
= 66.67 𝐽𝐾 −1 𝑚𝑜𝑙 −1 = –2.5 atm × (4.50 – 2.50) L
450
= –2.5 × 2 L-atm ⇒ –5 × 101.3 J
For reaction to be spontaneous the
value of ∆𝑟 𝑆 must be greater than 66.67 = –506.5 J ⇒ –505 J
J that is 70 JK–1 mol–1
S12. Ans.(d) S17. Ans.(c)
𝐻2 (𝑔) + 𝐵𝑟2 (𝑔) → 2𝐻𝐵𝑟(𝑔) ; G = H – TS
−1
∆𝐻 = −109𝑘𝐽 𝑚𝑜𝑙 For spontaneous reaction, G must be
∆𝐻= sum of bond energy of reactants – negative. For negative value of G, H
sum of bond energy of products should be less than TS. It is possible
(𝐵𝐸𝐻−𝐻 ) + (𝐵𝐸𝐵𝑟−𝐵𝑟 ) − 2(𝐵𝐸𝐻−𝐵𝑟 ) when T > 425 K.

−109 = (435) + (192) − 2(𝐵𝐸𝐻−𝐵𝑟 ) H = 35.5 KJ/mol = 35500 J/mol


𝐵𝐸𝐻−𝐵𝑟 = 368 𝑘𝐽 𝑚𝑜𝑙 −1 TS = (425) (83.6) = 35530
S13. Ans.(a) TS > H
𝑊𝑖𝑟𝑟 = −𝑃𝑒𝑥𝑡 ∆𝑉 = −2 𝑏𝑎𝑟 × (0.25 − 0.1)𝐿 Hence, the reaction is spontaneous.
= –0.30 L-bar = –0.30 × 100 J S18. Ans.(c)
⇒ –30 J The molecule having more number of
bonds have largest value of entropy.
∴ C2H6 have large value of entropy.

35
S19. Ans.(b)
Under isothermal reversible conditions,
the term “free-energy” in
thermodynamics signifies that “No
expansion work done by the system”.
−∆𝐺𝑠𝑦𝑠𝑡𝑒𝑚 = 𝑊𝑛𝑜𝑛−𝑒𝑥𝑝𝑎𝑛𝑠𝑖𝑜𝑛
S20. Ans.(d)
For isothermal process
𝑃
∆𝑆 = 𝑛𝑅𝑙𝑛 𝑃 𝑖
𝑓

S21. Ans.(d)
For any spontaneous process, G = (–ve)
and S = (+ve) that is increase in
entropy.
So, at H < 0 and S > 0 at all
temperatures according the reaction will
be spontaneous.
S22. Ans.(b)
𝑑𝑙𝑛𝑃 −∆𝐻𝑣
=
𝑑𝑇 𝑅𝑇 2
equation).
S23. Ans.(a)
𝐶(𝑠) + 𝑂2 → 𝐶𝑂 (𝑔) ∆𝐻 = −393.5 𝐾𝐽/𝑚𝑜𝑙𝑒
∆𝐻𝑟 = ∆𝐻𝑓𝐶𝑂2
So heat of formation of CO2
= 393.5 kJ/mole
or we can say
Heat released on formation of 44 gm
CO2 = 393.5 kJ
Heat released on formation of 35.2 gm
393.5
of 𝐶𝑂2 = 44
× 35.2 = 314.8
⇒ ≃315 kJ
S24. Ans.(a)
According + 2 × 2 × 300 = 3300
G = H – TS = 3300 – 300 × 20
= –2700 cal = –2.7 kcal

36
Chapter 6
Equilibrium

1. For a weak acid HA, the percentage of (a) 3.6


dissociation is nearly 1% at equilibrium. If (b) 0.36
the concentration of acid is 0.1 mol L-1, then (c) 3.6 × 10-2
the correct option for its Ka at the same (d) 3.6 × 10-3
temperature is: (2023) 6. The pH of the solution containing 50 mL
(a) 1 × 10 -4
each of 0.10 M sodium acetate and 0.01 M
(b) 1 × 10-6 acetic acid is (2022)
(c) 1 × 10-5 [Given: pKa of CH3COOH = 4.57]
(d) 1 × 10-3 (a) 5.57
2. An acidic buffer is prepared by mixing: (b) 3.57
(2023) (c) 4.57
(a) weak acid and it's salt with strong base (d) 2.57
(b) equal volumes of equimolar solutions of 7. 3O2(g) ⇌ 2O3(g)
weak acid and weak base For the above reaction at 298 K, Kc is found
(c) strong acid and it's salt with strong base to be 3.0 × 10-59. If the concentration of O2
(d) strong acid and it's salt with base (The at equilibrium is 0.040 M then
pKa of acid = pKb of the base) concentration of O3 in M is (2022)

3. 0.01 M acetic acid solution is 1% ionised, (b) 1.9 × 10


then pH of this acetic acid solution is: (c) 2.4 × 1031
(2022) (d) 1.2 × 1021
(a) 1 8. The pKb of dimethylamine and pKa of acetic
(b) 3 acid are 3.27 and 4.77 respectively at T(K).
(c) 2 The correct option for the pH of
(d) 4 dimethylammonium acetate solution is:
4. KH value for some gases at the same (2021)
temperature 'T' are given: (2022) (a) 5.50
Gas KH/k bar (b) 7.75
(c) 6.25
Ar 40.3
(d) 8.50
CO2 1.67
9. Find out the solubility of Ni(OH)2 in 0.1 M
HCHO 1.83 × 10-5 NaOH. Given that the ionic product of
CH4 0.413 Ni(OH)2 is 2 × 10–15. (2020)
(a) 2 × 10 M
–8
Where KH is Henry's Law constant in water.
(b) 1 × 10–13 M
The order of their solubility in water is:
(c) 1 × 108 M
(a) HCHO < CH4 < CO2 < Ar
(d) 2 × 10–13 M
(b) Ar < CO2 < CH4 < HCHO
(c) Ar < CH4 < CO2 < HCHO 10. HCl was passed through a solution of CaCl2,
(d) HCHO < CO2 < CH4 < Ar MgCl2 and NaCl. Which of the following
compound(s) crystallies(s)? (2020)
5. Kp for the following reaction is 3.0 at 1000
(a) Only NaCl
K.
(b) Only MgCl2
CO2(g) + C(s) ⇌ 2CO(g) (c) NaCl, MgCl2 and CaCl2
What will be the value of Kc for the reaction (d) Both MgCl2 and CaCl2
at the same temperature? (2022)
(Given: R = 0.083 L bar K-1 mol-1)
37
11. Which among the following salt solutions is (b) Low temperature and low pressure
basic in nature? (2020 covid Re-NEET) (c) High temperature and low pressure
(a) Ammonium sulphate (d) High temperature and high pressure
(b) Ammonium nitrate 18. The solubility of BaSO4 in water is 2.42 ×
(c) Sodium acetate 10–3 gL–1 at 298 K. The value of its solubility
(d) Ammonium chloride product (Ksp) will be: (2018)
12. The solubility product for a salt of the type (Given molar mass of BaSO4 = 233 g mol–1)
AB is 4 × 10–8. What is the molarity of its (a) 1.08 ×10–10 mol2 L–2
standard solution? (2020 Covid Re-NEET) (b) 1.08 ×10–12 mol2 L–2
(a) 16 × 10–16 mol/L (c) 1.08 × 10–8 mol2 L–2
(b) 2 × 10–16 mol/L (d) 1.08 × 10–14 mol2 L–2
(c) 4 × 10–4 mol/L 19. Following solutions were prepared by
(d) 2 × 10–4 mol/L mixing different volumes of NaOH and HCl
13. Hydrolysis of sucrose is given by the of different concentrations : (2018)
following reaction. 𝑀 𝑀
(A) 60𝑚𝐿 10 𝐻𝐶𝑙 + 40𝑚𝐿 10 𝑁𝑎𝑂𝐻
Sucrose + H2 O ⇌ Glucose + Fructose 𝑀 𝑀
If the equilibrium constant (Kc) is 2 × 1013 (B) 55𝑚𝐿 10 𝐻𝐶𝑙 + 45𝑚𝐿 10 𝑁𝑎𝑂𝐻
𝑀 𝑀
at 300 K, the value of ∆r G° at the same (c) 75𝑚𝐿 5 𝐻𝐶𝑙 + 25𝑚𝐿 5 𝑁𝑎𝑂𝐻
temperature will be : 𝑀 𝑀
(d) 100𝑚𝐿 𝐻𝐶𝑙 + 100𝑚𝐿 𝑁𝑎𝑂𝐻
(a) 8.314 𝐽 𝑚𝑜𝑙 −1 𝐾 −1 × 300 𝐾 × ln (2 × 1013 ) 10 10

(b) 8.314 𝐽 𝑚𝑜𝑙 −1 𝐾 −1 × 300 𝐾 × ln (3 × 1013 ) pH of which one of them will be equal to 1?
(c) −8.314 𝐽 𝑚𝑜𝑙 −1 𝐾 −1 × 300 𝐾 × ln (4 × 1013 ) (a) B
(d) – 8.314 𝐽 𝑚𝑜𝑙 −1 𝐾 −1 × 300 𝐾 × 𝑙𝑏(2 × 1013 ) (b) A
(c) C
14. pH of a saturated solution of Ca(OH2) is 9. (d) D
The solubility product (Ksp) of Ca(OH)2 is:
(2019) 20. The equilibrium constants of the following
(a) 0.5 × 10–15 are : (2017-Delhi)
(b) 0.25 × 10–10 𝑁2 + 3𝐻2 ⇌ 2𝑁𝐻3 𝐾1
(c) 0.125 × 10–15 𝑁2 + 𝑂2 ⇌ 2𝑁𝑂 𝐾2
1
(d) 0.5 × 10–10 𝐻2 + 𝑂2 → 𝐻2 𝑂
2
𝐾3
15. Conjugate base for Bronsted acids H2O and The equilibrium constant (K) of the reaction:
5 𝐾
HF are : (2019) 2𝑁𝐻3 + 2 𝑂2 ⇌ 2𝑁𝑂 + 3𝐻2 𝑂 , will be:
(a) OH– and H2F+, respectively (a) 𝐾23 𝐾3 /𝐾1
(b) H3O+ and F–, respectively (b) 𝐾1 𝐾33 /𝐾2
(c) OH– and F–, respectively
(c) 𝐾2 𝐾33 /𝐾1
(d) H3O+ and H2F+, respectively
(d) 𝐾2 𝐾3 /𝐾1
16. Which will make basic buffer? (2019)
21. Concentration of Ag+ ions in a saturated
(a) 50 mL of 0.1 M NaOH + 25 mL of 0.1 M
solution of Ag2C2O4 is 2.2 × 10–4 mol L–1.
CH3COOH
Solubility product of Ag2CO4 is:
(b) 100 mL of 0.1 M CH3COOH + 100 mL of
(2017-Delhi)
0.1 M NaOH
(a) 5.3 × 10 –12
(c) 100 mL of 0.1 M HCl + 200 mL of 0.1 M
(b) 2.42 × 10–8
NH4OH
(c) 2.66 × 10–12
(d) 100 mL of 0.1 M HCl + 100 mL of 0.1 M
(d) 4.5 × 10–11
NaOH
22. Which one of the following statements is not
17. Which one of the following conditions will correct? (2017-Delhi)
favour maximum formation of the product (a) Coenzymes increase the catalytic
in the reaction, activity of enzyme
𝐴2 (𝑔) + 𝐵2 (𝑔) ⇌ 𝑋2 (𝑔) ∆𝑟 𝐻 = −𝑋 𝑘𝐽 (2018) (b) Catalyst does not initiate any reaction
(a) Low temperature and high pressure

38
(c) The value of equilibrium constant is (b) 1.6%
changed in the presence of a catalyst in (c) 0.0060%
the reaction at equilibrium (d) 0.013%
(d) Enzymes catalyse mainly bio-chemical 29. Consider the nitration of benzene using
reactions mixed conc. H2SO4 and HNO3. If a larger
23. The standard equilibrium constant KP at amount of KHSO4 is added to the mixture,
298 K for the reaction, 𝑁2 (𝑔) + 3𝐻2 (𝑔) ⇌ the rate of nitration will be: (2016-I)
2𝑁𝐻3 (𝑔) is 5.8 × 105. The value of standard (a) Doubled
equilibrium constant, if the concentration (b) Increase
of gases is expressed in terms of mol L–1, will (c) Decrease
be: (2017-Gujarat) (d) Unchanged
[Given : R = 0.08314 L bar K–1 mol–1]
(a) 3.99 × 109 30. MY and NY3, two nearly insoluble salts,
(b) 3.51 × 106 have the same Ksp values of 6.2 × 10–13 at
(c) 3.84 × 107 room temperature, which statements would
(d) 3.56 × 108 be true in regard to MY and NY3?
(2016-I)
24. Consider the following reaction for which (a) The addition of the salt of KY to solution
the change in enthalpy is positive of MY and NY3 will have no effect on
2𝐴(𝑔) + 𝐵(𝑔) ⇌ 𝐶(𝑔) + 𝐷(𝑔) their solubilities
Which of the following will not affect the (b) The molar solubilities of MY and NY3 in
equilibrium? (2017-Gujarat) water are identical
(a) Presence of catalyst (c) The molar solubility of MY in water is
(b) Change in concentration of reactants less than of NY3
(c) Change in pressure The salts MY and NY3 are more soluble
(d) Change in temperature in 0.5 M KY than in pure water
25. For the reaction 𝐶𝑂(𝑔) + 𝐶𝑙(𝑔) ⇌ 𝐶𝑂𝐶𝑙2 (𝑔) 𝐾
𝐾𝑝 31. tant for 𝑁2 (𝑔) +
𝑐
𝑂2 (𝑔) ⇌ 2𝑁𝑂(𝑔) is K, the equilibrium
is equal to: (2017-Gujarat) 1 1
constant for 𝑁2 (𝑔) + 𝑂2 (𝑔) ⇌ 𝑁𝑂(𝑔) will
(a) (𝑅𝑇)2 2 2
1
(b) 𝑅𝑇 be: (2015 Re)
(a) 𝐾 2
(c) 𝑅𝑇
(b) 𝐾 1/2
(d) √𝑅𝑇 1
(c) 2 𝐾
26. Which of the following fluoro-compounds is (d) 𝐾
most likely to behave as a Lewis base?
(2016-II) 32. Which one of the following pairs of solution
(a) CF4 is not an acidic buffer? (2015 Re)
(b) SiF4 (a) H3PO4 and Na3PO4
(c) BF3 (b) HClO4 and NaClO4
(d) PF3 (c) CH3COOH and CH3COONa
(d) H2CO3 and Na2CO3
27. The solubility of AgCl(s) with solubility 33. If the value of an equilibrium constant for a
product 1.6 × 10–10 in 0.1 M NaCl solution particular reaction is 1.6 × 1012, then at
would be: (2016-II) equilibrium the system will contain: (2015)
(a) 1.6 × 10 M
–11
(a) Mostly reactants
(b) Zero (b) Mostly products
(c) 1.26 × 10–5 M (c) Similar amounts of reactants and
(d) 1.6 × 10–9 M products
28. The percentage of pyridine (C5H5N) that (d) All reactants
forms pyrimidine ion (C5H5N+H) in a 0.10 M 34. The Ksp of Ag2CrO4, AgBr and AgI are
aqueous pyridine solution (Kb for C5H5N = respectively, 1.1 × 10–12, 1.8 × 10–10, 5.0 ×
1.7 × 10–9) is : (2016-II) 10–13, 8.3 × 10–17. Which one of the following
(a) 0.77% salts will precipitate last if AgNO3 solution
39
is added to the solution containing equal 40. For the reversible reaction,
moles of NaCl, NaBr, NaI and Na2CrO4 ? 𝑁2 (𝑔) + 3𝐻2 (𝑔) ⇌ 2𝑁𝐻3 (𝑔) + 𝐻𝑒𝑎𝑡.
(2015) The equilibrium shifts in forward direction:
(a) AgCl (2014)
(b) AgBr (a) By decreasing the pressure
(c) Ag2CrO4 (b) By decreasing the concentration of N2(g)
(d) AgI and H2(g)
35. What is the pH of the resulting solution (c) By Increasing pressure and decreasing
when equal volumes of 0.1 M NaOH and temperature
0.01 M HCl are mixed? (2015 Re) (d) By increasing the concentration of
(a) 2.0 NH3(g)
(b) 7.0 41. Which of these is least likely to act as a
(c) 1.04
Lewis base? (2013)
(d) 12.65 (a) CO
36. Aqueous solution of which of the following (b) F–
compounds is the best conductor of electric (c) BF3
current? (2015 Re) (d) PF3
(a) Hydrochloric acid, HCl 42. KMnO4 can be prepared from K2MnO4 as
(b) Ammonia, NH3 per the reaction:
(c) Fructose, C6H12O6
3𝑀𝑛𝑂42− + 2𝐻2 𝑂 ⇌ 2𝑀𝑛𝑂4− + 𝑀𝑛𝑂2 + 4𝑂𝐻 −
(d) Acetic acid, C2H4O2
The reaction can go to completion by
37. Which of the following statements is correct removing OH– ions by adding: (2013)
for a reversible process in a state of (a) HCl
equilibrium? (2015)
(a) G° = –2.303 RT log K (c) CO2
(b) G° = 2.303 RT log K (d) SO2
(c) G = –2.303 RT log K
(d) G = 2.303 RT log K
38. Which of the following salts will give highest
pH in water? (2014)
(a) NaCl
(b) Na2CO3
(c) CuSO4
(d) 4KCl
39. Using the Gibb’s energy change, G° =
+63.3 kJ, for the following reaction,
𝐴𝑔2 𝐶𝑂3 (𝑠) ⇌ 2𝐴𝑔+ (𝑎𝑞) + 𝐶𝑂3− (𝑎𝑞)
The Ksp of Ag2CO3(s) in water at 25°C is (R =
5.314 JK–1 mol–1) (2014)
(a) 3.2 × 10–26
(b) 8.0 × 10–12
(c) 2.9 × 10–3
(d) 7.9 × 10–2

40
Answer Key

S1. Ans. (c) S30. Ans. (c)

S2. Ans. (a) S31. Ans. (b)

S3. Ans. (d) S32. Ans. (b)

S4. Ans. (b) S33. Ans. (b)

S5. Ans. (c) S34. Ans. (c)

S6. Ans. (a) S35. Ans. (d)

S7. Ans. (a) S36. Ans. (a)

S8. Ans. (b) S37. Ans. (a)

S9. Ans. (d) S38. Ans. (b)

S10. Ans. (a) S39. Ans. (b)

S11. Ans. (c) S40. Ans. (c)

S12. Ans. (d) S41. Ans. (c)

S13. Ans. (d) S42. Ans. (c)

S14. Ans. (a)

S15. Ans. (c)

S16. Ans. (c)

S17. Ans. (a)

S18. Ans. (a)

S19. Ans. (c)

S20. Ans. (c)

S21. Ans. (a)

S22. Ans. (c)

S23. Ans. (d)

S24. Ans. (a)

S25. Ans. (b)

S26. Ans. (d)

S27. Ans. (c)

S28. Ans. (d)

S29. Ans. (c)

41
Solutions
S1. Ans.(c) S6. Ans.(a)
Ka = Cα2 It is a mixture of weak acid and salt of

Ka = (0.1) × (0.01)2 its conjugate base. Hence, it is acidic


buffer.
Ka = 1 × 10-5
[Salt]
S2. Ans.(a) pH = pKa + log [Acid]

Acidic buffer is prepared by mixing weak 0.1


= 4.57 + log (0.01)
acid and its salt with strong base.
= 4.57 + 1
S3. Ans.(d)
For weak acid (i.e., CH3COOH) = 4.57

[H+] = Cα S7. Ans.(a)


1 3O2(g) ⇌ 2O3(g)
= 0.01 × 100
= 10-4 M
[O3 ]2
Kc = [O2 ]3
pH = − log H+ = −log 10-4 = 4

S4. Ans.(b) [O3]2 = Kc[O2]3 = 3 × 10-59 × (0.04)3

According to Henry's Law, [O ] = 1.9 × 10 = 19 × 10-64


p = KHx [O ] = 4.38 × 10
Where 'p' is partial pressure of gas in Concentration of O at equilibrium =
vapour phase. 4.38 × 10-32 M
S8. Ans.(b)
'KH' is Henry's Law constant. 1
𝑝𝐻 = 2 [𝑃𝑘𝑤 + 𝑃𝑘𝑎 − 𝑃𝑘𝑏 ]
'x' is mole fraction of gas in liquid.
1 1
= 7 + 𝑃𝑘𝑎 − 𝑝𝑘𝑏
2 2
Higher the value of KH at a given
1 1
pressure, lower is the solubility of the gas = 7 + 2 × 4.77 − 2 × 3.27

in the liquid. = 7.75


S9. Ans.(d)
∴ Solubility: Ar < CO2 < CH4 < HCHO
Let the solubility of Ni(OH)2 is s
S5. Ans.(c)
𝑁𝑖(𝑂𝐻)2 ⇌ 𝑁𝑖 2+ + 2𝑂𝐻 −
𝑠 2𝑠
CO2(g) + C(s) ⇌ 2CO(g) 𝑠

𝑁𝑎𝑂𝐻 → 𝑁𝑎 + 𝑂𝐻 −
∆ng = 2 – 1 = 1
0.1 0.1 0.1
Kp = Kc (RT)∆ng As Ksp is small 2s <<0.10
Kp = Kc(RT) Therefore (0.10 + 2s) ≈ 0.10
So total [OH]– = 0.10
[∵ Kp = 3]
Ionic product = [Ni2+] [OH–]2
Kp 3
Kc = RT
= 0.083×1000 2 × 10–15 = s(0.10)2
= 0.036 s = 2 × 10–13 M
S10. Ans.(a)
= 3.6 × 10-2

42
When HCl is passed through the 𝐻𝐹 + 𝐻2 𝑂 ⇋ 𝐹⊖ + 𝐻3 𝑂⊕
𝐴𝑐𝑖𝑑 𝐵𝑎𝑠𝑒 𝐶𝑜𝑛𝑗𝑢𝑔𝑎𝑡𝑒 𝑏𝑎𝑠𝑒 𝐶𝑜𝑛𝑗𝑢𝑔𝑎𝑡𝑒 𝑎𝑐𝑖𝑑
mixture of CaCl2, MgCl2 and NaCl, Cl–
ion concentration increases. As CaCl2 S16. Ans.(c)
and MgCl2 are more stable than NaCl. (i) 𝐶𝐻3 𝐶𝑂𝑂𝐻 + 𝑁𝑎𝑂𝐻 → 𝐶𝐻3 𝐶𝑂𝑂𝑁𝑎 + 𝐻2 𝑂
Hence ionic product [Na+] [Cl–] becomes Initial 25 mL 50 mL 0
more than solubility product. Thus, × 0.1 M × 0.1 M
precipitation of NaCl occurs (due to
= 2.5 mmol = 5 mmol
common ion effect)
S11. Ans.(c) Final 0 2.5 mmol 2.5 mmol

Acetic acid, CH3COOH, will react with This is basic solution due to NaOH.
sodium hydroxide, NaOH, to produce This is not basic buffer.
sodium acetate, CH3COONa, and water. (ii) 𝐶𝐻3 𝐶𝑂𝑂𝐻 + 𝑁𝑎𝑂𝐻 → 𝐶𝐻3 𝐶𝑂𝑂𝑁𝑎 + 𝐻2 𝑂
Sodium acetate is a salt of strong base Initial 100 mL 100 mL 0
and Weak acid therefore its salt solution
× 0.1 M × 0.1 M
is basic in nature.
= 10 mmol = 10 mmol
S12. Ans.(d)
Final 0 0 10 mmol
For salt AB
Hydrolysis of salt takes place.
𝐾𝑠𝑝 = (𝑠)2
This is not basic buffer.
𝑠 = √𝐾𝑠𝑝
(iii) 𝐻𝐶𝑙 + 𝑁𝐻4 𝑂𝐻 → 𝑁𝐻4 𝐶𝑙 + 𝐻2 𝑂
𝑠 = √4 × 10
Initial 100 mL 200 mL 0
𝑠 = 2 × 10−4 𝑀
× 0.1 M × 0.1 M
S13. Ans.(d)
= 10 mmol = 20 mmol
Given : K = 2 × 1013
Final 0 10 mmol 10 mmol
∆𝐺 = ∆𝐺° + 𝑅𝑇 𝑙𝑛𝑄
This will make basic buffer.
At equilibrium ∆𝐺 = 0, 𝑄 = 𝐾𝑒𝑞
(iv) 𝐻𝐶𝑙 + 𝑁𝑎𝑂𝐻 → 𝑁𝑎𝐶𝑙 + 𝐻2 𝑂
So ∆𝑟 𝐺° = −𝑅𝑇 𝑙𝑛𝐾𝑒𝑞 Initial 100 mL 100 mL 0
−1 −1
∆𝑟 𝐺° = −8.314 𝐽 𝑚𝑜𝑙 𝐾 × 300𝐾 × ln × 0.1 M × 0.1 M
13
(2 × 10 ) = 10 mmol = 10 mmol
S14. Ans.(a) Final 0 0 10 mmol
2+ −
𝐶𝑎(𝑂𝐻)2 ⇌ 𝐶𝑎 + 2𝑂𝐻 ⇒ This is Neutral solution.
𝑝𝐻 + 𝑝𝑂𝐻 = 14 S17. Ans.(a)
pH = 9, thus pOH = 14 – 9 = 5 𝐴2 (𝑔) + 𝐵2 (𝑔) ⇌ 𝑋2 (𝑔); ∆𝐻 = −𝑋 𝑘𝐽
[𝑂𝐻 − ] −5
= 10 𝑀 On increasing pressure, equilibrium
Thus [𝐶𝑎2+ ] =
10−5 shifts in a direction where pressure
2 decreases i.e. forward direction.
So, 𝐾𝑠𝑝 = [𝐶𝑎2+ ][𝑂𝐻 − ]2
On decreasing temperature, equilibrium
10−5 shifts in forward direction as it is
= (10−5 )2 = 0.5 × 10−15
2
exothermic direction.
S15. Ans.(c)
So, high pressure and low temperature
−𝐻 + +𝐻 +

𝑂𝐻 ← 𝐻2 𝑂 → 𝐻3 𝑂 ⊕ favours maximum formation of product.
Conjugate Conjugate S18. Ans.(a)
base acid 2.42×10−3
Solubility of BaSO4, 𝑠 = 233
(𝑚𝑜𝑙 𝐿−1 )

43
= 1.04 × 10−5 (𝑚𝑜𝑙 𝐿−1 ) S24. Ans.(a)
𝐵𝑎𝑆𝑂4 (𝑠) ⇌ 𝐵𝑎2+ (𝑎𝑞) + 𝑆𝑂42− (𝑎𝑞) Presence of catalyst does not affect the
equilibrium of the reaction. Catalyst
𝐾𝑠𝑝 = [𝐵𝑎2+ ][𝑆𝑂42− ] = 𝑠 2
only affect the rate of reaction.
= (1.04 × 10−5 )2
S25. Ans.(b)
= 1.08 × 10−10 𝑚𝑜𝑙 2 𝐿−2
𝐶𝑂(𝑔) + 𝐶𝑙2 (𝑔) ⇌ 𝐶𝑂𝐶𝑙2 (𝑔)
S19. Ans.(c)
∆𝑛 = −1
1
Meq of HCl = 75 × × 1 = 15 𝐾𝑝 = 𝐾𝑐 (𝑅𝑇)∆𝑛𝑔
5
1
– Meq of NaOH = 25 × 5 × 1 = 5 𝐾𝑝 1
= 𝑅𝑇
𝐾𝑐
– Meq of HCl in resulting solution = 10
– Molarity of [H+] in resulting mixture S26. Ans.(d)

=
10
=
1 Among the 4 options PF3 will behave as
100 10
1
a Lewis base due to presence of lone
pH= − log[𝐻 + ] = −𝑙𝑜𝑔 [10] = 1.0 pair of electrons in p-orbitals.
S20. Ans.(c) S27. Ans.(d)
2𝑁𝐻3 ⇌ 𝑁2 + 3𝐻2
1 𝑁𝑎𝐶𝑙(𝑎𝑞) ⇌ 𝑁𝑎+ 𝐶𝑙 −
𝐾1
Initially 0.1 M 0 0
𝑁2 + 𝑂2 ⇌ 2𝑁𝑂 𝐾2
3 At equilibrium 0 0.1 0.1 + S
3𝐻2 + 2 𝑂2 ⇌ 3𝐻2 𝑂 𝐾33
𝐴𝑔𝐶𝑙 ⇌ 𝐴𝑔+ + 𝐶𝑙 −
5
2𝑁𝐻3 + 𝑂
2 2
⇌ 2𝑁𝑂 + 3𝐻2 𝐾 0 0

𝐾=
𝐾2 𝐾33 At equilibrium 0 S 0.1 + S
𝐾1
𝐾𝑠𝑝 = 1.6 × 10−10 = [𝐴𝑔+ ] [𝐶𝑙 − ]
S21. Ans.(a)
= 𝑆(0.1 + 𝑆)
𝐴𝑔2 𝐶2 𝑂4 → 2𝐴𝑔+ + 𝐶2 𝑂42−
2𝑠 𝑠 1.6 × 10−10 = 𝑆 × 0.1
𝐾𝑠𝑝 = [𝐴𝑔+ ]2 [𝐶2 𝑂4 𝑆 = 1.6 × 10−9 𝑀
= (2s)2 (s) ; [Ag+ = 2.2 × 10–4] S28. Ans.(d)
= (2.2 × 10–4)2 (1.1 × 10–4); Pyrimidine is a very weak base so
[2s = 2.2 × 10–4] 𝐾𝑏 = 𝑐𝛼 2
= 4.84 ×10–8 × 1.1 × 10–4
𝐾𝑏
= 5.3 × 10–12 𝛼=√
𝑐
S22. Ans.(c)
The value of equilibrium constant is not 1.7 × 10−9
=√
changed in the presence of a catalyst in 0.9
the reaction at equilibrium. The catalyst
= 1.3 × 10−4
only increases the rates of reaction, it
does not affect the equilibrium constant. % 𝛼 = 0.013%
S23. Ans.(d) S29. Ans.(c)

𝐾𝑝 = 𝐾𝑐 (𝑅𝑇)∆𝑛 If large amount of KHSO4 would be


added then concentration of 𝑁𝑂2+ will
𝐾𝑝 5.8×105
𝐾𝑐 = (𝑅𝑇)∆𝑛 = (0.08314×298)2−4 decrease, rate of nitration will also
5.8×105
decrease.
= (0.08314×298)−2
S30. Ans.(c)
= 3.56 × 108 For 𝑀𝑌 ⇌ 𝑀+ + 𝑌 −
𝑆 𝑆
44
𝐾𝑠𝑝 = 6.2 × 10−13 S36. Ans.(a)
𝑆 = 7.9 × 10−7 𝑚𝑜𝑙 𝐿−1 HCl is strong acid and dissociated
completely into ions in aq. Solution.
For 𝑁𝑌3 ⇌ 𝑁 3+ + 3𝑌 −
S37. Ans.(a)
𝐾𝑠𝑝 1/4
𝐾𝑠𝑝 = 27𝑆 4 ⇒ 𝑆 = ( 27 ) ∵ ∆𝐺 = ∆𝐺° + 2.303 𝑅𝑇 log 𝑄
𝑆 = 3.89 × 10−4 𝑚𝑜𝑙 𝐿−1 At equilibrium
So, solubility of NY3 is more than ∆𝐺 = 0
solubility of MY in pure water. And 𝑄 = 𝐾𝑒𝑞
addition of KY decrease the solubility
because of common ion effect. So, ∆𝐺° = (−)2.303 𝑅𝑇 log 𝐾𝑒𝑞

S31. Ans.(b) S38. Ans.(b)


1 Highest pH is shown by alkali
If the reaction is multiplied by 2, then
So, Na2CO3 being salt of a strong base and
new equilibrium constant, 𝐾 ′ = 𝐾 1/2.
weak acid will exhibit highest pH.
S32. Ans.(b)
NaCl → neutral pH = 7
HClO4 and NaClO4 will not make a
buffer solution because a buffer solution CuSO4 → Acidic pH < 7
is formed by the mixture of a weak acid KCl → Neutral pH = 7
and its conjugate base and HClO4 is not S39. Ans.(b)
a weak acid. ∵ ∆𝐺° = −2.303 𝑅𝑇 log 𝐾 𝑝
S33. Ans.(b)
314 × 298 log 𝐾𝑠𝑝
Such big equilibrium constant indicates
large concentration of products so
equilibrium is mostly towards product. S40. Ans.(c)
S34. Ans.(c) On increasing pressure, equilibrium
According to solubility law: shifts in forward direction, where
2−
number of moles decreases, it is an
𝐴𝑔2 𝐶𝑟𝑂4 ⇌ 2𝐴𝑔+ + 𝐶𝑟𝑂4 example of exothermic reaction therefore
(2S)2 S decreasing temperature favours the
1.1 × 10−12 = 4𝑆 3 forward direction.

𝐴𝑔2 𝐶𝑟𝑂4 = 𝑆~10−3 S41. Ans.(c)


Lewis base species are those which are
𝐴𝑔𝐶𝑙 = 𝑆~10−5
electron deficient and does require
𝐴𝑔𝐵𝑟 = 𝑆~10−6 electron to complete their octet. BF3 is
𝐴𝑔𝐼 = 𝑆~10−8 an electron deficient species and is a
So, 𝐴𝑔2 𝐶𝑟𝑂4 will be the most soluble. Lewis acid.
S42. Ans.(c)
S35. Ans.(d)
HCl and SO2 are reducing agents so
∵ 1 mole of NaOH is completely
they can reduce 𝑀𝑛𝑂4− .
neutralized by 1 mole of HCl.
CO2 is neither oxidizing nor reducing
So, 0.01 mole is neutralized by 0.01 agent it will provide only acidic medium
mole of HCl. and shift the direction in forward
Left NaOH = 0.09 mole reaction towards completion.
0.09
[𝑂𝐻 − ] = = 0.045𝑀
2

𝑝𝑂𝐻 = − log[𝑂𝐻 − ] = 1.35


𝑝𝐻 = 14 − 𝑝𝑂𝐻 = 14 − 1.35 = 12.65

45
Chapter 7
Redox Reaction

1. The correct option for a redox couple is: Identify the incorrect option.
(2023) (2020 Covid Re-NEET)
(a) Both are oxidised forms involving same (a) 𝐶𝑙𝑂3− is +5
element. (b) 𝐾2 𝐶𝑟2 𝑂7 is +6
(b) Both are reduced forms involving same (c) 𝐻𝐴𝑢𝐶𝑙4 is +3
element. (d) 𝐶𝑢2 𝑂 is –1
(c) Both the reduced and oxidised forms
involve same element. 7. Which of the following reactions are
(d) Cathode and anode together. disproportionation reaction? (2019)
A. 2𝐶𝑢 → 𝐶𝑢 + 𝐶𝑢
+ 2+ 0
2. On balancing the given redox reaction, B. 3𝑀𝑛𝑂42− + 4𝐻 + → 2𝑀𝑛𝑂4− + 𝑀𝑛𝑂2 + 2𝐻2 𝑂
aCr2O2− 2−
7 + bSO3 (aq) + cH (aq) ⟶ 2aCr (aq)
+ 3+ ∆
c C. 2𝐾𝑀𝑛𝑂4 → 𝐾2 𝑀𝑛𝑂4 + 𝑀𝑛𝑂2 + 𝑂2
+ bsO2−
4 (aq) + 2H2O(𝑙), the coefficients a, b
D. 2𝑀𝑛𝑂4− + 3𝑀𝑛2+ + 2𝐻2 𝑂 → 5𝑀𝑛𝑂2 + 4𝐻 ⊕
and c are found to be, respectively-
(2023) Select the correct option from the following
(a) 3, 8, 1 (a) A and B only
(b) 1, 8, 3 (b) A, B and C
(c) 8, 1, 3
(d) 1, 3, 8
3. Which of the following reactions is a 8. The correct structure of tribromooctaoxide
decomposition redox reaction? (2022) is (2019)
(a) P4(s) + + 3OH − (aq) + 3H2O(𝑙) ⟶ PH3(g) +
3H2PO− 2 (aq)
(b) 2Pb(NO3)2(s) ⟶ 2PbO(s) + 4NO2(g) + (a)
O2(g)
(c) N2(g) + O2(g) ⟶ 2NO(g)
(d) Cl2(g) + 2OH − (aq) ⟶ ClO− (aq) + Cl− (aq) +
4H2O(𝑙) (b)
4. Which of the following reactions is the metal
displacement reaction? Choose the right
option. (2021)
∆ (c)
(a) 𝐶𝑟2 𝑂3 + 2𝐴𝑙 → 𝐴𝑙2 𝑂3 + 2𝐶𝑟
(b) 𝐹𝑒 + 2𝐻𝐶𝑙 → 𝐹𝑒𝐶𝑙2 + 𝐻2 ↑
(c) 2𝑃𝑏(𝑁𝑂3 )2 → 2𝑃𝑏𝑂 + 4𝑁𝑂2 + 𝑂2 ↑

(d) 2𝐾𝐶𝑙𝑂3 → 2𝐾𝐶𝑙 + 3𝑂2 (d)
5. What is the change in oxidation number of
carbon in the following reaction? (2020)
9. For the redox reaction
𝐶𝐻4 (𝑔) + 4𝐶𝑙2 (𝑔) → 𝐶𝐶𝑙4 (𝐼) + 4𝐻𝐶𝑙(𝑔)
𝑀𝑛𝑂4− + 𝐶2 𝑂42− + 𝐻 + → 𝑀𝑛2+ + 𝐶𝑂2 + 𝐻2 𝑂
(a) 0 to +4
The correct coefficients of the reactants for
(b) –4 to +4
the balanced equation are (2018)
(c) 0 to –4
𝑴𝒏𝑶− 𝑪 𝑶𝟐−
𝑯 +
(d) +4 to +4 𝟒 𝟐 𝟒
(a) 16 5 2
6. The oxidation number of the underlined (b) 2 5 16
atom in the following species. (c) 5 16 2
(d) 2 16 5
46
10. Hot concentrated sulphuric acid is a
moderately strong oxidizing agent. Which of
the following reactions does not show
oxidizing behavior? (2016-I)
(a) 𝐶 + 2𝐻2 𝑆𝑂4 → 𝐶𝑂2 + 2𝑆𝑂2 + 2𝐻2 𝑂
(b) 𝐶𝑎𝐹2 + 𝐻2 𝑆𝑂4 → 𝐶𝑎𝑆𝑂4 + 2𝐻𝐹
(c) 𝐶𝑢 + 2𝐻2 𝑆𝑂4 → 𝐶𝑢𝑆𝑂4 + 𝑆𝑂2 + 2𝐻2 𝑂
(d) 3𝑆 + 2𝐻2 𝑆𝑂4 → 3𝑆𝑂2 + 2𝐻2 𝑂
11. Assuming complete ionization, same moles
of which of the following compounds will
require the least amount of acidified KMnO4
for complete oxidation? (2015 Re)
(a) 𝐹𝑒(𝑁𝑂2 )2
(b) 𝐹𝑒𝑆𝑂4
(c) 𝐹𝑒𝑆𝑂3
(d) 𝐹𝑒𝐶2 𝑂4
12. In acidic medium, 𝐻2 𝑂2 changes 𝐶𝑟2 𝑂7−2 to
𝐶𝑟𝑂5 which has two (–O–O–) bonds.
Oxidation state of Cr in 𝐶𝑟𝑂5 is: (2014)
(a) +3
(b) +6
(c) –10
(d) +5

47
Answer Key
S1. Ans. (c)

S2. Ans. (d)

S3. Ans. (b)

S4. Ans. (a)

S5. Ans. (b)

S6. Ans. (d)

S7. Ans. (a)

S8. Ans. (a)

S9. Ans. (b)

S10. Ans. (b)

S11. Ans. (b)

S12. Ans. (b)

48
Solutions
S1. Ans.(c) Cl oxidation state is –1
Redox couples is both the reduced and x + 4 × (–1) = 0
oxidised form involve same element. x = +4
S2. Ans.(d) Thus, Change in oxidation state of
Reaction has to be balanced in acidic carbon is from –4 to +4.
medium 'O' atoms are balanced by S6. Ans.(d)
adding H2O and then H-atom is balanced
(a) 𝐶𝑙𝑂3−
by adding H+ ions and charge is balanced
by e− . x + 3 (–2) = –1

Oxidation: SO2− 2− x = +5
3 + H2O ⟶ SO4 + 2H +
+

2e− × 3 (b) 𝐾2 𝐶𝑟2 𝑂7


Reduction: Cr2O2−
7 + 14H+ + 6e ⟶ 2Cr3+

2(+1) + 2x + 7(–2) = 0
+ 7H2O X = +6
Cr2O2−
7 + 3SO2−
3 + 8H+ ⟶ 2Cr3+ + 3SO2−
4 + (c) 𝐻𝐴𝑢𝐶𝑙4
4H2O
(+1) + x + 4(–1) = 0
a=1
x = +3
b=3
(d) 𝐶𝑢2 𝑂
c=8
2(+1) + x = 0
S3. Ans.(b)
x = –2
Decomposition redox reaction leads to
breakdown of a compound into two or S7. Ans.(a)
more compounds at least one of which
must be in the elemental state with
change in oxidation number. A.
2Pb(NO2−
3 )2 (s) ⟶ 2PbO(s) + 4NO2(g) +
°
O2 (g)
S4. Ans.(a)
Aluminium is more electropositive than
B.
Cr, so it displaced chromium from
𝐶𝑟2 𝑂3.

𝐶𝑟2 𝑂3 + 𝐴𝑙 → 𝐴𝑙2 𝑂3 + 𝐶𝑟
C.
S5. Ans.(b)
∴ not a disproportionation
𝐶𝐻4 (𝑔) + 4𝐶𝑙2 (𝑔) → 𝐶𝐶𝑙4 (𝑙) + 4𝐻𝐶𝑙(𝑔)
In the given reaction
D.
Let Carbon oxidation number assumes
to be x S8. Ans.(a)
H oxidation state is +1 The correct structure of 𝐵𝑟3 𝑂8.
16
In 𝐶𝐻4 The oxidation no of bromine is 3
in this
x+4×1=0 structure
x = –4
In 𝐶𝐶𝑙4

49
S9. Ans.(b)
2𝑀𝑛𝑂4− + 5𝐶2 𝑂42− + 16𝐻 + →
2𝑀𝑛2+ + 10𝐶𝑂2 + 8𝐻2 𝑂
S10. Ans.(b)
𝐶𝑎𝐹2 + 𝐻2 𝑆𝑂4 → 𝐶𝑎𝑆𝑂4 + 2𝐻𝐹
This reaction is not a oxidation reaction
as none of the atom in the reaction is
showing any change in the oxidation
number. 𝐻2 𝑆𝑂4 , here in the reaction is
not acting as a reducing nor an
oxidizing agent.
S11. Ans.(b)
𝐹𝑒(𝑁𝑂2 )2 → 𝐹𝑒 3+ + 2𝑁𝑂3−
𝐹𝑒 2+ → 𝐹𝑒 3+ change in oxidation state =
1
2𝑁𝑂2− → 2𝑁𝑂3− change in oxidation state
=4
Total change in oxidation state = 5
So, 𝐹𝑒(𝑁𝑂2 )
number of moles.
𝐹𝑒(𝑁𝑂2 )2 w
acidic 𝐾𝑀𝑛𝑂4
For 𝐹𝑒𝑆𝑂4
𝐹𝑒 2+ → 𝐹𝑒 3+ change in oxidation state =
1
𝐹𝑒𝑆𝑂4 will need least amount of 𝐾𝑀𝑛𝑂4 .
S12. Ans.(b)
CrO5 has a very famous butterfly
structure

x + 4 (–1) + 1 (–2) = 0
x = +6

50
Chapter 8
Solutions

1. Which amongst the following aqueous 6. If 8 g of a non-electrolyte solute is dissolved


solution of electrolytes will have minimum in 114 g of n-octane to reduce its vapour
elevation in boiling point? Choose the pressure to 80%, the molar mass (in
correct option: (2023) 𝑔 𝑚𝑜𝑙– 1) of the solute is
(a) 0.05 M NaCl [Given that molar mass of n-octane is 114 g
(b) 0.1 M KCl 𝑚𝑜𝑙 −1 ] (2020 Covid Re-NEET)
(c) 0.1 M MgSO4 (a) 60
(d) 1 M NaCl (b) 80
2. The following solutions were prepared by (c) 20
dissolving 10 g of glucose (𝐶6 𝐻12 𝑂6 ) in 250 (d) 40
ml of water (P1), 10 g of urea (𝐶𝐻4 𝑁2 𝑂) in
7. Isotonic solutions have same
250 ml of water (P2) and 10 g of sucrose
(2020 Covid Re-NEET)
(𝐶12 𝐻22 𝑂11 ) in 250 ml of water (P3). The right
(a) Freezing temperature
option for the decreasing order of osmotic
(b) Osmotic pressure
pressure of these solutions is: (2021) (c) Boiling temperature
(a) 𝑃1 > 𝑃2 > 𝑃3 (d) Vapour pressure
(b) 𝑃2 > 𝑃3 > 𝑃1
(c) 𝑃3 > 𝑃1 > 𝑃2 For an ideal solution, the correct option is:
(d) 𝑃2 > 𝑃1 > 𝑃3 (2019)
(a) ∆𝑚𝑖𝑥 𝑆 = 0 at constant T and P
3. The correct option for the value of vapour (b) ∆𝑚𝑖𝑥 𝑉 ≠ 0 at constant T and P
pressure of a solution at 45°C with benzene
(c) ∆𝑚𝑖𝑥 𝐻 = 0 at constant T and P
to octane in molar ratio 3 : 2 is: (2021)
(d) ∆𝑚𝑖𝑥 𝐺 = 0 at constant T and P
[At 45°C vapour pressure of benzene is 280
mm Hg and that of octane is 420 mm Hg. 9. The mixture that forms maximum boiling
Assume Ideal gas] azeotrope is: (2019)
(a) 168 mm of Hg (a) Water + Nitric acid
(b) 336 mm of Hg (b) Ethanol + Water
(c) 350 mm of Hg (c) Acetone + Carbon disulphide
(d) 160 mm of Hg (d) Heptane + Octane

4. The mixture which shows positive deviation 10. If molality of the dilute solution is doubled,
from Raoult’s law is: (2020) the value of molal depressin constant (Kf)
(a) Benzene + Toluene will be: (2017-Delhi)
(b) Acetone + Chloroform (a) Unchanged
(c) Chloroethane + Bromoethane (b) Doubled
(d) Ethanol + Aceton (c) Halved
(d) Tripled
5. The freezing point depression constant (Kf)
of benzene is 5.12 K kg mol–1. The freezing 11. Which of the following is dependent on
point depression for the solution of molality temperature? (2017-Delhi)
0.078 m containing a non-electrolyte solute (a) Weight percentage
in benzene is (rounded off upto two decimal (b) Molality
places): (2020) (c) Molarity
(a) 0.80 K (d) Mole fraction
(b) 0.40 K 12. Toluene in the vapour phase is in
(c) 0.60 K equilibrium with a solution of benzene and
(d) 0.20 K toluene having mole fraction of toluene
51
0.50. If vapour pressure of pure benzene is (d) 0.0354
119 torr and that of toluene is 37.0 torr at 18. Which one of the following electrolytes has
the same temperature, mole fraction of
the same value of van’t Hoff’s factor (i) as
toluene in vapour phase will be:
that of 𝐴𝑙2 (𝑆𝑂4 )3 (if all are 100% ionized)?
(2018-Gujarat) (2015)
(a) 0.325
(a) 𝐾3 [𝐹𝑒(𝐶𝑁)6 ]
(b) 0.462
(b) 𝐴𝑙(𝑁𝑂3 )3
(c) 0.237
(c) 𝐾4 [𝐹𝑒(𝐶𝑁)6 ]
(d) 0.506
(d) 𝐾2 𝑆𝑂4
13. Which one of the following is incorrect for
19. Which one is not equal to zero for an ideal
ideal solution? (2016-II)
solution? (2015)
(a) ∆𝑃 = 𝑃𝑜𝑏𝑠 − 𝑃𝑐𝑎𝑙𝑐𝑢𝑙𝑎𝑡𝑒𝑑 𝑏𝑦 𝑅𝑎𝑜𝑢𝑙𝑡 ′ 𝑠 𝑙𝑎𝑤 = 0
(a) ∆𝑆𝑚𝑖𝑥
(b) ∆𝐺𝑚𝑖𝑥 = 0 (b) ∆𝑉𝑚𝑖𝑥
(c) ∆𝐻𝑚𝑖𝑥 = 0 (c) ∆𝑃 = 𝑃𝑜𝑏𝑠𝑒𝑟𝑣𝑒𝑑 − 𝑃𝑅𝑎𝑜𝑢𝑙𝑡
(d) ∆𝑈𝑚𝑖𝑥 = 0 (d) ∆𝐻𝑚𝑖𝑥
14. The van’t Hoff factor (i) for a dilute aqueous 20. The boiling point of 0.2 𝑚𝑜𝑙 𝑘𝑔−1 solution of
solution of the strong electrolyte barium X in water is greater than equimolal
hydroxide is: (2016-II) solution of Y in water. Which one of the
(a) 2 following statements is true in this case?
(b) 3 (2015)
(c) 0 (a) Molecular mass of X is greater than the
(d) 1 molecular mass of Y
15. At 100°C, the vapour pressure of a solution (b) Molecular mass of X is less than the
of 6.5 g of a solute in 100 g water is 732 molecular mass of Y
mm. If 𝐾𝑏 = 0.52, the boiling point of this (c) Y is undergoing dissociation in water
solution will be: (2016-I) while X undergoes no change
(a) 103°C (d) X is undergoing dissociation in water
(b) 101°C 21. Of the following 0.10 m aqueous solutions,
(c) 100°C which one will exhibit the largest freezing
(d) 102°C point depression? (2014)
16. Which of the following statements about the (a) 𝐶6 𝐻12 𝑂6
composition of the vapour over an ideal 1:1 (b) 𝐴𝑙2 (𝑆𝑂4 )3
molar mixture of benzene and toluene is (c) 𝐾2 𝑆𝑂4
correct? Assume that the temperature is at (d) 𝐾𝐶𝑙
25°C. 22. How many grams of concentrated nitric acid
(Given, vapour pressure data at 25°C, solution should be used to prepare 250 mL
benzene = 12.8 kPa, toluene = 3.85 kPa) of 2.0 M HNO3? The concentrated acid is
(2016) 70% HNO3. (2013)
(a) The vapour will contain equal amounts (a) 70.0 g of conc. HNO3
of benzene and toluene (b) 54.0 g conc. HNO3
(b) Not enough information is given to make (c) 45.0 g conc. HNO3
a prediction (d) 90.0 g conc. HNO3
(c) The vapour will contain a higher
percentage of benzene
(d) The vapour will contain a higher
percentage of toluene
17. What is the mole fraction of the solute in a
1.00 m aqueous solution? (2015 Re)
(a) 0.0177
(b) 0.177
(c) 1.770
52
Answer Key
S1. Ans. (a)

S2. Ans. (d)

S3. Ans. (b)

S4. Ans. (d)

S5. Ans. (b)

S6. Ans. (d)

S7. Ans. (b)

S8. Ans. (c)

S9. Ans. (a)

S10. Ans. (a)

S11. Ans. (c)

S11. Ans. (c)

S13. Ans. (b)

S14. Ans. (b)

S15. Ans. (b)

S16. Ans. (c)

S17. Ans. (a)

S18. Ans. (c)

S19. Ans. (a)

S20. Ans. (d)

S21. Ans. (b)

S22. Ans. (c)

53
Solutions
S1. Ans.(a) 𝑚𝐵 =
8×100
= 40 𝑔 𝑚𝑜𝑙 −1
20
i × M↓ ⟹ ∆Tb↓ S7. Ans.(b)
Electrolyte i×M Solutions that have same osmotic
NaCl 2 × 0.05 = 0.1 pressure at a given temperature are
KCl 2 × 1.0 = 0.2 called isotonic solutions.
S8. Ans.(c)
MgSO4 2 × 0.1 = 0.2
For ideal solution,
NaCl 2×1=2
∆𝑚𝑖𝑥 𝐻 = 0 , i.e., no heat should be
absorbed or evolved during mixing
S2. Ans.(d)
∆𝑚𝑖𝑥 𝑉 = 0
𝜋 = 𝑖𝐶𝑅𝑇
S9. Ans.(a)
10
𝑃1 = 1 × × 𝑅×𝑇 (For Glucose)
180 Maximum boiling azeotrope will forms
𝑃2 = 1 ×
10
×𝑅×𝑇 (For Urea) by solutions that show negative
60
deviation from Raoult’s law.
10
𝑃3 = 1 × 342 × 𝑅 × 𝑇 (For Sucrose) Water and Nitric acid → forms maximum
∴ 𝑃2 > 𝑃1 > 𝑃3 boiling azeotrope
S3. Ans.(b) S10. Ans.(a)
S4. Ans.(d) If molality of a dilute solution is
While ethanol-acetone mixture shows doubled, the value of molal depression
positive deviation due to weaker A-B constant (K ) will be unchanged because
interaction in comparison to A-A or B-B the value of molal depression constant
interaction. will depend only on number of solute
and solvent particle, as it is colligative
Pure ethanol molecules are hydrogen
property.
bonded. When acetone is added, its
molecules get in between the ethanol S11. Ans.(c)
molecules and break some of the Molarity will depend upon temperature
hydrogen bonds between them. as molarity will depend upon the volume
of the solution which change with
Thus, the intermolecular attractive
temperature.
interactions are weakens and the
solution shows positive deviation from S12. Ans.(c)
Raoult’s law. 𝑃𝑏° = 37 𝑡𝑜𝑟𝑟 , 𝑃𝑏° = 119 𝑡𝑜𝑟𝑟
S5. Ans.(b) 𝑃𝑡° 𝑋𝑡
𝑦1 = , 𝑋𝑡 = 0.5
𝑃𝑡° 𝑋𝑡 +𝑃𝑏° 𝑋𝑏
∆𝑇𝑓 = 𝐾𝑓 × 𝑚𝑜𝑙𝑎𝑙𝑖𝑡𝑦(𝑚)
37×0.5 37
∆𝑇𝑓 = 5.12 × 0.078 𝑦𝑡 = (37)(0.5)+(119)(0.5) = = 0.237
37+119

⇒ 0.3993 S13. Ans.(b)


= 0.40 K In case of an ideal solution ∆𝑆𝑚𝑖𝑥 ≠ 0
S6. Ans.(d) ∆𝑈𝑚𝑖𝑥 = 0 but ∆𝑆𝑚𝑖𝑥 ≠ 0
Assuming dilute solution According to 3rd law of thermodynamics:
∆𝑃 𝑛𝐵 𝑤𝐵 𝑚𝐴 ∆𝐺 = ∆𝐻 − 𝑇∆𝑆
= = .
𝑃𝐴0 𝑛𝐴 𝑚𝐵 𝑤𝐴
20 8 114
∆𝐺𝑚𝑖𝑥 ≠ 0
100
= 𝑚 . 114
𝐵

54
S14. Ans.(b) 𝐴𝑙2 (𝑆𝑂4 )3 will have 2𝐴𝑙 3+ + 3𝑆𝑂42− = 5
Ba(OH)2 is a strong electrolyte. It will entities exhibiting maximum depression
100% dissociate in aqueous solution: in freezing point.
𝐵𝑎(𝑂𝐻)2 ⇌ 𝐵𝑎2+ + 2𝑂𝐻 − S22. Ans.(c)
𝑊×1000
van’t Hoff factor becomes 3. 𝑀𝑜𝑙𝑎𝑟𝑖𝑡𝑦 = 𝑀×𝑉
(𝑚𝐿)
S15. Ans.(b) 𝑊 1000
2 = 63 × 250
Two colligative properties will be used to
63
solve the question. 𝑊= 2
𝑝0 −𝑝𝑠 𝑤 /𝑀
= 𝑤𝐴 /𝑀𝐴 Because 70% HNO3 ;
𝑝𝑠 𝐵 𝐵
70 63
760−732 6.5×𝑚
= 100/18 𝑀𝑎𝑠𝑠 𝑜𝑓 𝑎𝑐𝑖𝑑 × 100 = 2
732
𝑀𝑎𝑠𝑠 𝑜𝑓 𝑎𝑐𝑖𝑑 = 45 𝑔
m = 32 g/mol
S16. Ans.(c)
𝑃𝐵𝑒𝑛𝑧𝑒𝑛𝑒 = 𝑥 𝑃𝑜
𝑃𝑇𝑜𝑙𝑢𝑒𝑛𝑒 = 𝑥 𝑃𝑜
For an ideal 1:1 molar mixture of
Benzene and Toluene.
1 1
𝑋𝐵𝑒𝑛𝑧𝑒𝑛𝑒 = ; 𝑋𝑇𝑜𝑙𝑢𝑒𝑛𝑒 =
2 2
1
𝑃𝐵𝑒𝑛𝑧𝑒𝑛𝑒 = 2
1
𝑃𝑇𝑜𝑙𝑢𝑒𝑛𝑒 = 2
Thus, the vapour will contain a high
percentage of Benzene than Toluene.
S17. Ans.(a)
1 1
𝑋𝑠𝑜𝑙𝑢𝑡𝑖𝑜𝑛 = 55.5+1 = 56.5 = 0.0177
S18. Ans.(c)
𝐴𝑙2 (𝑆𝑂4 )3 ⇌ 2𝐴𝑙 3+ + 3𝑆𝑂42−
𝑖= 5

𝐾4 [𝐹𝑒(𝐶𝑁)6 ] ⇌ 4𝐾 + + [𝐹𝑒(𝐶𝑁)6 ]−
𝑖=5

S19. Ans.(a)
∆𝑆𝑚𝑖𝑥 > 0
As entropy increases after mixing.
S20. Ans.(d)
X must be going dissociation in water
thus increasing vant’s Hoff factor (i) as
after dissolution the number of solute
particles increases.
S21. Ans.(b)
Depression in freezing point depends on
vant’s Hoff factor which depends on
dissociation entities.
So, among 4 options

55
Chapter 9
Electrochemistry

1. The E ⊝ values for (c) 3.34 cm-1


Al+/Al = +0.55 V and Tl+/Tl = −0.34 V (d) 1.34 cm-1
Al3+/Al = −1.66 V and T3+/Tl = +1.26 V 5. Given below are two statements: One is
Identify the incorrect statement labelled as Assertion A and the other is
(2023) labelled as Reason R:
(a) Al is more electropositive than Tl Assertion: In equation ∆rG = −nFEcell, vlaue
(b) Tl3+ is a good reducing agent than Tl1+ of ∆rG depends on n.
(c) Al+ is unstable in solution Reason: Ecell is an intensive property and
(d) Al+ can be easily oxidised Tl than Tl3+ ∆rG is an extensive property.
2. Molar conductance of an electrolyte In the light of the above statements, choose
increase with dilution according to the the correct answer from the options given
equation: below: (2023)
⋀m = ⋀°m − A√c (a) Both A and R are true but R is NOT the
Which of the following statements are true? correct explanation of A.
(a) This equation applies to both strong and (b) A is true but R is false.
weak electrolytes. (c) A is false but R is true.
(b) Value of the constant A depends upon (d) Both A and R are true R R is the correct
the nature of the solvent. explanation of A.
(c) Value of constant A is same for both 6. Two half cell reactions are given below:
BaCl2 and MgSO4. °
3+ = −1.81 V
(d) Value of constant A is same for both °
2Al3+ + 6e− ⟶ 2Al(s), EAl/Al3+ = +1.66 V
BaCl2 and Mg(OH)2.
The standard EMF of a cell with feasible
Choose the most appropriate answer from
redox reaction will be: (2022)
the options given below: (2023)
(a) −3.47 V
(a) A and B only (b) +7.09 V
(b) A, B and C only (c) +0.15 V
(c) B and C only (d) +3.47 V
(d) B and D only
7. Standard electrode potential for the cell
3. The correct value of cell potential in volt for with cell reaction
the reaction that occurs when the following Zn(s) + Cu2+(aq) ⟶ Zn2+(aq) + Cu(s)
two half cells are connected, is is 1.1 V. Calculate the standard Gibbs
Fe2+(aq) + 2e− ⟶ Fe(s), E° = −0.44 V energy change for the cell reaction.
Cr2O2−
7 (aq) + 14H + 6e ⟶ 2Cr + 7H2O
+ − 3+
(Given: F = 96487 C mol-1) (2022)
E° = +1.33 V (2023) (a) −200.27 J mol-1
(a) +1.77 V (b) −200.27 kJ mol-1
(b) +2.65 V (c) −212.27 kJ mol-1
(c) +0.01 V (d) −212.27 J mol-1
(d) +0.89 V
8. At 298 K, the standard electrode potentials
4. The conductivity of centimolar solution of of Cu2+/Cu, Zn2+/Zn, Fe2+/Fe and Ag+/Ag
KCl at 25°C is 0.0210 ohm-1 cm-1 and the are 0.34 V, −0.76 V, −0.44 V and 0.80 V,
resistance of the cell containing the solution respectively.
at 25°C is 60 ohm. The value of cell constant On the basis of standard electrode
is- (2023) potential, predict which of the following
(a) 3.28 cm -1
reaction cannot occur?
(b) 1.26 cm-1 (a) CuSO4(aq) + Zn(s) ⟶ ZnSO4(aq) + Cu(s)

56
(b) CuSO4(aq) + Fe(s) ⟶ FeSO4(aq) + Cu(s) (a) Oxygen gas
(c) FeSO4(aq) + Zn(s) ⟶ ZnSO4(aq) + Fe(s) (b) H2S gas
(d) 2CuSO4(aq) + 2Ag(s) ⟶ 2Cu(s) + (c) SO2 gas
Ag2SO4(aq) (d) Hydrogen gas
9. Given below are half cell reactions: 14. The number of Faradays (F) required to
MnO− 4 + 8H + 5e ⟶ Mn
+ − 2+ + 4H O,
2 produce 20 g of calcium from molten CaCl2
°
EMn2+/MnO−4 = −1.510 V (Atomic mass of Ca = 40 g mol–1) is: (2020)
1
O2 + 2H+ + 2e− ⟶ H 2O (a) 2
2
(b) 3
EO° 2 /H2 O = +1.223 V
(c) 4
will the permanganate ion, MnO− 4 liberate O2 (d) 1
from water in the presence of an acid?
(2022) 15. Identify the reaction from following having
° top position in EMF series (Std. red.
(a) Yes, because Ecell = +0.287 V
° Potential) according to their electrode
(b) No, because Ecell = −0.287 V
°
potential at 298 K. (2020 Covid Re-NEET)
(c) Yes, because Ecell = +2.733 V
°
(a) 𝐹𝑒 2+ + 2𝑒 − → 𝐹𝑒(𝑠)
(d) No, because Ecell = −2.733 V (b) 𝐴𝑢.3+ + 3𝑒 − → 𝐴𝑢(𝑠)
10. Find the emf of the cell in which the (c) 𝐾 + + 𝑙𝑒 − → 𝐾(𝑠)
following reaction takes place at 298 K (d) 𝑀𝑔2+ + 2𝑒 − → 𝑀𝑔(𝑠)
Ni(s) + 2Ag+(0.001 M) ⟶ Ni2+(0.001 M) + 16. In a typical fuel cell, the reactants (R) and
2Ag(s) product (P) are (2020 Covid Re-NEET)
2.303 RT
(Given that Ec° = 0.059 at (a) 𝑅 = 𝐻2(𝑔) , 𝑂2(𝑔) ; 𝑃 = 𝐻2 𝑂(𝑙)
F
298 K) (2022) (b) 𝑅 = 𝐻2(𝑔) , 𝑂2(𝑔) , 𝐶𝑙2(𝑔) ; 𝑃 = 𝐻𝐶𝑙𝑂4(𝑎𝑞)
(a) 1.0385 V (c) 𝑅 = 𝐻2(𝑔) , 𝑁2(𝑔) ; 𝑃 = 𝑁𝐻3(𝑎𝑞)
(b) 1.385 V (d) 𝑅 = 𝐻2(𝑔) , 𝑂2(𝑔) ; 𝑃 = 𝐻2 𝑂2(𝑙)
(c) 0.9615 V
(d) 1.05 V 17. For a cell involving one electron E°cell =
0.59 V at 298 K, the equilibrium constant
11. The molar conductance of NaCl, HCl and for the cell reaction is: (2019)
CH3COONa at infinite dilution are 126.45, 2.303𝑅𝑇
[𝐺𝑖𝑣𝑒𝑛 𝑡ℎ𝑎𝑡 = 0.059𝑉 𝑎𝑡 𝑇 = 298𝐾]
426.16 and 91.0 S cm2 mol–1 respectively. 𝐹
The molar conductance of CH3COOH at (a) 1.0 × 102
infinite dilution is. Choose the right option (b) 1.0 × 105
for your answer. (2021) (c) 1.0 × 1010
(a) 390.71 𝑆 𝑐𝑚 𝑚𝑜𝑙
2 −1 (d) 1.0 × 1030
(b) 698.28 𝑆 𝑐𝑚2 𝑚𝑜𝑙 −1 18. For the cell reaction
(c) 540.48 𝑆 𝑐𝑚2 𝑚𝑜𝑙 −1 2𝐹𝑒 3+ (𝑎𝑞) + 2𝐼 − (𝑎𝑞) → 2𝐹𝑒 2+ (𝑎𝑞) + 𝐼2 (𝑎𝑞)
(d) 201.28 𝑆 𝑐𝑚2 𝑚𝑜𝑙 −1 ⊖
𝐸𝑐𝑒𝑙𝑙 = 0.24 𝑉 at 298 K. The standard Gibbs
12. The molar conductivity of 0.007 M acetic energy (∆𝑟 𝐺 ⊖ ) of the cell reaction is:
acid is 20 S cm2 mol–1. What is the [Given that Faraday constant F = 96500 C
dissociation constant of acetic acid? mol–1] (2019)
Choose the correct option. (2021) (a) –46.32 kJ mol –1
° 2
Λ + = 350 𝑆 𝑐𝑚 𝑚𝑜𝑙 −1 (b) –23.16 kJ mol–1
[ ° 𝐻 ] (c) 46.32 kJ mol–1
Λ 𝐶𝐻3 𝐶𝑂𝑂− = 50 𝑆 𝑐𝑚2 𝑚𝑜𝑙 −1
(d) 23.16 kJ mol–1
(a) 2.50 × 10−4 𝑚𝑜𝑙 𝐿−1
(b) 1.75 × 10−5 𝑚𝑜𝑙 𝐿−1 19. Consider the change in oxidation state of
(c) 2.50 × 10−5 𝑚𝑜𝑙 𝐿−1 Bromine corresponding to different emf
(d) 1.75 × 10−4 𝑚𝑜𝑙 𝐿−1 values as shown in the diagram below:
13. On electrolysis of dil sulphuric acid using
Platinum (Pt) electrode, the product
obtained at anode will be: (2020)

57
24. Zinc can be coated on iron to produce
Then the species undergoing galvanized iron but the reverse is not
disproportionation is: (2018) possible. It is because: (2016-II)
(a) 𝐵𝑟𝑂3− (a) Zinc has lower negative electrode
(b) 𝐵𝑟𝑂4− potential than iron
(c) 𝐻𝐵𝑟𝑂 (b) Zinc has higher negative electrode
(d) 𝐵𝑟2 potential than iron
(c) Zinc is lighter than iron
20. In the electrochemical cell
(d) Zinc has lower melting point than iron
𝑍𝑛|𝑍𝑛𝑆𝑂4 (0.01𝑀)||𝐶𝑢𝑆𝑂4 (1.0𝑀)|𝐶𝑢, the emf of
this Daniel cell is E1. When the 25. The number of electrons delivered at the
concentration of 𝑍𝑛𝑆𝑂4 is changed to 1.0 M cathode during electrolysis by a current of
and that of 𝐶𝑢𝑆𝑂4 changed to 0.01 M, the 1 ampere in 60 seconds is: (2016-II)
emf changes to E2. From the following, (charge on electron = 1.60 × 10 C)
–19

which one is the relationship between E1 (a) 3.75 × 1020


and E2? (Given,
𝑅𝑇
= 0.059) (b) 7.48 × 1023
𝐹
(c) 6 × 1023
(2017-Gujarat)
(d) 6 × 1020
(a) 𝐸2 = 0 ≠ 𝐸
(b) 𝐸1 = 𝐸2 26. If the E°cell for a given reaction has a
(c) 𝐸1 < 𝐸2 negative value, which of the following gives
(d) 𝐸1 > 𝐸2 the correct relationships for the values of
∆𝐺° and 𝐾𝑒𝑞 ? (2016-II)
21. Given that Λ°𝑚 = 133.4, 5 𝑐𝑚2 𝑚𝑜𝑙 −1 (𝐴𝑔𝑁𝑂3 );
(a) ∆𝐺 < 0; 𝐾𝑒𝑞 > 1
0
Λ°𝑚 = 149.9 𝑆 𝑐𝑚2 𝑚𝑜𝑙 −1 (𝐾𝐶𝑙);
Λ°𝑚 = 144.9 𝑆 𝑐 (b) ∆𝐺 0 < 0; 𝐾𝑒𝑞 < 1
conductivity at infinite dilution for AgCl is:
(2017-Gujarat) (d) ∆𝐺 0 > 0; 𝐾𝑒𝑞 > 1
(a) 132 𝑆 𝑐𝑚 𝑚𝑜𝑙
2 −1
27. The molar conductivity of a 0.5 mol dm–3
(b) 140 𝑆 𝑐𝑚2 𝑚𝑜𝑙 −1 solution of AgNO3 with electrolytic
(c) 138 𝑆 𝑐𝑚2 𝑚𝑜𝑙 −1 conductivity of 5.76 × 10–3 S cm–1 at 298 K
(d) 134 𝑆 𝑐𝑚2 𝑚𝑜𝑙 −1 is: (2016-II)
22. The zinc/silver oxide cell is used in electric (a) 0.086 S cm mol
2 –1

watches. The reaction is as following: (b) 28.8 S cm2 mol–1


𝑍𝑛2+ + 2𝑒 − → 𝑍𝑛 ; 𝐸° = −0.760 𝑉 (c) 2.88 S cm2 mol–1
𝐴𝑔2 𝑂 + 𝐻2 𝑂 + 2𝑒 → 2𝐴𝑔 + 2𝑂𝐻 − ;
− (d) 11.52 S cm2 mol–1
𝐸° = 0.344 𝑉 28. The pressure of H2 required to make the
If F is 96,500 C mol–1, ∆𝐺° of the cell will be: potential of H2 electrode zero in pure water
(2017-Gujarat) at 298 K is: (2016-I)
(a) 413.21 kJ mol–1 (a) 10–4 atm
(b) 113.072 kJ mol–1 (b) 10–14 atm
(c) –213.072 kJ mol–1 (c) 10–12 atm
(d) 4313.082 kJ mol–1 (d) 10–10 atm

23. During the electrolysis of molten sodium 29. A device that converts energy of combustion
chloride, the time required to produce 0.10 of fuels like hydrogen and methane, directly
mol of chlorine gas using a current of 3 into electrical energy is known as: (2015)
amperes is: (2016-II) (a) Electrolytic cell
(a) 220 minutes (b) Dynamo
(b) 330 minutes (c) Ni-Cd cell
(c) 55 minutes (d) Fuel cell
(d) 110 minutes 30. The pair of compounds that can exist
together is: (2014)
(a) 𝐻𝑔𝐶𝑙2 , 𝑆𝑛𝐶𝑙2

58
(b) 𝐹𝑒𝐶𝑙2 , 𝑆𝑛𝐶𝑙2 36. A button cell used in watches functions as
(c) 𝐹𝑒𝐶𝑙3 , 𝐾𝐼 following
(d) 𝐹𝑒𝐶𝑙3 , 𝑆𝑛𝐶𝑙2 𝑍𝑛(𝑠) + 𝐴𝑔2 𝑂(𝑠) + 𝐻2 𝑂(𝑙) ⇌
2𝐴𝑔(𝑠) + 𝑍𝑛2+ (𝑎𝑞) + 2𝑂𝐻 − (𝑎𝑞).
31. Using the Gibbs energy change, ∆𝐺 0 =
If half cell potentials are
+63.3 𝑘𝐽, for the following reaction,
𝑍𝑛2+ (𝑎𝑞) + 2𝑒 − → 𝑍𝑛(𝑠) ; 𝐸° = – 0.76 𝑉
𝐴𝑔2 𝐶𝑂3 (𝑠) ⇌ 2𝐴𝑔+ (𝑎𝑞) + 𝐶𝑂32− (𝑎𝑞) the 𝐾𝑠𝑝 of
𝐴𝑔2 𝑂(𝑠) + 𝐻2 𝑂(𝑙) + 2𝑒 − → 2𝐴𝑔(𝑠) + 2𝑂𝐻 − (𝑎𝑞)
𝐴𝑔2 𝐶𝑂3 (𝑠) in water at 25°C is: (2014)
𝐸° = 0.34 𝑉 . The cell potential will be:
(𝑅 = 8.314 𝐽𝐾 −1 𝑚𝑜𝑙 −1 )
(2013)
(a) 8.0 × 10–12
(a) 1.10 V
(b) 2.9 × 10–3
(b) 0.42 V
(c) 7.9 × 10–2
(c) 0.84 V
(d) 3.2 × 10–26
(d) 1.34 V
32. When 0.1 mol 𝑀𝑛𝑂42− is oxidized, the
quantity of electricity required to completely
oxidise 𝑀𝑛𝑂42− to 𝑀𝑛𝑂4− is: (2014)
(a) 96500 C
(b) 2 × 96500 C
(c) 9650 C
(d) 96.50 C
33. The weight of silver (atomic weight = 108)
displaced by a quantity of electricity which
displaces 5600 mL of O at STP will be:

(a) 10.8 g
(b) 54.0 g
(c) 108.0 g
(d) 5.4 g
34. At 25°C, molar conductance of 0.1 molar
aqueous solution of ammonium hydroxide
is 9.54 ohm–1 cm2 mol–1 and at infinite
dilution its molar conductance is
238 ohm–1 cm2 mol–1. The degree of
ionization of ammonium hydroxide at the
same concentration and temperature is:
(2013)
(a) 2.080%
(b) 20.800%
(c) 4.008%
(d) 40.800%
35. A hydrogen gas electrode is made by dipping
platinum wire in a solution of HCl of pH =
10 and by passing hydrogen gas around the
platinum wire at one atm pressure. The
oxidation potential of electrode would be?
(2013)
(a) 0.059 V
(b) 0.59 V
(c) 0.118 V
(d) 1.18 V

59
Answer Key
S1. Ans. (b) S33. Ans. (c)

S2. Ans. (d) S34. Ans. (c)

S3. Ans. (a) S35. Ans. (b)

S4. Ans. (b) S36. Ans. (a)

S5. Ans. (d)

S6. Ans. (d)

S7. Ans. (c)

S8. Ans. (d)

S9. Ans. (a)

S10. Ans. (a)

S11. Ans. (a)

S12. Ans. (b)

S13. Ans. (a)

S14. Ans. (d)

S15. Ans. (b)

S16. Ans. (a)

S17. Ans. (c)

S18. Ans. (a)

S19. Ans. (c)

S20. Ans. (d)

S21. Ans. (c)

S22. Ans. (c)

S23. Ans. (d)

S24. Ans. (b)

S25. Ans. (a)

S26. Ans. (c)

S27. Ans. (d)

S28. Ans. (b)

S29. Ans. (d)

S30. Ans. (b)

S31. Ans. (a)

S32. Ans. (c)


60
Solutions
S1. Ans. (b) ∎ For, FeSO4(aq) + Zn(s) ⟶ ZnSO4(aq) +
Tl3 act as an oxidising agent not reducing Fe(s)
agent. °
Ecell °
= Ecathode °
− Eanode
S2. Ans. (d) = −0.44 V – (−0.76 V)
B and D statements are correct. = 0.32 V
S3. Ans. (a) ∎ For, 2CuSO4(aq) + 2Ag(s) ⟶ 2Cu(s) +
° Ag2SO4(aq)
Ecell = EC° − EA°
°
Ecell = 0.34 V – 0.80 V
= (1.33) – (−0.44)
= −0.46 V
= +1.77 V
∎ For, CuSO4(aq) + Zn(s) ⟶ ZnSO4(aq) +
S4. Ans. (b)
Cu(s)
Centimolar solution = 1/100 M = 0.01 M °
Ecell = 0.34 V – (−0.76 V)
Conductivity (k) = 0.0210 ohm-1 cm-1
= 1.1 V
Resistance (R) = 60 ohm
∎ Foe, CuSO4(aq) + Fe(s) ⟶ FeSO4(aq) +
l
k = 1/R( ) Cu(s)
A
°
⟹ 0.0210 = 1/60( ) ⟹
l 𝑙
= 1.26 cm-1 Ecell = 0.80 V – (−0.44 V)
A 𝐴
= 1.24 V
S5. Ans. (d)
S9. Ans. (a)
∆rG = −nFEcell
∎ MnO−
4 + 8H + 5e ⟶ Mn + 4H2O -(i)
+ − 2+
Ecell is an intensive property and ∆ is an
° °
extensive property as it depends on EMnO − /Mn2+
4 O−
4
= 1.51 V
number of e− transferred in cell reaction. 1
∎ H2O ⟶ 2O2 + 2H+ + 2e− -(ii)
S6. Ans. (d)
° EO° 2 /H2 O = 1.223 V
Since, EOP of Al is more than Co2+, so at
anode Al will oxidise and at cathode Co3+ Using 2 × (i) + 5 × (ii), net cell reactions
will reduce. is
° ° ° 5
Ecell = (ECathode )RP − (EAnode )RP 2MnO−
4 + 6H ⟶ 2Mn
+ 2+ + O2 + 3H2O
2
° °
= ECo3+ /Co2+ − EAl3+ /Al
°
Ecell °
= EC° − EA° = EMnO °
− /Mn2+ − EO /H O
2 2
4

= (1.81) – (−1.66) = 1.51 – 1.223 = 0.287 V


= +3.47 V °
Since Ecell > 0, therefore net cell reaction
S7. Ans. (c) is spontaneous and so MnO− 4 liberate O2

Zn(s) + Cu2+(aq) ⟶ Zn2+(aq) + Cu(s) from H2O in presence of an acid.


° S10. Ans. (a)
Ecell = 1.1 V
° Ni(s) + 2Ag+(0.001 M) ⟶ Ni2+(0.001 M) +
∆G° = −nFEcell
2Ag(s)
∴n=2 °
Ecell = 10.5 V
∆G° = −2× 96487 × 1.1 [ni2+ ]
° ° 0.059
Ecell = Ecell − log
∆G° = −212271.4 J mol-1 n [Ag+ ]2

∆ G° = −212.27 kJ mol-1 0.059 (10−3 )


⟹ 10.5 – 2
log (10 −3 )2
S8. Ans. (d) 0.059
° ⟹ 10.5 – log(10)3
For a reaction to be spontaneous, Ecell 2

must be positive. ⟹ 10.5 – 0.0295 × 3


⟹ 10.5 – 0.0885
61
⟹ 10.4115 V Number of required moles = mass/molar
S11. Ans.(a) mass
Λ 𝑁𝑎𝐶𝑙 = Λ 𝑁𝑎+ + Λ 𝐶𝑙− Molar mass of Ca is 40 g/mol and
required of Ca is 20 g
Λ 𝐻𝐶𝑙 = Λ 𝐻+ + Λ 𝐶𝑙−
Hence number of moles = 20/40
Λ 𝐶𝐻3 𝐶𝑂𝑂𝑁𝑎 = Λ Na+ + Λ 𝐶𝐻3 𝐶𝑂𝑂−
= 0.5 mol
Let, Λ 𝑁𝑎+ = 𝑥, Λ 𝐶𝑙− = 𝑦, Λ 𝐻+ = Λ 𝐶𝐻3 𝐶𝑂𝑂−=𝑤
Electricity required to produce 1 mol of
Given, calcium = 2F
x + y = 126.45 …(i) The electricity required to produce 0.5
y + z = 426.16 …(ii) mol of calcium = 0.5 × 2F
x + w = 91 …(iii) = 1F
From the above 3 equations, value of S15. Ans.(b)
z +w = 390.71d 𝐴𝑢3+ + 3𝑒 − → 𝐴𝑢(𝑠) 𝐸° = 1.40 𝑉
S12. Ans.(b) 𝐹𝑒 2+ + 2𝑒 − → 𝐹𝑒(𝑠) 𝐸° = −0.44 𝑉
Λ°𝑀(𝐶𝐻3 𝐶𝑂𝑂𝐻) = Λ°𝑀(𝐻+ ) + Λ°𝑀(𝐶𝐻3 𝐶𝑂𝑂−) 𝑀𝑔2+ + 2𝑒 − → 𝑀𝑔(𝑠) E°=-2.36 V
= 350 + 50 = 400 S cm2 mol–1 𝐾 + + 1𝑒 − → 𝐾(𝑠) 𝐸° = −2.93 𝑉
Λ𝐶 𝐴𝑢3+ occupies the top position in the
𝛼 = Λ𝑀
°
𝑀 electrochemical series.
20
𝑎= = 5 × 10−2 S16. Ans.(a)
400

𝐾𝑎(𝐶𝐻3 𝐶𝑂𝑂𝐻) Cell reaction involved in hydrogen-


oxygen fuel cell is
= 0.007 × (5 × 10 ) = 1.75 × 10 𝑚𝑜𝑙 𝐿
2𝐻2 (𝑔) + 𝑂2 (𝑔) → 2𝐻2 𝑂(𝑙)
S13. Ans.(a)
Thus, 𝑅 = 𝐻2 (𝑔), 𝑂2 (𝑔); 𝑃 = 𝐻2 𝑂(𝑙)
The following reactions take place on
S17. Ans.(c)
electrolysis of dilute sulphuric acid on
0.059
using pt electrodes are: 𝐸𝑐𝑒𝑙𝑙 = 𝐸°𝑐𝑒𝑙𝑙 − 𝑙𝑜𝑔𝑄 …(i)
𝑛
At cathode: At equilibrium, 𝑄 = 𝐾𝑒𝑞 and 𝐸𝑐𝑒𝑙𝑙 = 0)
+ − (𝑎𝑞)
4𝐻 + 4𝑒 → 2𝐻2 (𝑔) 0.059
0°𝐸°𝑐𝑒𝑙𝑙 − 1
𝑙𝑜𝑔𝐾𝑒𝑞 (from equation (i))
At anode:
𝐸°𝑐𝑒𝑙𝑙 0.59
2𝐻2 𝑂(𝑙) → 𝑂2 (𝑔) + 4𝐻 + (𝑎𝑞) + 4𝑒 − 𝑙𝑜𝑔𝐾𝑒𝑞 = 0.059
= 0.059 = 10

Thus, the product obtained at anode is 𝐾𝑒𝑞 = 1010 = 1 × 1010


O 2. S18. Ans.(a)
S14. Ans.(d) Θ
∆𝐺 Θ = −𝑛𝐹 𝐸𝑐𝑒𝑙𝑙
1 equivalent of any substance is = −2 × 96500 × 0.24
deposited by 1 F of charge. −46320
= −46320 𝐽𝑚𝑜𝑙 −1 =
We have, 20 g calcium 1000

The balance reaction will = −46.32 𝑘𝐽/𝑚𝑜𝑙

The charge on Ca in CaCl2 S19. Ans.(c)


+1 0
Cl has –1 charge so that 0
𝐻𝐵𝑟𝑂 → 𝐵𝑟2 , 𝐸𝐻𝐵𝑟𝑂/𝐵𝑟 = 0.595 𝑉′
2

Ca + 2(–1) = 0 +1 +5
°
𝐻𝐵𝑟𝑂 → 𝐵𝑟𝑂3− , 𝐸𝐵𝑟𝑂3 /𝐻𝐵𝑟𝑂
= 1.5 𝑉
Ca = 2
°
𝐸𝑐𝑒𝑙𝑙 for the disproportionation of 𝐻𝐵𝑟𝑂,
We have to get Ca from Ca2+

62
° ° ° − 35.5
𝐸𝑐𝑒𝑙𝑙 = 𝐸𝐻𝐵𝑟𝑂/𝐵𝑟2
− 𝐸𝐵𝑟𝑂3 /𝐻𝐵𝑟𝑂 0.1 × 71 = × 3×𝑡 as x factor 2
96500
= 1.595 − 1.5 𝑡 = 110 𝑚𝑖𝑛
= 0.095 𝑉 = +𝑣𝑒 S24. Ans.(b)
Hence, option (c) is correct answer. Zn have a higher (–ve) electrode
S20. Ans.(d) potential that is more reactive than Fe.
𝑍𝑛|𝑍𝑛𝑆𝑂4 (0.01𝑀)||𝐶𝑢𝑆𝑂4 (1.0𝑀)|𝐶𝑢 It is coated on iron substances to
provide resistance against rusting such
° 2.303𝑅𝑇 (0.01)
∴ 𝐸1 = 𝐸𝑐𝑒𝑙𝑙 − 2×𝐹
× log 1 a process is called galvanization. But in
When concentrations are changed reverse, that is Fe cannot be coated on
2.303𝑅𝑇 1
Zn, as corrosion will occur. In above, Zn
°
∴ 𝐸2 = 𝐸𝑐𝑒𝑙𝑙 − 2𝐹
× log 0.01 displaces Fe from its salt solution.
𝑖. 𝑒, 𝐸1 > 𝐸2 S25. Ans.(a)
S21. Ans.(c) According to Faraday’s law
Λ°𝑚 (𝐴𝑔𝐶𝑙) = Λ°𝑚 (𝐴𝑔𝑁𝑂3 ) + Λ°𝑚 (𝐾𝐶𝑙) − Q = ne
Λ°𝑚 (𝐾𝑁𝑂3 ) Q = it
2 −1
= (133.4 + 149 − 144.9) 𝑆 𝑐𝑚 𝑚𝑜𝑙 ne = it
2 −1
= 138.4 𝑆 𝑐𝑚 𝑚𝑜𝑙 1×60
𝑛= = 3.75 × 1020 𝑒𝑙𝑒𝑐𝑡𝑟𝑜𝑛𝑠
1.6×10−19
S22. Ans.(c)
S26. Ans.(c)
𝑍𝑛2+ + 2𝑒 − → 𝑍𝑛, 𝐸° = −0.760 𝑉
∆𝐺° = −𝑛𝐹𝐸° 𝑐𝑒𝑙𝑙 𝐸° 𝑐𝑒𝑙𝑙 = (−𝑣𝑒)
𝐴𝑔2 𝑂 + 𝐻2 𝑂
So, ∆𝐺° = (+𝑣𝑒) ∆𝐺 > 0
𝐸° = 0.344 𝑉
Also, ∆𝐺° = −2.303 𝑅𝑇 log 𝐾𝑒𝑞
Both are reducing potential :
∴ 𝐾𝑒𝑞 < 1
° °
As, 𝐸𝐴𝑔2 𝑂/2𝐴𝑔
> 𝐸𝑍𝑛 2+ /𝑍𝑛
S27. Ans.(d)
∴ Cell reaction will be
Concentration = 0.5 mol dm–3,
At anode; 𝑍𝑛 → 𝑍𝑛 2+
+ 2𝑒 −
E° = +0.760 V K = 5.76 × 10–3 S cm–1
At cathode; T = 298 K
− −
𝐴𝑔2 𝑂 + 𝐻2 𝑂 + 2𝑒 → 2𝐴𝑔 + 2𝑂𝐻 𝐾×1000
𝜆𝑚 =
𝐸° = +0.344 𝑉 𝑀
5.76×10−3
Cell reaction: = = 11.52 𝑆 𝑐𝑚2 𝑚𝑜𝑙 −1
0.5
𝑍𝑛 + 𝐴𝑔2 𝑂 + 𝐻2 𝑂 → 𝑍𝑛2+ + 2𝐴𝑔 + 2𝑂𝐻 − S28. Ans.(b)
°
𝑛=2 𝐸𝑐𝑒𝑙𝑙 = 1.104 𝑉 2𝐻 + + 2𝑒 − → 𝐻2
° 0.059 𝑃
∵ ∆𝐺° = −𝑛𝐹𝐸𝑐𝑒𝑙𝑙 𝐸 = 𝐸° − log (𝐻𝐻+2)2
2
∆𝐺° = −2 × 96500 × 1.10 0.059 𝑃
0=0− log (10𝐻−72)2
= −213072 𝐽 𝑚𝑜𝑙 −1 2

= −213.072 𝑘𝐽 𝑚𝑜𝑙 −1 log 1 = 0


|∆𝐺°| = 213.072 𝑘𝐽 𝑚𝑜𝑙 −1 𝑃𝐻 2 = (10−7 )2 = 10−14 𝑎𝑡𝑚 .
S23. Ans.(d)
According to Faraday’s first law: S29. Ans.(d)
w = z.i.t S30. Ans.(b)
𝐸
𝑧 = 96500 (𝑚𝑜𝑙𝑎𝑟 𝑚𝑎𝑠𝑠) Sn2+ cannot reduce Fe2+, so FeCl2 and
SnCl2 can exists together.

63
S31. Ans.(a)
∆𝐺° = −2.303 𝑅𝑇 log 𝐾𝑠𝑝
63300 = −2.303 × 8.314 × 298 log 𝐾𝑠𝑝
𝐾𝑠𝑝 ~ 8 × 10−12
S32. Ans.(c)
+6
𝑀𝑛𝑂4−2 → 𝑀𝑛𝑂4− + 𝑒 −
0.1 mole 0.1 mole
Q = nF = 0.1 × 96500 = 9650 C
S33. Ans.(c)
5600
𝑊𝑂 2 = 22400 × 32 = 8𝑔 = 1 𝑒𝑞𝑢𝑖𝑣𝑎𝑙𝑒𝑛𝑡
= 1 𝑒𝑞𝑢𝑖𝑣𝑎𝑙𝑒𝑛𝑡 𝑜𝑓 𝐴𝑔
= 108 𝑔
S34. Ans.(c)
According to Kohlrausch’s law of limiting
molar conductivity:
Λ 9.54
𝛼 = Λ𝑚
° = 238
= 0.04008
𝑚

% Dissociation () = 4.008%


S35. Ans.(b)
According to Nernst equation
0.0592 [𝐶𝑎𝑡𝑖𝑜𝑛]
𝐸𝑜𝑥𝑖 = 𝐸𝑟𝑒𝑑 − 𝑛
log [𝐴𝑛𝑖𝑜𝑛]
0.592 [10−10 ]
=0− log
2 [1]

pH = 0.59 V
S36. Ans.(a)
𝐸°𝑐𝑒𝑙𝑙 = 𝐸°𝑐𝑎𝑡ℎ𝑜𝑑𝑒 − 𝐸°𝑎𝑛𝑜𝑑𝑒 = 0.76 − (−0.34)
= 1.1 𝑉

64
Chapter 10
Chemical Kinetics

1. For a reaction, 3A ⟶ 2B (d) Both A and R are true and R is the


The average rate of appearance of B is given correct explanation of A.
∆[B]
by . The correct relation between the 5. For a chemical reaction,
∆t
average rate of appearance of B with the 4A + 3B ⟶ 6C + 9D
average rate of disappearance of A is given Rate of formation of C is 6 × 10-2 mol L-1 s-1
in option: (2023) and rate of disappearance of A is 4 × 10-2
−∆[A] mol L-1 s-1. The rate of reaction and amount
(a) ∆t of B consumed in interval of 10 seconds,
−3∆[A]
(b) respectively will be: (2022)
2∆t
−2∆[A] (a) 10 × 10 mol L s & 30 × 10 mol L-1
-2 -1 -1 -2
(c) 3∆t (b) 1 × 10-2 mol L-1 s-1 & 30 × 10-2 mol L-1
∆[A]
(d) (c) 10 × 10-2 mol L-1 s-1 & 10 × 10-2 mol L-1
∆t
(d) 1 × 10-2 mol L-1 s-1 & 10 × 10-2 mol L_1
2. The correct options for the rate law that
corresponds to overall first order reaction is 6. The given graph is a representation of
(2023) kinetic of a reaction (2022)
(a) Rate = k[A] [B]
0 2

(b) Rate = k[A][B]


(c) Rate = k[A]1/2[B]2
(d) Rate = k[A]-1/2[B]3/2
3. For a certain reaction, the rate = k[A]2[B],
when the initial concentration of A is tripled
keeping concentration of B constant, the
initial rate would (2023) The 'y' and 'x' axes for zero and first order
(a) increase by a factor of six reactions, respectively are
(b) increase by a factor of nine (a) zero order (y = concentration & x = time),
(c) increase by a factor of three first order (y = t1/2 & x = concentration)
(d) decrease by a factor of nine (b) zero order (y = concentration & x = time),
4. Given below are two statements: One is first order (y = rate constant & x =
labelled as Assertion (A) and the other is concentration)
labelled as Reason (R). (c) zero order (y = rate & x = concentration),
Assertion: A reaction can have zero first order (y = t1/2 & x = concentration)
activation energy. (d) zero order (y = rate & x = concentration),
Reason: The minimum extra amount of first order (y = rate & x = t1/2)
energy absorbed by reactant molecules so 7. For a first order reaction A ⟶ Products,
that their energy becomes equal to initial concentration of A is 0.1 M, which
threshold value, is called activation energy. becomes 0.001 M after 5 minutes. Rate
In the light of the above statements, choose constant for the reaction in min-1 is (2022)
the correct answer from the options given (a) 1.3818
below: (2023) (b) 0.9212
(a) Both A and R are true but R is NOT the (c) 0.4606
correct explanation of A. (d) 0.2303
(b) A is true but R is false.
(c) A is false but R is true.

65
8. For a reaction 𝐴 → 𝐵, enthalpy of reaction is (d) 100 s
–4.2 kJ mol–1 and enthalpy of activation is 12. In collision theory of chemical reaction, ZAB
9.6 kJ mol–1. The correct potential energy represents (2020 Covid Re-NEET)
profile for the reaction is shown in option. (a) The collision frequency of reactants, A
(2021) and B
(b) Steric factor
(c) The fraction of molecules with energies
equal to Ea
(a) (d) The fraction of molecules with energies
greater than Ea
13. The half-life for a zero order reaction having
0.02 M initial concentration of reactant is
100 s. The rate constant (in mol L–1 S–1) for
the reaction is (2020 Covid Re-NEET)
(b)
(a) 2.0 × 10–4
(b) 2.0 × 10–3
(c) 1.0 × 10–2
(d) 1.0 × 10–4
14. If the rate constant for a first order reaction
(c) is k, the time (t) required for the completion
of 99% of the reaction is given by: (2019)
(a) t = 0.693/k

(d) t = 2.303/k
(d) 15. For the chemical reaction
𝑁2 (𝑔) + 3𝐻2 (𝑔) ⇌ 2𝑁𝐻3 (𝑔)
The correct option is: (2019)
1 𝑑[𝐻2 ] 1 𝑑[𝑁𝐻3 ]
(a) − =−
9. The slope of Arrhenius Plot (ln K v/s 1/T) of 3 𝑑𝑡 2 𝑑𝑡
𝑑[𝑁2 ] 𝑑[𝑁𝐻3 ]
first order reaction is –5 × 103 K. The value (b) − 𝑑𝑡 = 2 𝑑𝑡
of Ea of the reaction is. Choose the correct (c)
𝑑[𝑁 ]
− 2 =
1 𝑑[𝑁𝐻3 ]
option for your answer. (2021) 𝑑𝑡
𝑑[𝐻2 ]
2 𝑑𝑡
𝑑[𝑁𝐻3 ]
[Given R = 8.314 JK mol ]
–1 –1 (d) 3 =2
𝑑𝑡 𝑑𝑡
(a) 83.0 kJ mol–1
16. When initial concentration of the reactant is
(b) 166 kJ mol–1
doubled, the half-life period of a zero order
(c) –83 kJ mol–1
reaction (2018)
(d) 41.5 kJ mol–1
(a) Is halved
10. An increase in the concentration of the (b) Is doubled
reactants of a reaction leads to change in: (c) Is tripled
(2020) (d) Remains unchanged
(a) Heat of reaction
17. The correct difference between first and
(b) Threshold energy
second order reactions is that: (2018)
(c) Collision frequency
(a) The rate of a first-order reaction does
(d) Activation energy
not depend on reactant concentrations;
11. The rate contant for a first order reaction is the rate of a second-order reaction does
4.606 × 10–3 s–1. The time required to reduce depend on reactant concentrations
2.0 g of the reactant to 0.2 g is: (2020) (b) The half-life of a first-order reaction
(a) 200 s does not depend on [A]0; the half-life of
(b) 500 s a second-order reaction does depend on
(c) 1000 s [A]0

66
(c) The rate of a first-order reaction does (a) Rate is proportional to the surface
depend on reactant concentrations; the coverage
rate of a second-order reaction does not (b) Rate is inversely proportional to the
depend on reactant concentrations surface coverage
(d) A first-order reaction can catalysed; a (c) Rate is independent of the surface
second-order reaction cannot be coverage
catalyzed (d) Rate of decomposition is very low
18. Mechanism of a hypothetical reaction 23. The rate of a first-order reaction is 0.04 mol
𝑋2 + 𝑌2 → 2𝑋𝑌 is given below: (2017-Delhi) L–1 s–1 at 10 seconds and 0.03 mol L–1 s–1 at
(i) 𝑋2 → 𝑋 + 𝑋 (fast) 20 seconds after initiation of the reaction.
(ii) 𝑋 + 𝑌2 ⇌ 𝑋𝑌 + 𝑌 (slow) The half-life period of the reaction is:
(2016-I)
(iii) 𝑋 + 𝑌 → 𝑋𝑌 (fast)
(a) 54.1 s
The overall order of the reaction will be
(b) 24.1 s
(a) 1.5
(c) 34.1 s
(b) 1
(d) 44.1 s
(c) 2
(d) 0 24. The addition of a catalyst during a chemical
reaction alters which of the following
19. A first order reaction has a specific reaction
quantities? (2016-I)
rate of 10–2 s–1. How much time will it take
(a) Activation energy
for 20 g of the reactant to reduce to 5 g?
(b) Entropy
(2017-Delhi)
(c) Internal energy
(a) 693.0 second
(d) Enthalpy
(b) 238.6 second
(c) 138.6 second The rate constant of the reaction 𝐴 → 𝐵 is
(d) 346.5 second 0.6 × 10 mole/second. If the concentration
20. Consider the reaction between chlorine and of A is 5 M, then concentration of B after 20
minutes is: (2015 Re)
nitric oxide
(a) 0.72 M
𝐶𝑙2 (𝑔) + 2𝑁𝑂(𝑔) → 2𝑁𝑂𝐶𝑙(𝑔)
(b) 1.08 M
On doubling the concentration of both
reactants the rate of the reaction increases (c) 3.60 M
by a factor of 8. However, if only the (d) 0.36 M
concentration of Cl2 id doubled, the rate is 26. When initial concentration of a reactant is
increased by a factor of 2. The order of this doubled in a reaction, its half-life period is
reaction with respect to NO is: not affected. The order of the reaction is:
(2017-Gujarat) (2015)
(a) 3 (a) First
(b) 0 (b) Second
(c) 1 (c) More than zero but less than first
(d) 2 (d) Zero
21. For the reaction, 𝑋𝐴 + 𝑌𝐵 → 𝑍𝐶, 27. The activation energy of a reaction can be
if
−𝑑[𝐴]
=
−𝑑[𝐵]
=
1.5𝑑[𝐶]
, then the correct determined from the slope of which of the
𝑑𝑡 𝑑𝑡 𝑑𝑡
following graphs? (2015)
statement among the following is: ln 𝑘
(2017-Gujarat) (a) 𝑇
𝑣𝑠. 𝑇
(a) The value of Y = 2 (b) ln 𝑘 𝑣𝑠. 𝑇
1

(b) The value of X = Y = Z = 3 𝑇 1


(c) 𝑣𝑠. 𝑇
(c) The value of X = Y = 3 ln 𝑘
(d) The value of X = 2 (d) ln 𝑘 𝑣𝑠. 𝑇
22. The decomposition of phosphine (PH3) on 28. For a given exothermic reaction, 𝑘𝑝 and 𝑘𝑝′
tungsten at low pressure is a first-order are the equilibrium constants at
reaction. It is because the: (2016-II) temperatures T1 and T2, respectively.

67
Assuming that heat of reaction is constant
in temperature range between T1 and T2, it
is readily observed that: (2014)
(a) 𝑘𝑝 < 𝑘𝑝′
(b) 𝑘𝑝 = 𝑘𝑝′
1
(c) 𝑘𝑝 < 𝑘 ′
𝑝
(d) 𝑘𝑝 > 𝑘𝑝′
29. A reaction having equal energies of
activation for forward and reverse reactions
has: (2013)
(a) S = 0
(b) G = 0
(c) H = 0
(d) H = G = S = 0
30. What is the activation energy for a reaction
if its rate doubles when the temperature is
raised from 20°C to 35°C?
(R = 8.314 J mol–1 K–1) (2013)
(a) 342 kJ mol–1
(b) 269 kJ mol–1
(c) 34.7 kJ mol–1
(d) 15.1 kJ mol–1

68
Answer Key
S1. Ans. (c)

S2. Ans. (d)

S3. Ans. (b)

S4. Ans. (c)

S5. Ans. (b)

S6. Ans. (c)

S7. Ans. (b)

S8. Ans. (a)

S9. Ans. (d)

S10. Ans. (c)

S11. Ans. (b)

S12. Ans. (a)

S13. Ans. (d)

S14. Ans. (c)

S15. Ans. (c)

S16. Ans. (b)

S17. Ans. (b)

S18. Ans. (a)

S19. Ans. (c)

S20. Ans. (d)

S21. Ans. (c)

S22. Ans. (a)

S23. Ans. (b)

S24. Ans. (a)

S25. Ans. (a)

S26. Ans. (a)

S27. Ans. (b)

S28. Ans. (d)

S29. Ans. (c)

S30. Ans. (c)

69
Solutions
S1. Ans.(c) r = k(constant)
3A ⟶ 2B Hence, 'y' as 'rate' & 'x' as concentration
1 ∆[A] 1 ∆[B] will give desired graph.
r= =+
3 ∆t 2 ∆t
For first order reaction:
∆[B] 2 ∆[A]
+ ∆t = − 3 ∆t 0.693
t1/2 = k
(constant)
S2. Ans.(d)
Hence, 'y' as 't1/2' and 'x' as concentration
r = k[A]-1/2[B]3/2 will give desired graph.
1 3
order = − 2 + 2 S7. Ans.(b)
= 2/2 For first order reaction,
2.303 [A0 ]
=1 K = log ; where A0 is the initial
t [A]
S3. Ans.(b) concentration of reactant A.
Rate = k[A]2[B] A0 = 0.1 M
If [A] is tripled and [B] is kept constant. A = 0.001 M
r1 = k[3A]2[B] t = 5 minutes
r1 = 9k[A]2[B] K=
2.303
log
0.1
=
2.303
log 102 =
2.303
× 2
5 0.001 5 5
r1 = 9r
K = 0.9212 min
Increased by a factor of nine
Ans.(a)
S4. Ans.(c)
For a given H is negative. Hence
A reaction cannot have zero activation potential energy profile is an exothermic
energy. reaction.
Ea is minimum extra amount of energy S9. Ans.(d)
absorbed by reactant molecules so that 𝐸𝑎 1
their energy becomes equal to threshold 𝑙𝑛𝑘 = 𝑙𝑛𝐴 − ( )
𝑅 𝑇
value. In lnk v/s 1/T graph
S5. Ans.(b) Slope = –Ea/R
4A + 3B ⟶ 6C + 9D –5 × 103 = –Ea/8.313
−d[A] 1 −d[B] 1
Rate of reaction = ×4 = ×3 = Ea = 5 × 103 × 8.314
dt dt
+d[C] 1
×6 =
+d[D] 1
×9 = 41500 J mol–1
dt dt
+d[C] 1 6×10−2
= 41.5 KJ/mol
Rate of reaction = ×6 = = 10-2
dt 6 S10. Ans.(c)
mol L-1 s-1
When the concentrations of the reactants
−1 d[B]
Rate of reaction = are raised, the reaction proceeds more
3 dt
−d[B]
quickly. This is due to an increase in the
dt
= 3 × rate of reaction = 3 × 10-2 mol number of molecules that have the
L-1 s-1 minimum required energy.
After interval of 10 sec. = 3 × 10-2 × 10 Collision frequency ∝ no. of reacting
= 30 × 10-2 mol L-1 molecules or atoms.

S6. Ans.(c) Higher the concentration of reactant


molecules higher is the probability of
For zero order reaction:
collision and so the collision frequency.
r = k[A]0

70
S11. Ans.(b) S16. Ans.(b)
For first order reaction: (𝑡1/2 )𝑧𝑒𝑟𝑜 =
[𝐴]0
2𝐾
2.303 [𝑅 ]
𝑘= 𝑡
𝑙𝑜𝑔. [𝑅0] ∴ If [A]0 = doubled, t1/2 = doubled
𝑡

4.606 × 10−3 =
2.303 [2]
log [0.2] S17. Ans.(b)
𝑡
0.693
2.303 – For first order reaction, 𝑡1/2 =
𝑘
𝑡= 𝑙𝑜𝑔10
4.606×10−3 which is independent of initial
103 concentration of reactant.
𝑡= 2
= 500 𝑠
1
– For second order reaction, 𝑡1/2 = ,
S12. Ans.(a) 𝑘[𝐴0 ]
which depends on initial
Rate of reaction according to collision
concentration of reactant.
theory can be expressed as
S18. Ans.(a)
Rate = 𝑍𝐴𝐵 𝑒 −𝐸𝑎/𝑅𝑇
In the slowest step.
Where, ZAB represents the collision
frequency of the reactants, A & B and 𝑋 + 𝑌2 ⇌ 𝑋𝑌 + 𝑌
𝑒 −𝐸𝑎/𝑅𝑇 represents the fraction of Half molecule of Y2 reacts with the one
molecules with energies equal to or molecule of X
greater than Ea ∴ 1 + 0.5 = 1.5
The number of collisions per second per S19. Ans.(c)
unit volume of the reaction mixture (A
K = 10 s , t = ? [R ] = 20 g
and B) is known as collision frequency
ZAB.
S13. Ans.(d) 𝑘 [𝑅]
For zero order reaction 2.303 20
𝑡= 10−2
log 5
𝑎
𝑡1/2 =
2𝑘 𝑡 = 138.6 𝑠𝑒𝑐
𝑎
𝑘= 2𝑡1/2 S20. Ans.(d)
0.02 𝑅 = 𝐾[𝐶𝑙2 ][𝑁𝑂]2
𝑘 = 2×100
Hence, order reaction = 1 + 2 = 3
= 1 × 10−4 𝑚𝑜𝑙 𝐿−1 𝑠 −1
∴ Order with respect to NO = 2
S14. Ans.(c)
S21. Ans.(c)
First order rate constant is given as,
−𝑑[𝐴] −𝑑[𝐵] 1.5𝑑[𝐶]
2.303 [𝐴 ] 𝑑𝑡
= 𝑑𝑡
= 𝑑𝑡
…(i)
𝑘= 𝑡
log [𝐴]0
𝑡
For the reaction
99% completion of reaction,
𝑥𝐴 + 𝑦𝐵 ⇌ 𝑍𝑐
2.303 100
𝑘= log 1 −𝑑[𝐴] −𝑑[𝐵] 𝑑[𝐶]
𝑡 = = …(ii)
2.303 𝑋𝑑𝑡 𝑌𝑑𝑡 𝑍𝑑𝑡
𝑘= × 2𝑙𝑜𝑔10
𝑡 Comparing equation (i) and (ii)
2.303 1
𝑡= 𝑘
×2 𝑋: 𝑌: 𝑍 = 1: 1: 1.5
4.606 2
𝑡= = 1: 1: 3 = 3: 3: 2
𝑘

S15. Ans.(c) 𝑋 = 𝑌 = 3, 𝑍 = 2
𝑁2 + 3𝐻2 ⇌ 2𝑁𝐻3 S22. Ans.(a)
Rate of reaction is given as At Low P, rate is proportional to the
𝑑[𝑁 ]
− 𝑑𝑡2 =
1 𝑑[𝐻 ]
− 3 𝑑𝑡2 =
1 𝑑[𝑁𝐻 ]
+ 2 𝑑𝑡 3 surface coverage and is of Ist order while

71
at high P, it follows zero order due to
complete coverage of surface area. S29. Ans.(c)
S23. Ans.b() Enthalpy (H) = Ef activation energy of
For a first order reaction, A → products forward – Eb activation energy of
and for concentration of the reactant at backward reaction
two different times. When 𝐸𝑓 = 𝐸𝑏
2.303 [𝐴]
𝑘= 𝑡2 −𝑡1
log [𝐴]1 ∆𝐻 = 0
2

2.303 (𝑟𝑎𝑡𝑒) S30. Ans.(c)


𝑘=𝑡 log (𝑟𝑎𝑡𝑒)1 (∵ rate ∝ [𝐴])
2 −𝑡1 2 𝑘 𝐸 1 1
2.303 0.04
log 𝑘2 = 2.303𝑅
𝑎
(𝑇 − 𝑇 )
−1 1 1 2
𝑘= (20−10)
log (0.03) = 0.0287 sec
𝑘2 = 2𝑘1 , 𝑇1 = 20 + 273 = 293 𝐾
0.693 0.693
𝑡1/2 = 𝑘
= 0.0287 sec−1
= 24.14 𝑠𝑒𝑐 or 𝑇2 = 35 + 273 = 308 𝐾
S24. Ans.(a) 𝑅 = 8.314 𝐽 𝑚𝑜𝑙 −1 𝐾 −1
When we add catalysts to any chemical
reaction. It provides an alternative
pathway for the reaction and lowers the
activation energy. So the reaction
becomes fast. As Ef and Eb (activation
energy for forward and backward
reaction) is reduced to same extent, so
there is no change in enthalpy of
reaction.
S25. Ans.(a)
Reaction is of zero order as the unit of
rate constant is 𝑚𝑜𝑙 𝐿−1 𝑠 −1 .
∴ Concentration of B = k × t = 0.6 × 10–3
× 20 × 60 = 0.72 M
S26. Ans.(a)
0.693
𝑡1/2 = 𝑘
𝑠𝑒𝑐

Ist order reaction is independent of


concentration of reactant.
S27. Ans.(b)
According to Arrhenius equation:
−𝐸𝑎 /𝑅𝑇
𝑘 = 𝐴𝑒
𝐸
ln 𝑘 = ln 𝐴 − 𝑅𝑇𝑎
Comparing with y = c + mx
Slope will be ln k
S28. Ans.(d)
𝑘𝑝 > 𝑘𝑝′ that is the previous pressure
constant was higher as the reaction
being exothermic T2 > T1.

72
Chapter 11
The p-Block Elements (Part – A)

1. Match the List I with List II. A. Peroxodisulphuric I. Two S-OH,


List I List II acid Four S=O,
(Hydride) (Type of Hydride) One S-O-S

A. NaH I. Electron precise B. Sulphuric acid II. Two S-OH,


PH3 II. Saline One S=O
B.
GeH4 III. Metallic C. Pyrosulphuric III. Two S-OH,
C. acid Four S=O,
D. LaH2.87 IV. Electron rich One S-O-O-
Choose the correct answer from option S
given below: (2023) D. Sulphurous acid IV. Two S-OH,
(a) A-III, B-IV, C-II, D-I Two S=O
(b) A-II, B-III, C-IV, D-I
(c) A-I, B-III, C-II, D-IV Choose the correct answer from the option
(d) A-II, B-IV, C-I, D-III given below: (2023)
(a) A-III, B-IV, C-I, D-II
2. Match List I with List II: (b) A-I, B-III, C-IV, D-II
List I

A. Coke I. Carbon atoms are (d) A-I, B-III, C-II< D-IV

B. Diamond II. Used as a dry List I List II


lubricant (Compounds) (Molecular
Fullerene III. Used as a reducing formula)
C.
agent A. Borax I. NaBO2
D. Graphite IV. Cage like molecules B. Kernite II. Na2B4O7.4H2O
Choose the correct answer from the options C. Orthoboric III. H3BO3
given below: (2023) acid
(a) A-IV, B-I, C-II, D-III D. Borax bead IV. Na2B4O7.10H2O
(b) A-III, B-I, C-IV, D-II Choose the correct answer from the options
(c) A-III, B-IV, C-I, D-II given below: (2022)
(d) A-II, B-IV, C-I, D-III (a) A-I, B-III, C-IV, D-II
3. The element expected to form largest ion to (b) A-IV, B-II, C-III, D-I
achieve the nearest noble gas configuration (c) A-II, B-IV, C-III, D-I
is (2023) (d) A-III, B-I, C-IV, D-II
(a) F 6. Which of the following reactions is a part of
(b) N the large-scale industrial preparation of
(c) Na nitric acid? (2022)
(d) O Pt
(a) Cu(NO3)2 + 2NO2 + 2H2O → 4HNO3
4. Match List I with List II: 500K,9bar

List I List II + Cu
Pt
(Oxoacids or Sulphur) (Bonds) (b) NaNO3 + H2SO4 → NaHSO4 + HNO3
500K,9bar
Pt
(c) 4NH3 + 5O2 (from air) → 4NO +
500K,9bar
6H2O
73
(d) 4HPO3 + 2N2O5 →
Pt
4HNO3 + P4O10 11. Identify the correct statements from the
500K,9bar following: (2020)
heat (1) CO2(g) is used as refrigerant for ice-
7. Na2B4O7 → X + NaBO2
cream and frozen food.
In the above reaction the product "X" is:
(2) The structure of C60 contains twelve six
(2022)
carbon rings and twenty five carbon
(a) NaB3O5
rings.
(b) H3BO3
(3) ZSM-5, a type of zeolite, is used to
(c) B2O3
convert alcohols into gasoline.
(d) Na2B2O5
(4) CO is colorless and odourless gas.
8. Match List I with List II. (a) 1 and 3 only
List I List II (b) 2 and 3 only
(Hydrides) (Nature) (c) 3 and 4 only
MgH2 I. Electron precise (d) 1, 2 and 3 only
A.
GeH4 II. Electron deficient 12. Match the following : (2020)
B.
Oxide Nature
C. B2H6 III. Electron rich
A. CO (i) Basic
D. HF IV. Ionic
B. BaO (ii) Neutral
Choose the correct answer from the options Al2O3 (iii) Acidic
given below: (2022) C.
(a) A-IV, B-I, C-II, D-III D. Cl2O7 (iv) Amphoteric
(b) A-III, B-I, C-II, D-IV Which of the following is correct option?
(c) A-I, B-II, C-IV, D-III (A) (B) (C) (D)
(d) A-II, B-III, C-I, D-IV (iii)
9. Which of the following statement is not (b) (iii) (iv) (i) (ii)
correct about diborane? (2022) (c) (iv) (iii) (ii) (i)
(a) There are two 3-centre-2-electron bonds (d) (i) (ii) (iii) (iv)
(b) The four terminal B-H bonds are two 13. Which of the following is not correct about
centre two electron bonds carbon monoxide? (2020)
(c) The four terminal Hydrogen atoms and (a) It reduces oxygen carrying ability of
the two Boron atoms lie in one plane blood.
(d) Both the Boron atoms are sp2 (b) The carboxyhaemoglobin (haemoglobin
hybridised bound to CO) is less stable than
10. Given below are two statements. oxyhaemoglobin.
Statement I: The boiling points of the (c) It is produced due to incomplete
following hydrides of group 16 elements combustion.
increases in the order- (d) It forms carboxyhaemoglobin.
H2O < H2S < HeSe < H2Te
14. Which of the following oxide is amphoteric
Statement II: The boiling points of these
in nature? (2020 Covid Re-NEET)
hydrides increase with increase in molar
(a) SiO2
mass.
(b) GeO2
In the light of above statements, choose the
(c) CO2
most appropriate answer from options given
(d) SnO2
below: (2022)
(a) Both Statement I and Statement II are 15. Which of the following species is not stable?
correct. (2019)
(b) Both Statement I and Statement II are (a) [𝑆𝑖𝐹6 ]2−

incorrect. (b) [𝐺𝑒𝐶𝑙6 ]2−


(c) Statement I is correct but Statement II is (c) [𝑆𝑛(𝑂𝐻)6 ]2−
incorrect. (d) [𝑆𝑖𝐶𝑙6 ]2−
(d) Statement I is incorrect but Statement II
is correct.
74
16. Which of the following is incorrect 23. The stability of +1 oxidation state among Al,
statement? (2019) Ga, In and Tl increases in the sequence:
(a) PbF4 is covalent in nature (2015 Re)
(b) siCl4 is easily hydrolysed (a) In < Tl < Ga < Al
(c) GeX4 (X = F, Cl, Br, I) is more stable than (b) Ga < In < Al < Tl
GeX2 (c) Al < Ga < In < Tl
(d) SnF4 is ionic in nature (d) Tl < In < Ga < Al
17. The correct order of atomic radii in group 13 24. Which of these is not a monomer for a high
elements is (2018) molecular mass silicone polymer? (2013)
(a) B < Al < In < Ga < Tl (a) MeSiCl3
(b) B < Al < Ga < In < Tl (b) Me2SiCl2
(c) B < Ga < Al < In < Tl (c) Me3SiCl
(d) B < Ga < Al < Tl < In (d) PhSiCl3
18. Which one of the following elements is 25. The basic structural unit of silicates is:
unable to form 𝑀𝐹63− ion? (2018) (2013)
(a) Ga (a) 𝑆𝑖𝑂
(b) Al (b) 𝑆𝑖𝑂44−
(c) In (c) 𝑆𝑖𝑂32−
(d) B (d) 𝑆𝑖𝑂42−
19. It is because of inability of ns2 electrons of 26. Which of the following structure is similar
the valence shell to participate in bonding to graphite? (2013)
that : (2017-Delhi) (a) BN
(a) Sn is reducing while Pb is oxidising
4+ 4+
(b) B
(b) Sn2+ is reducing while Pb4+ is oxidising
(c) Sn2+ is oxidising while Pb4+ is reducing (d) B H
(d) Sn2+ and Pb are both oxidising and
reducing
20. The tendency to form monovalent
compounds among the Group 13 elements
is correctly exhibited in: (2017-Gujarat)
(a) 𝐵 ≈ 𝐴 ≈ 𝐺𝑎 ≈ 𝐼𝑛 ≈ 𝑇𝑙
(b) 𝐵 < 𝐴𝑙 < 𝐺𝑎 < 𝐼𝑛 < 𝑇𝑙
(c) 𝑇𝑙 < 𝐼𝑛 < 𝐺𝑎 < 𝐴𝑙 < 𝐵
(d) 𝑇𝑙 ≈ 𝐼𝑛 < 𝐺𝑎 < 𝐴𝑙 < 𝐵
21. Boric acid is an acid because its molecule:
(2016-II)
(a) Accepta OH– from water releasing
proton
(b) Combines with proton from water
molecule
(c) Contains replaceable H+ ion
(d) Gives up a proton
22. AlF3 is soluble in HF only in presence of KF.
It is due to the formation of: (2016-II)
(a) 𝐾3 [𝐴𝑙𝐹3 𝐻3 ]
(b) 𝐾3 [𝐴𝑙𝐹6 ]
(c) 𝐴𝑙𝐻3
(d) 𝐾[𝐴𝑙𝐹3 𝐻3 ]

75
Answer Key
S1. Ans. (d)

S2. Ans. (b)

S3. Ans. (b)

S4. Ans. (a)

S5. Ans. (b)

S6. Ans. (c)

S7. Ans. (c)

S8. Ans. (a)

S9. Ans. (d)

S10. Ans. (b)

S11. Ans. (c)

S12. Ans. (a)

S13. Ans. (b)

S14. Ans. (d)

S15. Ans. (d)

S16. Ans. (a)

S17. Ans. (c)

S18. Ans. (d)

S19. Ans. (b)

S20. Ans. (b)

S21. Ans. (a)

S22. Ans. (b)

S23. Ans. (c)

S24. Ans. (c)

S25. Ans. (b)

S26. Ans. (a)

76
Solutions
S1. Ans.(d)
LaH2.87 ⟶ Non-stoichiometric
⟶ Metallic/Interstitial hydride.
S2. Ans.(b) D ⟶ Sulphurous acid H2SO3
Coke: It is used as reducing agent in
carbon reduction methods. (in
metallurgical process)
Diamond: It is a allotrope of carbon in
which each carbon is sp3 hybridised. S5. Ans.(b)
Borax:
Na2B4O7.10H2O = Na2[B4O5(OH)4].8H2O
Kernite: Na2B4O7.4H2O
Orthoboric acid: H3BO3=B(OH)3
Borax bead: NaBO2 (Sodium metaborate)
S6. Ans.(c)
Fullerene: It contains pentagonal & Pt
hexagonal rings (cage like structure) 4NH + 5O (from air) → 4NO +
500K,9bar

Graphite: It is soft solid because graphite 6H O


layers are bonded with weak Vander Wall 2NO(g) + O (g) ⇌ 2NO (g)
attractions. 3NO2(g) + H2O(l) ⟶ 2HNO3(aq) + NO(g)
S3. Ans.(b) This is industrial method of preparation
F-1, N-3, Na+ and O-2 of nitric acid.
all ions are isoelectronic containing 10e− S7. Ans.(c)
Zeff ⟶ Na+ > F − > O−2 > N −3 Na2B4O7 →

B2O3 + 2NaBO2
order of radius ⟶ N −3 > O−2 > F − > Na+ Product X is B2O3
⟶ Nitrogen to achieve Noble gas S8. Ans.(a)
configuration is tan 3e− and form N −3
List I List II
S4. Ans.(a) (Nature)
(Hydrides)
A ⟶ Peroxodisulphuric acid
A. MgH2 ⟶ Ionic

B. GeH4 ⟶ Electron precise

C. B2H6 ⟶ Electron deficient

D. HF ⟶ Electron rich
B ⟶ Sulphuric acid A-IV, B-I, C-II, D-III
S9. Ans.(d)
Each boron atoms in diborane uses sp3
hybrid orbitals for bonding.

C ⟶ Pyrosulphuric acid H2S2O7

77
𝑆𝑖𝐶𝑙62− does not exist because six large
chloride ions cannot be accommodated
around Si4+ due to its small size.
S16. Ans.(a)
PbF4 is ionic in nature, because cation
is bigger and anion is smaller.
SiF4 is easily hydrolysed because it has
S10. Ans.(b)
unoccupied 3d orbital that are able to
Compound Boiling point (K) accept electron pairs from the oxygen
H 2O 373 atoms on water to form bond.
H 2S 213 𝑆𝑖𝐶𝑙4 + 3𝐻2 𝑂 → 𝐻2 𝑆𝑖𝑂3 + 4𝐻𝐶𝑙
H2Se 232 GeX4 is more stable than GeX2 as in
GeX4 all the orbitals are fully filled.
H2Te 269
SnF4 is ionic in nature as F atom is very
The boiling points of these hybrids not
small and Sn atom is very large. So, it is
exactly increases with increase in molar
ionic in nature according to Fajan’s rule.
mass.
S17. Ans.(c)
H2O has maximum boiling point due to
intermolecular hydrogen bonding. Group 13 order of atomic radius is not
S11. Ans.(c) regular due to transition contraction.

Dry ice, CO (s), is used as refrigerant So order is: B < Ga < Al < In < Tl

C60 contains 20 six membered rings, 12 S18. Ans.(d)


five membered rings statement 3 and 4 ∵ ‘B’ has no vacant d-orbitals in its
are correct. valence shell, so it can’t extend its
S12. Ans.(a) covalency beyond 4, i.e., ‘B’ cannot form
the ion like 𝑀𝐹63− i.e. 𝐵𝐹63−.
CO is Neutral oxide
Hence, the correct option is (d).
BaO is Basic oxide
S19. Ans.(b)
Al2O3 is Amphoteric oxide
Sn2+ is reducing while Pb4+ is oxidizing.
Cl2O7 is Acidic oxide
This is because of absence of f-orbital
S13. Ans.(b) (fully filled) in Sn2+ and presence in Pb4+
Carbon monoxide binds with Hb to form due to which Pb4+ show inert pair effect
300 times more stable compound but not Sn2+.
carboxyhaemoglobin than S20. Ans.(b)
oxyhaemoglobin complex.
+1 oxidation state increases down the
S14. Ans.(d) group as the fully filled d- and f- orbitals
CO2 : acidic will get added due to which inert pair
SnO2 : amphoteric effect will come into role.

SiO2 : acidic Tl > In > Ga > Al > B

GeO2 : acidic S21. Ans.(a)


Boric acid is a Lewis acid, it accepts a
S15. Ans.(d)
pair of electron in aqueous solution to
Due to presence of vacant d-orbital in si,
complete its octet because it is an
Ge and Sn they form species like 𝑆𝑖𝐹62− , electron deficient compound.
[𝐺𝑒𝐶𝑙6 ]2 , [𝑆𝑛(𝑂𝐻)6 ]2− .
𝐵(𝑂𝐻)3 + 𝐻2 𝑂 ⇌ [𝐵(𝑂𝐻)4 ]− 𝐻+

78
S22. Ans.(b)
𝐻𝐹
𝐴𝑙𝐹3 + 3𝐾𝐹 → 𝐾3 [𝐴𝑙𝐹6 ]
AlF3 is insoluble in Anhydrous HF
because F– ion are not available in
Hydrogen bonded HF.
S23. Ans.(c)
Order of oxidation state in group 13 is
Al < Ga < In < Tl
(+1) oxidation state stability increases
down the group because of inert pair
effect.
S24. Ans.(c)
Me3SiCl is not a monomer for high
molecular mass silicone polymer bcause
it generates Me3SiOH when subjected to
hydrolysis which contains only one
reacting site.
Hence, the polymerization stops just
after first step.

S25. Ans.(b)
Basic unit for silicates is 𝑆𝑖𝑂44−

S26. Ans.(a)
BN is also known as inorganic graphite
with sp2 hybridisation and hexagonal
structure.

79
Chapter 13
D and F Block Elements

1. Given below are two statements: One is B. The highest oxidation number
labelled as Assertion (A) and the other is corresponding to the group number in
labelled as Reason (R). (2023) transition metal oxides is attained in
Assertion (A): Ionization enthalpy Sc2O3 to Mn2O7.
increases along each series of the transition C. Basic character increases from V2O3 to
elements from left to right. However, small V2O4 to V2O5.
variations occur. D. V2O4 dissolves in acids to give VO3−
4 salts.
Reason (R): There is corresponding E. CrO is basic but Cr2O3 is amphoteric.
increase in nuclear charge which Choose the correct answer from the options
accompanies the filling of electrons in the given below: (2023)
inner d-orbitals. (a) B and D only
In the light of the above statements, choose (b) C and D only
the most appropriate answer from the (c) B and C only
options given below: (d) A and E only
(a) (A) is correct but (R) is not correct.
5. Given below are two statements:
(b) (A) is not correct but (R) is correct.
Statement I: Cr2+ is oxidising and Mn3+ is
(c) Both (A) and (R) are correct and (R) is
reducing in nature.
the correct explanation of (A).
Statement II: Sc compounds are repelled
(d) Both (A) and (R) are correct but (R) is not
by the applied magnetic field.
the correct explanation of (A). In the light of the above statements, choose
2. Given below are two statements: One is the most appropriate answer from the
labelled as Assertion (A) and the other is options given below: (2022)
labelled as Reason (R). (a) Statement I is incorrect but Statement II
Assertion (A): Helium is used to dilute is correct.
oxygen in diving apparatus. (b) Both Statement I and Statement II are
Reason (R): Helium has high solubility in correct.
O 2. (c) Both Statement I and Statement II are
In the light of the above statements, choose incorrect.
the most appropriate answer from the (d) Statement I is correct but Statement is
options given below: (2023) incorrect.
(a) Both (A) and (R) are true but (R) is NOT 6. Zr (Z = 40) and Hf (Z = 72) have similar
the correct explanation of (A). atomic and ionic radii because of:
(b) (A) is true but (R) is false. (2021)
(c) (A) is false but (R) is true. (a) Diagonal relationship
(d) Both (A) and (R) are true and (R) is the (b) Lanthanoid contraction
correct explanation of (A). (c) Having similar chemical properties
3. The solubility of Cu+2 is more than Cu+ salts (d) Belonging to same group
in aqueous solution due to- (2023)
7. The incorrect statement among the
(a) enthalpy of atomization.
following is: (2021)
(b) hydration energy.
(a) Most of the trivalent Lanthanoid ions
(c) second ionisation enthalpy.
are colorless in the solid state
(d) first ionisation enthalpy.
(b) Lanthanoids are good conductors of
4. Which of the following statements are heat and electricity
INCORRECT? (c) Actinoids are highly reactive metals,
A. All the transition metals except especially when finely divided
scandium form MO oxides which are
ionic.

80
(d) Actinoid contraction is greater for (d) A-(i) B-(iv) C-(ii) D-(iii)
element to element that Lanthanoid 11. Identify the incorrect statement from the
contraction following: (2020 Covid Re-NEET)
(a) Lanthanoids reveal only +3 oxidation
8. The calculated spin only magnetic moment state.
of 𝐶𝑟 2+ ion is (2020) (b) The lanthanoid ions other than 𝑓 0 type
(a) 4.90 BM and the 𝑓 14 type are all paramagnetic.
(b) 5.92 BM (c) The overall decreases in atomic and
(c) 2.84 BM ionic radii from lanthanum to luteium is
(d) 3.87 BM called lanthanoid contraction.
9. Identify the incorrect statement. (2020) (d) Zirconium and Hafnium have identical
(a) The transition metals and their
compounds are known for their catalytic radii of 160 pm and 159 pm,
activity due to their ability to adopt respectively as a consequence of
multiple oxidation states and to form
lanthanoid contraction.
complexes.
(b) Interstitial compounds are those that 12. Match the element in column I with that in
are formed when small atoms like H, C column II. (2020 Covid Re-NEET)
or N are trapped inside the crystal Column I Column II
lattices of metals. Copper (i) Non-metal
(A)
(c) The oxidation states of chromium in
(B) Fluorine (ii) Transition
𝐶𝑟𝑂42− and 𝐶𝑟2 𝑂72− are not the same.
metal
(d) 𝐶𝑟 2+ (𝑑4 ) is a stronger reducing agent
(C) Silicon (iii) Lanthanoid
than 𝐹𝑒 2+ (𝑑6 ) in water.
(D) (iv) Metalloid
10. Match the following aspects with the
respective metal. (2020 Covid Re-NEET) Select the correct option
Aspects Metal (a) A-(i) B-(ii) C-(iii) D-(iv)
(b) A-(ii) B-(iv) C-(i) D-(iii)
(A) The metal which (i) Scandium
(c) A-(ii) B-(i) C-(iv) D-(iii)
reveals a
(d) A-(iv) B-(iii) C-(i) D-(ii)
maximum number
of oxidation states 13. The manganate and permanganate ions are
The metal (ii) Copper tetrahedral, due to: (2019)
(B)
although placed in (a) The 𝜋-bonding involves overlap of p-
3d block is orbitals of oxygen with d-orbitals of
considered not as manganese
a transition (b) There is no 𝜋-bonding
element (c) The 𝜋-bonding involves overlap of p-
The metal which (iii) Manganese orbitals of oxygen with p-orbitals of
(C)
does not exhibit manganese
variable oxidation (d) The 𝜋-bonding involves overlap of d-
states orbitals of oxygen with d-orbitals of
(D) The metal which (iv) Zinc
in +1 oxidation manganese
state in aqueous 14. Which of the following ions exhibits d-d
solution transition and paramagnetism as well?
undergoes (2018)
disproportionation (a) 𝐶𝑟𝑂42−
Select the correct option: (b) 𝐶𝑟2 𝑂72−
(a) A-(iii) B-(iv) C-(i) D-(ii) (c) 𝑀𝑛𝑂42−
(b) A-(iii) B-(i) C-(iv) D-(ii) (d) 𝑀𝑛𝑂4−
(c) A-(ii) B-(iv) C-(i) D-(iii)

81
15. Match the metal ions given in Column I with (a) All the lanthanons are much more
the spin magnetic moments of the ions reactive than aluminium
given in Column II and assign the correct (b) Ce(+4) solutions are widely used as
code: (2018) oxidizing agent in volumetric analysis
Column I Column II (c) Europium shows +2 oxidation state.
𝐶𝑜 3+ (i) (d) The basicity decreases as the ionic
A. √8 BM
radius decreases from Pr to Lu.
B. 𝐶𝑟 3+ (ii) √35 BM
21. The electronic configurations of Eu (Atomic
C. 𝐹𝑒 3+ (iii) √3 BM
Number 63) Gd (Atomic Number 64) and Tb
D. 𝑁𝑖 2+ (iv) √24 BM (Atomic Number 65) are: (2016-I)
(v) (a) [𝑋𝑒]4𝑓 7 6𝑠 2 , [𝑋𝑒]4𝑓 7 5𝑑1 6𝑠 2 and [𝑋𝑒]4𝑓 9 6𝑠 2
√15 BM
(b) [𝑋𝑒]4𝑓 7 6𝑠 2 , [𝑋𝑒]4𝑓 8 6𝑠 2 and [𝑋𝑒]4𝑓 8 5𝑑1 6𝑠 2
(a) A-(iv) B-(v) C-(ii) D-(i) (c) [𝑋𝑒]4𝑓 6 5𝑑1 6𝑠 2 , [𝑋𝑒]4𝑓 7 5𝑑1 6𝑠 2 and
(b) A-(i) B-(ii) C-(iii) D-(iv) [𝑋𝑒]4𝑓 9 5𝑑1 6𝑠 2
(c) A-(iii) B-(v) C-(i) D-(ii) (d) [𝑋𝑒]4𝑓 6 5𝑑1 6𝑠 2 , [𝑋𝑒]4𝑓 7 5𝑑1 6𝑠 2 and
(d) A-(iv) B-(i) C-(ii) D-(iii)
[𝑋𝑒]4𝑓 8 5𝑑1 6𝑠 2
16. Name the gas that can readily decolourise
acidified 𝐾𝑀𝑛𝑂4 solution: (2017-Delhi) 22. Which one of the following statements is
(a) 𝑃2 𝑂5 correct when 𝑆𝑂2 is passed through
(b) 𝐶𝑂2 acidified 𝐾2 𝐶𝑟2 𝑂7 solution? (2016-I)
(c) 𝑆𝑂2 (a) 𝑆𝑂2 is reduced
(d) 𝑁𝑂2 (b) Green 𝐶𝑟2 (𝑆𝑂4 )3 is formed
(c) The solution turns blue
17. The reason for greater range of oxidation
The solution is decolourised
states in actinoids is attributed to:
(2017-Delhi) 23. Gadolinium belongs to 4f series. Its atomic
(a) 4𝑓 and 5𝑑 levels being close in energies number is 64. Which of the following is the
(b) The radioactive nature of actinoids correct electronic configuration of
(c) Actinoid contraction gadolinium? (2015 Re)
(d) 5𝑓, 6𝑑 and 7𝑠 levels having comparable (a) [𝑋𝑒]4𝑓 6 5𝑑2 6𝑠 2
(b) [𝑋𝑒]4𝑓 8 6𝑑2
energies
(c) [𝑋𝑒]4𝑓 9 5𝑠1
18. 𝐻𝑔𝐶𝑙2 and 𝐼2 both when dissolved in water (d) [𝑋𝑒]4𝑓 7 5𝑑1 6𝑠 2
containing 𝐼 − ions the pair of species formed
24. Because of lanthanoid contraction, which of
is: (2017-Delhi)
the following pairs of elements have nearly
(a) 𝐻𝑔2 𝐼2 , 𝐼 −
same atomic radii? (Numbers in the
(b) 𝐻𝑔𝐼2 , 𝐼3−
parenthesis are atomic numbers) (2015)
(c) 𝐻𝑔𝐼2 , 𝐼 −
(a) 𝑍𝑟(40) 𝑎𝑛𝑑 𝑁𝑏(41)
(d) 𝐻𝑔𝐼42− , 𝐼3−
(b) 𝑍𝑟(40) 𝑎𝑛𝑑 𝐻𝑓 (72)
19. Which of the following lanthanoids shows (c) 𝑍𝑟 (40) 𝑎𝑛𝑑 𝑇𝑎 (73)
+4 oxidation state to acquire noble gas (d) 𝑇𝑖 (22) 𝑎𝑛𝑑 𝑍𝑟 (40)
configuration? (Atomic noumber : La = 57,
25. Which of the following processes does not
Ce = 58, Eu = 63 and Yb = 70)
involve oxidation of iron? (2015)
(2017-Gujarat)
(a) Decolourisation of blue 𝐶𝑢𝑆𝑂4 solution
(a) Eu
by iron
(b) Ce
(b) Formation of 𝐹𝑒(𝐶𝑂)5 from Fe
(c) Yb
(c) Liberation of 𝐻2 from steam by iron at
(d) La
high temperature
20. Which one of the following statements (d) Rusting of iron sheets
related to lanthanons is incorrect?
26. Magnetic moment 2.84 B.M. is given by:
(2016-II)
(Atomic numbers, Ni = 28, Ti = 22, Cr = 24,
Co = 27) (2015)
82
(a) 𝑇𝑖 3+
(b) 𝐶𝑟 2+
(c) 𝐶𝑜 2+
(d) 𝑁𝑖 2+
27. Reason of lanthanoid contraction is:
(2014)
(a) Increasing nuclear charge
(b) Decreasing nuclear charge
(c) Decreasing screening effect
(d) Negligible screening effect of ‘𝑓’ orbitals
28. Magnetic moment 2.83 BM is given by
which of the following ions? (2014)
(Atomic Number Ti = 22, Cr = 24, Mn = 25,
Ni = 28)
(a) 𝑁𝑖 2+
(b) 𝐶𝑟 3+
(c) 𝑀𝑛2+
(d) 𝑇𝑖 3+
29. Which of the following statements about the
interstitial compounds is incorrect?
(2013)
(a) They retain metallic conductivity
(b) They are chemically reactive
(c) They are much harder than the pure
metal
(d) They have higher melting points than
the pure metal
30. Which of the following lanthanoid ions is
diamagnetic?
(Atomic Number Ce = 58, Sm = 62, Eu = 63,
Yb = 70) (2013)
(a) 𝐶𝑒 2+
(b) 𝑆𝑚2+
(c) 𝐸𝑢2+
(d) 𝑌𝑏 2+
31. Identify the correct order of solubility in
aqueous medium: (2013)
(a) 𝑁𝑎2 𝑆 > 𝐶𝑢𝑆 > 𝑍𝑛𝑆
(b) 𝑁𝑎2 𝑆 > 𝑍𝑛𝑆 > 𝐶𝑢𝑆
(c) 𝐶𝑢𝑆 > 𝑍𝑛𝑆 > 𝑁𝑎2 𝑆
(d) 𝑍𝑛𝑆 > 𝑁𝑎2 𝑆 > 𝐶𝑢𝑆

83
Answer Key
S1. Ans. (c)

S2. Ans. (b)

S3. Ans. (b)

S4. Ans. (b)

S5. Ans. (a)

S6. Ans. (b)

S7. Ans. (a)

S8. Ans. (a)

S9. Ans. (c)

S10. Ans. (a)

S11. Ans. (a)

S12. Ans. (c)

S13. Ans. (a)

S14. Ans. (c)

S15. Ans. (a)

S16. Ans. (c)

S17. Ans. (d)

S18. Ans. (d)

S19. Ans. (b)

S20. Ans. (a)

S21. Ans. (a)

S22. Ans. (b)

S23. Ans. (d)

S24. Ans. (b)

S25. Ans. (b)

S26. Ans. (d)

S27. Ans. (d)

S28. Ans. (a)

S29. Ans. (b)

S30. Ans. (d)

S31. Ans. (b)

84
Solutions
S1. Ans.(c) In the +6 oxidation state, the most
Reason is the correct explanation of important species formed by chromium
Assertion.
are the chromate, 𝐶𝑟𝑂4−2 , and
S2. Ans.(b)
dichromate, 𝐶𝑟2 𝑂72− , ion
Assertion is true because He has low
Thus correct option is c.
solubility in blood. (NCERT)
S10. Ans.(a)
S3. Ans.(b)
Cu(s) ⟶ Cu(g) ⟶ Cu+(g) ⟶ Cu+2(g) ⟶ Cu+2(aq)
(A) In 3d-series, Manganese shows
maximum number of oxidation
∆Hatomisation IE1 IE2 Hydration
states i.e., (+2, to +7)
energy
(B) Zinc atom has completely filled d-
Cu+2 is more stable than Cu+1 because
orbitals in its ground state as well as
released hydration energy is more in case
in its oxidized state, thus it is not
of Cu+2 than Cu+1.
regarded as a transition element.
S4. Ans.(b) (C) Scandium shows only one oxidation
+3 +4 +5
C ⟶ V2 O3 ⟶ V2 O4 ⟶ V2 O5 state i.e., +3.
(D) 𝐶𝑢+ undergoes disproportionation
Acidic Nature ↑
reaction in aqueous solution
D ⟶ V2O5 dissolve in acid to give VO−3
2𝐶𝑢 (𝑎𝑞) → 𝐶𝑢 (𝑎𝑞) + 𝐶𝑢(𝑠)
salts. This doesn't shown by V2O4
S11. Ans.(a)
S5. Ans.(a)
Lanthanoids can also show +2 or +4
Statement I: Cr2+ is reducing as its
oxidation states in solution or in solid
configuration changes from d4 to d3, the
latter having a half-filled t2g level. On the compounds.
other hand, the change from Mn3+ to
Most common oxidation state of
Mn2+ results in the half-filled (d5)
configuration which has extra stability. Lanthanoids is +3
Statement II: Sc3+ has zero unpaired S12. Ans.(c)
electron, so magnetic moment is also
(A) Copper (ii) Transition
zero. Hence, Sc3+ will repelled by the
applied magnetic field. metal
S6. Ans.(b) (B) Fluorine (i) Non-metal
S7. Ans.(a)
(C) Silicon (iv) Metalloid
S8. Ans.(a)
The electronic configuration of (D) Cerium (iii) Lanthanoid

𝐶𝑟 2+ [𝐴𝑟]3𝑑4

So, Number of unpaired electron is 4 S13. Ans.(a)


Spin only magnetic moment =

𝜇 = √𝑛(𝑛 + 2) = √4(4 + 2) = 4.9 B. M.


• Manganate (𝑀𝑛𝑂42− ):
S9. Ans.(c)
85
soluble
S19. Ans.(b)
• Permanganate (𝑀𝑛𝑂4− ): Ce a have +4 oxidation state.

25
𝑀𝑛 − 3𝑑5 4𝑠 2 S20. Ans.(a)
Lanthanon’s or lanthanoids are much
If bonding take place by overlap of p-
orbital of oxygen & d-orbital of Mn, then less reactive as compared to aluminium
manganate and permaganate ions are because of high ionization potential
tetrahedral.
because of lanthanoid contraction
S14. Ans.(c)
extremely over dominates on inert pair
S15. Ans.(a)
effect.
𝐶𝑜 3+ = [Ar]3𝑑6 , Unpaired electron (n) = 4
𝐶𝑒 4+ is a good oxidizing agent +4 state
Spin magnetic moment = 4√4(4 + 2)
and readily converts to +3 state. As one
= √24 B.M
moves from Ce to Lu, ionic radius
𝐶𝑟 3+ = [𝐴𝑟]3𝑑3 , Unpaired electron (n) = 3
regularly decreases or covalent
Spin magnetic moment = √3(3 + 2)
character increases because of which
= √15 B.M
basic character decreases.
𝐹𝑒 3+ = [𝐴𝑟]3𝑑5 , Unpaired electron (n) = 5
S21. Ans.(a)
Spin magnetic moment = √5(5 + 2)
Electronic configuration of 𝐸𝑢, 𝐺𝑑 and
= √35 B.M
𝑇𝑏:
𝑁𝑖 2+ = [𝐴𝑟]3𝑑 , Unpaired electron (n) = 2
Unpaired electron (n) = 0; Diamagnetic 𝐸𝑢 = [𝑋𝑒]4𝑓 7 6𝑠 2 𝐺𝑑 = [𝑋𝑒]4𝑓 7 5𝑑1 6𝑠 2
S16. Ans.(c) 𝑇𝑏 = [𝑋𝑒]4𝑓 9 6𝑠 2
𝑆𝑂2 is a gas that can readily decolourise S22. Ans.(b)
acidified 𝐾𝑀𝑛𝑂4 solution. 𝐾2 𝐶𝑟2 𝑂7 + 𝐻2 𝑆𝑂4 + 3𝑆𝑂2 → 𝐾2 𝑆𝑂4

S17. Ans.(d) +𝐶𝑟2 (𝑆𝑂4 )3 + 𝐻2 𝑂


5𝑓, 6𝑑, 7𝑠 level having comparable S23. Ans.(d)
energies. There is very less energy gap Gadolinium belongs to 4𝑓 series.
between them.
Atomic number = 64
S18. Ans.(d)
Electronic configuration for ‘𝑓’ series is
In a solution containing 𝐻𝑔𝐶𝑙2 , 𝐼2 and 𝐼 − ,
(𝑛 − 2)𝑓 1−14 (𝑛 − 1)𝑑1−10 𝑛𝑠 2
both 𝐻𝑔𝐶𝑙2 and 𝐼2 compete for 𝐼 − .
So, electronic configuration is
Since formation constant of [𝐻𝑔𝐼4 ]2− is
[𝑋𝑒]4𝑓 7 5𝑑1 6𝑠 2
1.9 × 1030 which is very large as
4𝑓 7 provides extra stability.
compared with 𝐼3− (𝐾𝑓 = 700) S24. Ans.(b)
 𝐼 − will preferentially combine with Zr (40) and Hf (72) will have same

𝐻𝐺𝐶𝑙2 + 2𝐼 − → 𝐻𝑔𝐼2 ↓ +2𝐶𝑙 − atomic size because of lanthanoid


contraction and poor shielding effect.
Red ppt
S25. Ans.(b)
𝐻𝑔𝐼2 + 2𝐼 − → [𝐻𝑔𝐼4 ]2−
86
Formation of 𝐹𝑒(𝐶𝑂)5 does not involve S30. Ans.(d)
oxidation of Fe because here oxidation Lanthanoid contraction is a significant
state of Fe is 0. decrement in atomic size of atoms in ‘𝑓’

S26. Ans.(d) block because of poor shielding effect of

Magnetic moment (𝜇) for 𝑁𝑖 2+ = ‘𝑓’ orbital electrons as nuclear charge


effectively increases & atom shrinks.
√𝑛(𝑛 + 2)
S31. Ans.(a)
𝜇 = √𝑛(𝑛 + 2)

n=2 n = unpaired electron

𝜇 = √2(2 + 2) = 2√2 𝑁𝑖 2+ = 3𝑑8 2 unpaired electron

~ 2.84 B.M = 2√2 = 2.83 BM

S27. Ans.(b)
Interstitial compounds are chemically
inert. They do not show reactivity & lie
at the end in reactivity series
S28. Ans.(d)
𝑌𝑏(70) = [𝑋𝑒]4𝑓 5𝑑 6𝑠
𝑌𝑏 2+ = [𝑋𝑒]4𝑓 5𝑑 6𝑠
𝑌𝑏 2+ has no unpaired electron so, it is a
diamagnetic species.
S29. Ans.(b)
Sodium sulphide is soluble in water.
The solubility product (and hence
solubility) of ZnS is larger than that of
CuS.

87
Chapter 14
Coordination Compounds

1. Select the element (M) whose trihalides B. [Cr(NH3)6][Co(CN)6] 2. Coordination


cannot be hydrolysed to produce an ion of and isomerism
the form [M(H2O6]3+ (2023) [Cr(CN)6][Co(NH3)6
(a) Ga C. [Co(NH3)5(SO4)]Br 3. Linkage
(b) In and [Co(NH3)5Br]SO4 isomerism
(c) Al D. [Cr(H2O)6]Cl3 and 4. Solvate
(d) B [Cr(H2O)5Cl]Cl2∙H2O isomerism

2. Which of the following forms a set of Choose the correct answer from the options
complex and a double salt, respectively? given below: (2022)
(2023) (a) A-4, B-3, C-2, D-1
(a) CuSO4∙5H2O and CuCl2∙4NH3 (b) A-3, B-1, C-2, D-4
(b) PtCl2∙2NH3 and PtCl4∙2HCl (c) A-2, B-3, C-4, D-1
(c) K2PtCl2∙2NH3 and KAl(SO4)2∙12H2O (d) A-3, B-2, C-1, D-4
(d) NiCl2∙6H2O and NiCl2(H2O)4 7. Given below are two statements: One is
3. Type of isomerism exhibited by compounds labelled as Assertion (A) and the other is
[Cr(H2O)6Cl3, [Cr(H2O)5Cl2Cl2∙H2O, labelled as Reason (R).
[Cr(H2O)4Cl2]Cl∙2H O and the value of Assertion (A): The metal carbon bond in
coordination number (CN) of central metal metal carbonyls possesses both 𝜎 and 𝜋
ion in all these compounds, respectively is: character.
(2023) Reason (R): The ligand to metal bond is a 𝜋
(a) Geometrical isomerism, CN = 2 bond and metal to ligand bond is a 𝜎 bond
(b) Optical isomerism, CN = 4 In the light of the above statements, choose
(c) Ionisation isomerism, CN = 4 the most appropriate answer from the
(d) Solvate isomerism, CN = 6 options given below: (2022)
(a) (A) is not correct but (R) is correct.
4. Homoleptic complex from the following
(b) Both (A) and (R) are correct and (R) is
complexes is: (2023)
the correct explanation of (A).
(a) Diamminechloridonitrito-N-platinum(II)
(c) Both (A) and (R) are correct but (R) is the
(b) Pentaamminecarbonatocobalt(III)
not the correct explanation of (A).
chloride
(d) (A) is correct but (R) is not correct.
(c) Triamminetriaquachromium(III)
chloride 8. The IUPAC name of the complex-
(d) Potassium trioxalatoaluminate(III) [Ag(H2O)2][Ag(CN)2] is: (2022)
(a) dicyanidosilver(II) diaquaargentate(II)
5. Which complex compound is most stable?
(b) diaquasilver(II) dicyanidoargentate(II)
(2023)
(c) dicyanidosilver(I) diaquaargentate(I)
(a) [Co(NH3)3(NO3)3]
(d) diaquasilver(I) dicyanidoargentate(I)
(b) [CoCl2(en)2]NO3
(c) [Co(NH3)6]2(SO4)3 9. The order of energy absorbed which is
(d) [Co(NH3)4(H2O)Br](NO3)2 responsible for the color of complexes
(A) [Ni(H2O)2(en)2]2+
6. Match List I with List II:
(B) [Ni(H2O)4(en)]2+ and
List I List II
(C) [Ni(en)3]2+
(Complexes) (Types)
is: (2022)
A. [Co(NH3)5NO2]Cl2 1. Ionisation (a) A > B > C
and isomerism
(b) C > B > A
[Co(NH3)5ONO]Cl2
(c) C > A > B
(d) B > A > C

88
10. Ethylene diaminetetraacetate (EDTA) ion is: 14. What is the correct electronic configuration
(2021) of the central atom in 𝐾4 [𝐹𝑒(𝐶𝑁)6 ] based on
(a) Unidentate ligand crystal field theory? (2019)
(b) Bidentate ligand with two “N” donor 4 2
(a) 𝑡2𝑔 𝑒𝑔
atoms 6 0
(b) 𝑡2𝑔 𝑒𝑔
(c) Tridentate ligand with three “N” donor 3 3
(c) 𝑒 𝑡2𝑔
atoms 2
(d) 𝑒 4 𝑡2𝑔
(d) Dexadentate ligand with four “O” and
two “N” donor atoms 15. Iron carbonyl, 𝐹𝑒(𝐶𝑂)5 is (2018)
11. Match List-I with List-II: (2021) (a) Tetranuclear
List-I List-II (b) Mononuclear
[𝐹𝑒(𝐶𝑁)6 ]3− (i) 5.92 BM (c) Dinuclear
(A) (d) Trinuclear
(B) [𝐹𝑒(𝐻2 𝑂)6 ]3+ (ii) 0 BM
16. The type of isomerism shown by the
(C) [𝐹𝑒(𝐶𝑁)6 ]4− (iii) 4.90 BM complex [𝐶𝑜𝐶𝑙2 (𝑒𝑛)2 ] is: (2018)
(D) [𝐹𝑒(𝐻2 𝑂6 )2+ (iv) 1.73 BM (a) Geometrical isomerism
(b) Coordination isomerism
Choose the correct answer from the options
(c) Linkage isomerism
given below
(d) Ionization isomerism
(a) A-(ii) B-(iv) C-(iii) D-(i)
(b) A-(i) B-(iii) C-(iv) D-(ii) 17. The geometry and magnetic behaviour of
(c) A-(iv) B-(i) C-(ii) D-(iii) the complex [𝑁𝑖(𝐶𝑂)4 ] are? (2018)
(d) A-(iv) B-(ii) C-(i) D-(iii) (a) Square planar geometry and
12. Which of the following is the correct order of diamagnetic
increasing field strength of ligands to form Tetrahedral geometry and diamagnetic
coordination compounds? (2020) (c) Tetrahedral geometry and paramagnetic
(a) 𝑆𝐶𝑁 − < 𝐹 − < 𝐶𝑁 − < 𝐶2 𝑂42− (d) Square planar geometry and
(b) 𝐹 − < 𝑆𝐶𝑁 − < 𝐶2 𝑂42− < 𝐶𝑁 − paramagnetic
(c) 𝐶𝑁 − < 𝐶2 𝑂42− < 𝑆𝐶𝑁 − < 𝐹 −
18. Correct increasing order for the
(d) 𝑆𝐶𝑁 − < 𝐹 − < 𝐶2 𝑂42− < 𝐶𝑁
wavelengths of absorption in the visible
13. Match the coordination number and type of region for the complexes of 𝐶𝑜 3+ is:
hybridization with distribution of hybrid (2017-Delhi)
orbitals in space based on Valence bond (a) [𝐶𝑜(𝑁𝐻3 )6 ]3+ , [𝐶𝑜(𝑒𝑛)3 ]3+ , [𝐶𝑜(𝐻2 𝑂)6 ]3+
theory: (2020 Covid Re-NEET) (b) [𝐶𝑜(𝑒𝑛)3 ]3+ , [𝐶𝑜(𝑁𝐻3 )6 ]3+ , [𝐶𝑜(𝐻2 𝑂)6 ]3+
Coordination Distribution (c) [𝐶𝑜(𝐻2 𝑂)6 ]3+ , [𝐶𝑜(𝑒𝑛)3 ]3+ , [𝐶𝑜(𝑁𝐻3 )6 ]3+
number and of hybrid (d) [𝐶𝑜(𝐻2 𝑂)6 ]3+ , [𝐶𝑜(𝑁𝐻3 )6 ]3+ , [𝐶𝑜(𝑒𝑛)3 ]3+
type of orbitals in
19. Pick out the correct statement with respect
hybridization space
to [𝑀𝑛(𝐶𝑁)6 ]3− (2017-Delhi)
(A) 4, 𝑠𝑝3 (i) Trigonal (a) It is 𝑑𝑠𝑝 hybridised and square planar
2
bipyramidal (b) It is 𝑠𝑝3 𝑑2 hybridised and octahedral
(B) 4, 𝑑𝑠𝑝2 (ii) Octahedral (c) It is 𝑠𝑝3 𝑑2 hybridised and tetrahedral
5, 𝑠𝑝3 𝑑 (iii) Tetrahedral (d) It is 𝑑2 𝑠𝑝3 hybridised and octahedral
(C)
2
6, 𝑑 𝑠𝑝 3 (iv) Square planar 20. The correct order of the stoichiometries of
(D)
𝐴𝑔𝐶𝑙 formed when 𝐴𝑔𝑁𝑂3 in excess is
Select the correct option: treated with the complexes:
(a) A-(iii) B-(iv) C-(i) D-(ii) 𝐶𝑜𝐶𝑙3 . 6𝑁𝐻3 , 𝐶𝑜𝐶𝑙3 . 5𝑁𝐻3 𝐶𝑜𝐶𝑙3 . 4𝑁𝐻3
(b) A-(iv) B-(i) C-(ii) D-(iii) respectively is: (2017-Delhi)
(c) A-(iii) B-(i) C-(iv) D-(ii) (a) 2𝐴𝑔𝐶𝑙, 3𝐴𝑔𝐶𝑙, 1𝐴𝑔𝐶𝑙
(d) A-(ii) B-(iii) C-(iv) D-(i) (b) 1𝐴𝑔𝐶𝑙, 3𝐴𝑔𝐶𝑙, 2𝐴𝑔𝐶𝑙
(c) 3𝐴𝑔𝐶𝑙, 1𝐴𝑔𝐶𝑙, 2𝐴𝑔𝐶𝑙
(d) 3𝐴𝑔𝐶𝑙, 2𝐴𝑔𝐶𝑙, 1𝐴𝑔𝐶𝑙

89
21. Which of the following complex ions is not 28. The name of complex ion, [𝐹𝑒(𝐶𝑁)6 ]3− is:
diamagnetic? (2017-Gujarat) (2015 Re)
(a) [𝑆𝑐(𝐻2 𝑂)3 (𝑁𝐻3 )3 ]3+
(a) Hexacyanidoferrate (III) ion
(b) [𝑇𝑖(𝑒𝑛)2 (𝑁𝐻3 )2 ]4+ (b) Hexacyanoiron (III) ion
(c) [𝐶𝑟(𝑁𝐻3 )6 ]3+ (c) Hexacyanoferrate (III) ion
(d) [𝑍𝑛(𝑁𝐻3 )6 ]2+ (d) Tricyanoferrate (III) ion
22. For the tetrahedral complex [𝑀𝑛𝐵𝑟4 ]2−, the 29. The hybridization involved in complex
spin only magnetic moment value is: [Ni(CN)4]2 is
(2017-Gujarat) (Atomic Number Ni = 28) (2015 Re)
(a) 2.4 (a) 𝑑2 𝑠𝑝3
(b) 1.7 (b) 𝑑𝑠𝑝2
(c) 5.9 (c) 𝑠𝑝3
(d) 4.8 (d) 𝑑2 𝑠𝑝2
23. The electron distribution in 𝑑𝑛 coordination 30. Number of possible isomers for the complex
complexes depends on magnitude of crystal [𝐶𝑜(𝑒𝑛)2 𝐶𝑙2 ] Cl will be:
field splitting, (Δ0 ) and pairing energy (P). (en = ethylenediamine) (2015 Re)
The condition which favours formation of (a) 4
high spin complexes is: (2017-Gujarat) (b) 2
(a) 𝑡2𝑔 𝑒𝑔
4 0 (c) 1
(b) Δ0 > 𝑃 (d) 3
(c) Δ0 < 𝑃 31. Which of these statements about
(d) Δ0 = 𝑃 [𝐶𝑜(𝐶𝑁)6 ]3− is true? (2015)
24. The [𝐶𝑜(𝐻2 𝑂)6 ]2+ ion has three unpaired (a) [𝐶𝑜(𝐶𝑁)6 ]3− has four unpaired electrons
electrons. The hybridization of Co in and will be in a low-spin configuration
[𝐶𝑜(𝐻2 𝑂)6 ]2+ i has four unpaired
(a) 𝑑2 𝑠𝑝3 electrons and will be in a high-spin
(b) 𝑠𝑝3 configuration
(c) 𝑑𝑠𝑝2 (c) [𝐶𝑜(𝐶𝑁)6 ]3− has no unpaired electrons
(d) 𝑠𝑝3 𝑑2 and will be in a high-spin configuration
(d) [𝐶𝑜(𝐶𝑁)6 ]3− has no unpaired electrons
25. The correct increasing order of trans-effect
of the following species is: and will be in a low-spin configuration
(a) 𝑁𝐻3 > 𝐶𝑁 − > 𝐵𝑟 − 𝐶6 𝐻5− 32. Cobalt(III) chloride forms several octahedral
(b) 𝐶𝑁 − > 𝐶6 𝐻5− > 𝐵𝑟 − > 𝑁𝐻3 complexes with ammonia. Which of the
(c) 𝐵𝑟 − > 𝐶𝑁 − > 𝑁𝐻3 > 𝐶6 𝐻5− following will not give test for chloride ions
(d) 𝐶𝑁 − > 𝐵𝑟 − > 𝐶6 𝐻5− > 𝑁𝐻3 with silver nitrate at 25°C? (2015)
26. Jahn-Teller effect is not observed in high (a) 𝐶𝑜𝐶𝑙3 . 4𝑁𝐻3
spin complexes of: (b) 𝐶𝑜𝐶𝑙3 . 5𝑁𝐻3
(a) 𝑑7 (c) 𝐶𝑜𝐶𝑙3 . 6𝑁𝐻3
(b) 𝑑8 (d) 𝐶𝑜𝐶𝑙3 . 3𝑁𝐻3
(c) 𝑑4 33. The sum of corrdination number and
(d) 𝑑9 oxidation number of the metal M in the
27. Which of the following has longest C-O bond complex [𝑀(𝑒𝑛)2 (𝐶2 𝑂4 )]𝐶𝑙 (where en is
length? ethylenediamine) is: (2017 Re)
(Free C–O bond length in CO is 1.128Å) (a) 6
(2016-I) (b) 7
(a) [𝑀𝑛(𝐶𝑂) ]+ (c) 8
6
(b) 𝑁𝑖(𝐶𝑂)4 (d) 9
(c) [𝐶𝑜(𝐶𝑂)4 ]− 34. Among the following complexes the one
(d) [𝐹𝑒(𝐶𝑂)4 ]2− which shows zero crystal field stabilization
energy (CFSE) is: (2014)

90
(a) [𝐹𝑒(𝐻2 𝑂)6 ]3+
(b) [𝐶𝑜(𝐻2 𝑂)6 ]2+
(c) [𝐶𝑜(𝐻2 𝑂)6 ]3+
(d) [𝑀𝑛(𝐻2 𝑂)6 ]3+
35. Which of the following complexes is used to
be as an anticancer agent? (2014)
(a) 𝑐𝑖𝑠 − [𝑃𝑡𝐶𝑙2 (𝑁𝐻3 )2 ]
(b) 𝑐𝑖𝑠 − 𝐾2 [𝑃𝑡𝐶𝑙2 𝐵𝑟2 ]
(c) 𝑁𝑎2 [𝐶𝑜𝐶𝑙4 ]
(d) 𝑚𝑒𝑟 − [𝐶𝑜(𝑁𝐻3 )3 𝐶𝑙3 ]

91
Answer Key
S1. Ans. (b) S31. Ans. (d)

S2. Ans. (c) S32. Ans. (d)

S3. Ans. (d) S33. Ans. (d)

S4. Ans. (d) S34. Ans. (a)

S5. Ans. (b) S35. Ans. (a)

S6. Ans. (d)

S7. Ans. (d)

S8. Ans. (d)

S9. Ans. (c)

S10. Ans. (d)

S11. Ans. (c)

S12. Ans. (d)

S13. Ans. (a)

S14. Ans. (b)

S15. Ans. (b)

S16. Ans. (a)

S17. Ans. (b)

S18. Ans. (b)

S19. Ans. (d)

S20. Ans. (d)

S21. Ans. (c)

S22. Ans. (c)

S23. Ans. (c)

S24. Ans. (d)

S25. Ans. (b)

S26. Ans. (b)

S27. Ans. (d)

S28. Ans. (a)

S29. Ans. (b)

S30. Ans. (d)

92
Solutions
S1. Ans.(b) ⟹ 'en' has a stronger field strength than
Maximum covalency of boron is four. 'H2O' according to spectrochemical
series.
S2. Ans.(c)
∴ Correct order of energy absorbed will
Complex salt is K2[Pt(NH3)2Cl2] be:
Double salt is KAl(SO4)2∙12H2O (ptash [Ni(en)3]2+ > [Ni(H2O)2(en)2]2+ >
alum) [Ni(H2O)4(en)]2+
S3. Ans.(d) i.e., C > A > B
Given complex compounds exhibit S10. Ans.(d)
solvate isomerism having co-ordination S11. Ans.(c)
number = 6.
S12. Ans.(d)
S4. Ans.(d)
Spectrochemical series the correct order
(1) [Pt(NH3)2Cl(NO2)]
of increasing field strength of ligands to
(2) [Co(NH3)5(CO3)]Cl
form coordination compounds
(3) [Cr(NH3)3(H2O)3]Cl3
(4) K3[Al(C3O4)3] 𝐼 − < 𝐵𝑟 − < 𝑆 2− < 𝑆𝐶𝑁 − < 𝐶𝑙 − < 𝑁3−

Option 4 contain all ligands are of same < 𝐹 − < 𝑂𝐻 − < 𝐶 𝑂2− < 𝐻2 𝑂 <
type i.e., why complex will be homoleptic.
< 𝑒𝑛 < 𝐶𝑁 −
S5. Ans.(b)
Due to Chelation effect of (en). < 𝐶𝑂
S6. Ans.(d) Thus option d is correct.
A-3, B-2, C-1, D-4
S13. Ans.(a)
S7. Ans.(d)
Hybridisation Geometry Coordination
number

𝑆𝑝3 Tetrahedral 4

𝑑𝑠𝑝2 Square 4
In case of metal carbonyls, the bonding planar
has both 𝜎 and 𝜋 nature, where ligand to
𝑆𝑝3 𝑑 Trigonal 5
metal bond is '𝜎' (coordinate) bond and
metal to ligand bond is '𝜋' (synergic) bipyramidal
bond. Octahedral 6
𝑑2 𝑠𝑝3
S8. Ans.(d)
Spectrochemical
[Ag(H2O)2][Ag(CN)2]
IUPAC name: S14. Ans.(b)
diaquasilver(I) dicyanidoargentate(I) 𝐾4 [𝐹𝑒(𝐶𝑁)6 ]

S9. Ans.(c) Fe ground state : [𝐴𝑟]3𝑑6 4𝑠 2

Stronger the field strength of ligand, 𝐹𝑒 2+ : 3𝑑6 4𝑠 0


higher will be the energy absorbed by the
complex.

93
For, four ‘CO’-ligands hybridization
would be 𝑠𝑝3 and thus the complex
would be diamagnetic and of tetrahedral
geometry.

S15. Ans.(d)
Based on the number of metal atoms
present in a complex, they are classified
into mononuclear, dinuclear, trinuclear
and so on.

Eg: 𝐹𝑒(𝐶𝑂)5 : mononuclear

𝐶𝑜2 (𝐶𝑂)8 : dinuclear


S18. Ans.(d)
𝐹𝑒3 (𝐶𝑂)12 :
The increasing order for the wavelengths
Hence, option (b) should be the right
of absorption in the visible region for the
answer.
complexes of 𝐶𝑜 is:-
S16. Ans.(a)
[𝐶𝑜(𝑒𝑛)]3+ , [𝐶𝑜(𝑁𝐻3 )6 ]3+ , [𝐶𝑜(𝐻2 𝑂)6 ]3+
In [𝐶𝑜𝐶𝑙2 (𝑒𝑛)2 ], coordination number of
As the spectro chemical series is:
𝐶𝑜 is 6 and this compound has
octahedral geometry. 𝐼 − < 𝐵𝑟 − < 𝑆𝐶𝑁 − < 𝐶𝑙 − < 𝑆 2− < 𝐹 −

< 𝑂𝐻 − < 𝐶2 𝑂42− < 𝐻2 𝑂 < 𝑁𝐶𝑆 − < 𝐸𝐷𝑇𝐴4+

< 𝑁𝐻3 < 𝑒𝑛 < 𝐶𝑁 − < 𝐶𝑂.

S19. Ans.(d)
[Mn(CN)6]3–

𝑥 + 6(−1) = −3

As per given option, type of isomerism is 𝑥 − 6 = −3


geometrical isomerism. 𝑥 = −3 + 6 ⟹ 𝑥 = +3
S17. Ans.(b)
𝑁𝑖(28): [𝐴𝑟]3𝑑8 4𝑠 2

∵ Co is a strong field ligand


In presence of 𝐶𝑁 − causes pairing
configuration would be:

94
S25. Ans.(b)
The intensity of the trans-effect as
measured by the increase in rate of
substitution of the trans ligand follows

𝑑2 𝑠𝑝3 the sequence: 𝐶𝑁 − > 𝐶6 𝐻5− > 𝐵𝑟 − > 𝑁𝐻3

∴ 𝑑2 𝑠𝑝3 is required S26. Ans.(b)


Jahn – teller distortion is usually
Hence, it is octahedral in shape.
significant for asymmetrically occupied
S20. Ans.(d)
𝑒𝑔 orbital. In case unevenly occupied 𝑡2𝑔
Complexes are respectively
orbital & Jahn-teller distortion is very
[𝐶𝑜(𝑁𝐻3 )6 ]𝐶𝑙3 , [𝐶𝑜(𝑁𝐻3 )5 𝐶𝑙] 𝐶𝑙2 and
weak.
[𝐶𝑜(𝑁𝐻3 )4 𝐶𝑙2 ]𝐶𝑙.

S21. Ans.(c)
[𝐶𝑟(𝑁𝐻3 )6 ]3+

S27. Ans.(d)
Due to back bonding between metal
∴ They have 3 unpaired electron.
carbonyl bond, length of 𝐶 − 𝑂 increase.
∴ They are paramagnetic in nature. Also, higher the charge on central metal
S22. Ans.(c) atom higher will be the back bonding
In [MnBr4]2– (synergic effect).
𝑀𝑛2+ = [𝐴𝑟]18 3𝑑5 4𝑠 0 . S28. Ans.(a)
[𝑀𝑛𝐵𝑟4 ]2− is tetrahedral complex Name of complex ion [𝐹𝑒(𝐶𝑁)6 ]3− is
𝑛 = 5 Hexacyaniodoferrate (III) ion
𝜇 = √𝑛(𝑛 + 2) = √5(5 + 2) = √5(7) = √35
Oxidation state of Fe is +3.
= 5.9
S29. Ans.(b)
S23. Ans.(c)
𝑃 > Δ0 is a condition which favours
formation of high spin complexes.

S24. Ans.(d)
[𝐶𝑂(𝐻2 𝑂)6 ]2+, N = 3 unpaired electrons.

𝐶𝑜 2+ : [𝐴𝑟]3𝑑7
As 𝐶𝑁 – is a strong field ligand it will
form a low spin complex with 𝑑𝑠𝑝2
hybridization.

𝑠𝑝3 𝑑 2

95
S30. Ans.(d)

S31. Ans.(d)

As 𝐶𝑁 − being a strong field ligand will


form a low-spin complex.

S32. Ans.(d)
Cobalt III Chloride means that the
coordinate ion number of 𝐶𝑜 3+ is 6, so
compound must be [𝐶𝑜(𝑁𝐻3 )𝐶𝑙3 ].

S33. Ans.(d)
[𝑀(𝑒𝑛2 )(𝐶2 𝑂 )]𝐶𝑙

Oxidation number of metal = 3

Coordination number of metal = 6

Sum of oxidation and coordination


number

=3+6

=9

S34. Ans.(a)

In [Fe(H2O)6]3+, Fe has oxidation state of


+3
3
𝑑5 = 𝑡2𝑔 & 𝑒𝑔2

C.F.S.E. = 0

A high spin complex is formed because


H2O is a weak field ligand.

+0.6 × 2 – 0.4 × 3 = 0

S35. Ans.(a)
Cis platin: Cis [PtCl2(NH3)2] is used as
an anticancer agent.

96
Chapter 15
Organic Chemistry - Some Basic Principles
and Techniques

1. Which amongst the following compounds/ (a) Statement I is true but Statement II is
species is least basic? (2023) false.
(b) Statement I is false but Statement II is
true.
(c) Both Statement I and Statement II are
(a) true.
(d) Both Statement I and Statement II are
false.
(b) 5. The number of 𝜎 bonds, 𝜋 bonds and lone
pair of electrons in pyridine, respectively
are: (2023)
(a) 12, 3, 0
(c) (b) 11, 3, 1
(c) 12, 2, 1
(d) 11, 2, 0
(d) 6. In Lassaigne's extract of an organic
compound, both nitrogen and Sulphur are
2. Which amongst the following compounds
present, which gives blood red colour with
will show geometrical isomerism?
Fe due to the formation of- (2023)
(a) Pent-1-ene
(a) NaSCN
(b) 2,3-Dimethylbut-2-ene
(b) [Fe(CN)5NO]4-
(c) 2-Methylprop-1-ene
(c) [Fe(SCN))]2+
(d) 3,4-Dimethylhex-3-ene
(d) Fe4[Fe(CN)6)]3∙xH2O
3. The correct order for the rate of 𝛼, 𝛽-
7. Consider the following compound/species:
dehydrohalogenation for the following
compound is _______. (2023)

(a) i < ii < iii


(b) ii < i < iii
(c) iii < ii < i
(d) ii < iii < i
4. Given below are two statements: (2023)
Statement I: In an organic compound,
when inductive and electromeric effects The number of compounds species which
operate in opposite directions, the inductive obey Hucker's rule is _______. (2023)
effect predominates. (a) 6
Statement II: Hyperconjugation is (b) 2
observed in o-xylene. (c) 5
In the light of the above statements, choose (d) 4
the correct answer from the options given
below:
97
8. Given below are two statements: One is
labelled as Assertion (A) and the other is
labelled as Reason (R). (2022)
Assertion (A): Chlorine is an electron
withdrawing group but it is ortho, para (c)
directing in electrophilic aromatic.
Reason (R): Inductive effect of chlorine
destabilises the intermediate carbocation
formed during the electrophilic
(d)
substitution, however due to carbocation at
ortho and para positions. 12. Which compound amongst the following is
In the light of the above statements, choose not an aromatic compound? (2022)
the most appropriate answer from the
options given below:
(a) (A) is not correct but (R) is correct.
(b) Both (A) and (R) are correct and (R) is
the correct explanation of (A). (a)
(c) Both (A) and (R) are correct but (R) is not
the correct explanation of (A).
(d) (A) is correct but (R) is not correct.
9. Predict the order of reactivity of the
following four isomers towards SN2
reaction. (2022) (b)
1. CH3CH2CH2CH2Cl
2. CH3CH2CH(Cl)CH3
3. (CH3)2CHCH2Cl (c)
4. (CH3)3CCl
(a) 4 > 2 > 3 > 1
(b) 4 > 3 > 2 > 1
(c) 1 > 2 > 3 > 4
(d) 1 > 3 > 2 > 4
(d)
10. What is the hybridization shown by C1 and
C2 carbons, respectively in the given 13. The incorrect statement regarding chirality
compound? (2022) is (2022)
OHC–CH=CH–CH3COOCH3 (a) SN1 reaction yields 1 : 1 mixture of both
(a) sp3 and sp3 enantiomers
(b) sp2 and sp3 (b) The product obtained by SN2 reaction of
(c) sp2 and sp2 haloalkane having chirality at the
(d) sp3 and sp2 reactive site shows inversion of
configuration
11. Which of the following is the most stable (c) Enantiomers are superimposable mirror
carbocation? (2022) images on each other
(d) A racemic mixture shows zero optical
rotation
14. The Kjeldahl's method for the estimation of
(a) nitrogen can be used to estimate the
amount of nitrogen in which one of the
following compounds? (2022)

(b)

98
(c)

(a)

(d)
18. Paper chromatography is an example of :
(2020)
(b)
(a) Partition chromatography
(b) Thin layer chromatography
(c) Column chromatography
(d) Adsorption chromatography
19. A tertiary butyl carbocation is more stable
than a secondary butyl carbocation
(c) because of which of the following?
(2020)
(a) +R effect of – CH3 groups
(b) –R effect of – CH3 groups
(c) Hyperconjugation
(d)
–I effect of –CH3 groups
15. The correct IUPAC name of the following
20. A liquid compound (x) can be purified by
compound is: (2022)
steam distillation only if it is
(2020 Covid Re-NEET)
(a) Not steam volatile, miscible with water
(b) Steam volatile, miscible with water
(c) Not steam volatile, immiscible with
(a) 1-bromo-5-chloro-4-methylhexan-3-ol water
(b) 6-bromo-2-chloro-4-methylhexan-4-ol (d) Steam volatile, immiscible with water
(c) 1-bromo-4-methyl-5-chlorohexan-3-ol 21. Which of the following molecules represents
(d) 6-bromo-4-methyl-2-chlorohexan-4-ol the order of hybridization 𝑠𝑝2 , 𝑠𝑝2 , 𝑠𝑝, 𝑠𝑝
16. The compound which shows metamerism from left to right atoms? (2018)
is: (2021) (a) 𝐻𝐶 ≡ 𝐶 − 𝐶 ≡ 𝐶𝐻
(a) 𝐶3 𝐻8 𝑂 (b) 𝐶𝐻2 = 𝐶𝐻 − 𝐶 ≡ 𝐶𝐻
(b) 𝐶3 𝐻6 𝑂 (c) 𝐶𝐻3 − 𝐶𝐻 = 𝐶𝐻 − 𝐶𝐻3
(c) 𝐶4 𝐻10 𝑂 (d) 𝐶𝐻2 = 𝐶𝐻 − 𝐶𝐻 = 𝐶𝐻2
(d) 𝐶5 𝐻12
22. Which of the following carbocations is
17. The correct structure of 2, 6-Dimethyl-dec- expected to be most stable? (2018)
4-ene is: (2021)

(a)
(a)

(b)

99
accepting a pair of electrons from a
nucleophile
(c) Electrophile is a negatively charged
species and can form a bond by
accepting a pair of electrons from
another electrophile
(b) (d) Electrophiles are generally neutral
species and can form a bond by
accepting a pair of electrons from a
nucleophile

(c) 27. Amongst the following compounds, the one


which is most easily sulphonated is:
(2017-Gujarat)
(a) Chlorobenzene
(b) Benzene
(c) Nitro benzene
(d) Toluene
(d)
28. Match Column-I with Column-II
23. Which of the following is correct with (2017-Gujarat)
respect to –I effect of the substituents? (R = Column-I Column-II
alkyl) (2018)
(a) −𝑁𝐻2 < −𝑂𝑅 < −𝐹
(b) −𝑁𝑅2 < −𝑂𝑅 < −𝐹
(c) −𝑁𝑅2 > −𝑂𝑅 > −𝐹
(d) −𝑁𝐻2 > −𝑂𝑅 > −𝐹
24. The most suitable method of separation of
(A)
1 : 1 mixture of ortho and para-
(P) Electrophilic
nitrophenols is (2017-Delhi)
Substitution
(a) Steam distillation
(b) Sublimation
(c) Chromatography
(d) Crystallisation
25. The IUPAC name of the compound

(B)
(Q) Nucleophilic
is Substitution
(2017-DELHI)
(a) 3-keto-2-methylhex-5-enal
(b) 3-keto-2-methylhex-4-enal
(C) d
(c) 5-formylhex-2-en-3-one
(R) Nucleophilic
(d) 5-methyl-4-oxohex-2-en-5-al
addition
26. The correct statement regarding
electrophile is: (2017-Delhi) (a) 𝐴 − 𝑅; 𝐵 − 𝑄; 𝐶 − 𝑃
(a) Electrophile can be either neutral or (b) 𝐴 − 𝑃; 𝐵 − 𝑄; 𝐶 − 𝑅
positively charged species and can form (c) 𝐴 − 𝑄; 𝐵 − 𝑅; 𝐶 − 𝑃
a bond by accepting a pair of electrons (d) 𝐴 − 𝑅; 𝐵 − 𝑃; 𝐶 − 𝑄
from a nucleophile
29. Which of the following statements is
(b) Electrophile is a negatively charged
incorrect? (2017-Gujarat)
species and can form a bond by
100
− +
(a) Of 𝑂 − 𝐶 ≡ 𝑂, 𝑂 = 𝐶 = 𝑂, the structures,
− +
𝑂 − 𝐶 ≡ 𝑂, is most stable structure
(b) The bond angle follows the order (c)
𝐶𝐻4 > 𝑁𝐻3 > 𝐻2 𝑂 > 𝐻2 𝑆
(c) The bond order follows the order
𝑂2+ > 𝑂2 > 𝑂2− > 𝑂22−
(d)
(d) Strength of ‘H’ bond follows the order
34. Which of the following is the most correct
𝐻𝐹 > 𝐻2 𝑂 > 𝑁𝐻3 > 𝐻𝐶𝑙
electron displacement for a nucleophilic
30. Which among the given molecules can reaction to take place? (2015)
exhibit tautomerism?
(2016-II)
(a)

(b)
(a) III only
(b) Both I and III
(c) Both I and II (c)
(d) Both II and III
31. Which of the following statements is not (d)
correct for a nucleophile? (2015 Re)
(a) Nucleophiles are not electron seeking 35. In Duma’s method for estimation of
(b) Nucleophile is a lewis acid nitrogen, 0.25 g of an organic compound
(c) Ammonia is a nucleophile gave 40 mL of nitrogen collected at 300 K
(d) Nucleophiles attack low electron density temperature and 725 mm pressure. If the
aqueous tension at 300 K is 25 mm, the
sites percentage of nitrogen in the compound is:
32. Consider the following compounds (a) 18.20
(b) 16.76
(c) 15.76
(d) 17.36
36. The reaction of 𝐶6 𝐻5 𝐶𝐻 = 𝐶𝐻𝐶𝐻3 with HBr
produces:
(2015) (2015)
Hyperconjugation occurs in:
(a) II only
(b) III only (a)
(c) I and III
(b)
(d) I only
33. In which of the following compounds, the
𝐶 − 𝐶𝑙 bond ionization shall give most stable
carbonium ion? (2015)

(c)
(a)
(d)
37. Which of the following is not the product of
dehydration of (2015 Re)
(b)

101
(a) (b)

(b)

(c)

(c)

(d)

(d) 42. In the Kjeldahl’s method for estimation of


nitrogen present in a soil sample, ammonia
evolved from 0.75 g of sample neutralized
10 mL of 1 𝑀𝐻2 𝑆𝑂4 . The percentage of
38. Treatment of cyclopentanone nitrogen in the soil is (2014)
with methyl lithium gives which of the (a) 45.33
following species? (2015) (b) 35.33
(a) Cyclopentanonyl radical (c) 43.33
(b) Cyclopentanonyl biradical (d) 37.33
(c) Cyclopentanoyl anion
43. The structure of isobutyl group in an
(d) Cyclopentanonyl cation
organic compound is: (2013)
39. Which of the following species contains
equal number of 𝜎-bonds and 𝜋-bonds?
(2015)
(a)
(a) (𝐶𝑁)2
(b) 𝐶𝐻2 (𝐶𝑁)2
(c) 𝐻𝐶𝑂3− (b)
(d) 𝑋𝑒𝑂4
(c)
40. The enolic form of ethyl acetoacetate as
shown below has (2015)

(d)

(a) 9 𝜎-bonds and 2 𝜋-bonds 44. Structure of the compound whose IUPAC
(b) 9 𝜎-bonds and 1 𝜋-bonds name is 3-Ethyl-2-hydroxy-4-methylhex-3-
(c) 18 𝜎-bonds and 2 𝜋-bonds en-5-ynoic acid is: (2013)
(d) 16 𝜎-bonds and 1 𝜋-bonds
41. Which one is most reactive towards
nucleophilic addition reaction? (2014)
(a)

(a)

(b)
102
(c)

(d)
45. Some meta-directing substituents in
aromatic substitution are given. Which one
is most deactivating? (2013)
(a) −𝐶𝑂𝑂𝐻
(b) −𝑁𝑂2
(c) −𝐶 ≡ 𝑁
(d) −𝑆𝑂3 𝐻

103
Answer Key
S1. Ans. (b) S32. Ans. (b)

S2. Ans. (d) S33. Ans. (a)

S3. Ans. (d) S34. Ans. (b)

S4. Ans. (b) S35. Ans. (b)

S5. Ans. (b) S36. Ans. (d)

S6. Ans. (c) S37. Ans. (a)

S7. Ans. (d) S38. Ans. (c)

S8. Ans. (d) S39. Ans. (d)

S9. Ans. (d) S40. Ans. (c)

S10. Ans. (b) S41. Ans. (c)

S11. Ans. (a) S42. Ans. (d)

S12. Ans. (d) S43. Ans. (a)

S13. Ans. (c) S44. Ans. (b)

S14. Ans. (c) S45. Ans. (b)

S15. Ans. (a)

S16. Ans. (c)

S17. Ans. (d)

S18. Ans. (a)

S19. Ans. (c)

S20. Ans. (d)

S21. Ans. (b)

S22. Ans. (d)

S23. Ans. (a, b)

S24. Ans. (a)

S25. Ans. (b)

S26. Ans. (a)

S27. Ans. (d)

S28. Ans. (d)

S29. Ans. (a)

S30. Ans. (a)

S31. Ans. (b)

104
Solutions
S1. Ans.(b) Cl has pronounced −I effect than +R
effect due to large size difference between
carbon and chlorine, also due to high
electronegativity of chlorine.
S9. Ans.(d)
S2. Ans.(d)
Lesser the steric hinderance on halide
carbon, more will be the reactivity of alkyl
halide towards SN2 reaction.
So correct order towards SN2 reactivity is:
S3. Ans.(d)

S10. Ans.(b)
−COOCH3 has higher priority than – C =
C – and – CHO in IUPAC nomenclature.

Rate of Dehydrohalogenation: II < III < I.

S4. Ans.(b)
C = sp
Statement I is false but Statement II is
C = sp
true.
S11. Ans.(a)
S5. Ans.(b)

(1)
⟶ Cross conjugation and 3α-H
S6. Ans.(c) [Hyperconjugation]
In case nitrogen and sulphur both are Due to cross-conjugation and 3α-H
present in an organic compound, sodium [Hyperconjugation], (1) is most stable.
thiocyanate is formed, it give blood red
colour and no Prussian blue since there
are no free cyanide Ions.
Na + C + N + S ⟶ NaSCN
(2)
Fe+3 + SCN ⊝ ⟶ [Fe(SCN)]2+ Blood red
⟶ Resonance & 2α-H [Hyperconjugation]
S7. Ans.(d)
Huckle's rule = (4n + 5)π electrons

Comp (i), (ii), (v), (vii) obey Huckle's rule

S8. Ans.(d) (3)


⟶ 1α-H [Hyperconjugation]

105
suspended in a mixture of solvents that
act as a mobile phase.

S19. Ans.(c)

(4)
⟶ 3α-H [Hyperconjugation]
S12. Ans.(d)
Planar, cyclic, conjugated species
containing (4n + 2)π electrons will be
aromatic in nature (n is an integer) A tertiary butyl carbocation has 9 alpha
hydrogen atoms whereas secondary
butyl carbocation has 5 alpha hydrogen
atoms.

So when more number of a-H atoms is


present, more will be the
are aromatic species.
hyperconjugation effect hence more will
S13. Ans.(c)
be the stability of carbocation.
The stereoisomers related to each other
as non-superimposable mirror image are Thus tertiary butyl carbocation is more
called enantiomers. stable than a secondary butyl
S14. Ans.(c) carbocation due to hyperconjugation
Kjeldahl method is not applicable to
compounds containing nitrogen in nitro S20. Ans.(d)
group, azo groups and nitrogen present Steam distillation technique is applied
in the ring (e.g., pyridine) as nitrogen of to separate the substances which are
these compounds does not change to
steam volatile and immiscible with
ammonium sulphate under these
conditions. water

S15. Ans.(a) S21. Ans.(b)


𝑠𝑝2 𝑠𝑝2 𝑠𝑝 𝑠𝑝
𝐶 𝐻2 = 𝐶 𝐻 − 𝐶 ≡ 𝐶 𝐻 Number of
orbital require in hybridization =
Number of 𝜎- bonds around each
1-bromo-5-chloro-4-methylhexan-3-ol
carbon atom.
S16. Ans.(c)
S17. Ans.(d) S22. Ans.(d)
S18. Ans.(a) NO2 group exhibit-I effect and it
The example of partition decreases with increase in distance. In
chromatography is seen in paper option (d), positive charge present on C
chromatography. atom at maximum distance S –I effect

In this process, chromatography paper reaching to it is minimum and stability

is used as a stationary phase which is is maximum.

106
S23. Ans.(a, b) Nucleophile: A negatively charged
–I effect increases on increasing species or neutral species can accepted
electronegativity of atom. So, correct by positive charge or donate their
order of –I effect is electron to positively charge species or

–NH2 < – OR < – F. neutral species.

Also –NR2 < –OR < –F S27. Ans.(d)


In toluene CH3 group is attached which
S24. Ans.(a)
is donating in nature. Therefore; it can
The most suitable method of separation
easily be sulphonated.
of 1 : 1 mixture of ortho and para –
nitrophenol is steam distillation, S28. Ans.(d)
because there is large difference in the
boiling point of ortho and para position
as ortho form intra hydrogen bonding
and there is large difference in the
boiling point of ortho and para position
as ortho form intra hydrogen bonding
and para form inter-molecular hydrogen
bonding.

S25. Ans.(b)

→ −𝑘𝑒𝑡𝑜 − 2 −
𝑚𝑒𝑡ℎ𝑦𝑙ℎ𝑒𝑥 − 4 − 𝑒𝑛𝑎𝑙

The order of decreasing priority for some


functional groups is:-
–COOH, –SO3H, –COOR, –COCl, –
CONH2, –CN, –HC = O, > C = O, –OH, – S29. Ans.(a)
⊖ ⊕
NH2, > C = C <, –C ≡C–.
𝑂 − 𝐶 ≡ 𝑂 is less stable than O = C = C,
S26. Ans.(a)
as charge separation occur in it.
Electrophile can be either neutral or
S30. Ans.(a)
positively charged species and can form
𝛼 − 𝐻 at bridge carbon never participate
a bond by accepting a pair of electrons
in tautomerism.
from nucleophile.
Thus, only (III) shows tautomerism.
An electron loving species is called
electrophile. S31. Ans.(b)
Nucleophiles are positive charge loving
Electrophile: Electron can be accepted
species or electron rich compounds.
by positively charged or neutral species.
Lewis acids are those species which are

107
electron deficient so nucleophiles % of nitrogen =
0.042
× 100 = 16.76%
0.25
cannot act as Lewis acids, they instead S36. Ans.(d)
are Lewis bases. Reaction of 𝐶6 𝐻5 = 𝐶𝐻𝐶𝐻3 with HBr
S32. Ans.(b) yields
Hyperconjugation provides extra
stability by removal of H from 𝛼 carbon
which is only possible in:
As benzoyl carbocation formed here will
be most stable due to resonance.

S37. Ans.(a)

S33. Ans.(a)

3° carbocation is most stable than


S38. Ans.(c)
benzyol carbocation because of 9 hyper-
conjugation of 3 inductive effects of –
CH3.

S34. Ans.(b)
So, the new species formed will be
resonance stabilisied (alternating
positive and negative charge).

S39. Ans.(a)
4 − 𝜎 and 4 − 𝜋 bonds

S35. Ans.(b)
According to Duma’s method:
𝑃1 𝑉1 𝑃2 𝑉2 𝑃2 𝑉2 𝑇1
𝑇1
= 𝑇2
⟹ 𝑉1 = 𝑃1 𝑇2

(725−25)×40 760𝑉2
= S40. Ans.(c)
300 273
𝑉2 = 33.53 ml S41. Ans.(c)
33.53
Mass = × 28 = 0.042 𝑔 𝑁𝑂2 is an electron withdrawing group, it
22400
28×𝑉×100 will favour nucleophilic attack.
%N= 22400×𝑚

108
S42. Ans.(d)
Kjeldahl’s method is used for estimation
of N in organic sample where
1.4 × 𝑁 × 𝑉
%N =
𝑤

N = Normality of acid 𝐻2 𝑆𝑂4 = 2


W = weight of 𝑁𝐻3 evolved
V = Vol. of 𝐻2 𝑆𝑂4
1.4×10×2
= 0.75
= 37.33%
S43. Ans.(a)
Isobutyl group is represented as:

S44. Ans.(b)
Structure of 3-Ethyl-2-hydroxy-4-
methyl hex 3-en-5-ynoic acid

S45. Ans.(b)
−𝑁𝑂2 is most deactivating due to –I and
–M effect.

109
Chapter 16
Hydrocarbons

1. Identify product (A) is the following reaction: Br


|
(2023) (d) CH3 − C − CH2 − CH3
|
CH3

3. Amongst the given options which of the


following molecules/ion acts as a Lewis
acid? (2023)
(a) H2O
(b) BF3
(c) OH −
(d) NH3
4. The decreasing order of boiling points of the
(a) following alkanes is: (2022)
(A) heptane
(B) butane
(C) 2-methylbutane
(D) 2-methylpropane
(E) hexane
(b) Choose the correct answer from the options
given below:
(a) A > E > C > B > D
(b) A > C > E > D > B
(c) C > D > A > E > B
(d) A > E > B > C > D
5. The products A and B in the following
(c) reaction sequence are: (2022)

(d)
2. Consider the following reaction and identify
the product (P). (2023)

(a)

(a) CH3 CH = CH − CH3


(b) CH3 − CH − CH − CH3
| |
CH3 Br
CH3
|
(c) CH3 − C − CH2 Br
|
CH3 (b)

110
(b) 180°
(c) 60°
(d) 0°
9. The major product of the following chemical
reaction is (2021)

(c)

(d) (a)
6. The incorrect method for the synthesis of
alkenes is (2022)
(a) Treating vicinal dihalides with Zn metal
(b) Treating of alkynes with Na inn liquid (b)
NH3
(c) Heating alkyl halides with alcoholic
KOH
(d) Treating alkyl halides in aqueous KOH
solution
7. The correct structure of 2,6-Dimethyl-dec-
4-ene. (2021)

(d)

10.
(a) Consider the above reaction and Identify the
missing reagent/chemical. (2021)
(a) 𝐵2 𝐻6
(b) Red. phosphorus
(c) 𝐶𝑎𝑂
(d) 𝐷𝐼𝐵𝐴𝐿 − 𝐻
(b)
11. Which of the following alkane cannot be
made in good yield by Wurtz reaction?
(2020)
(a) 2, 3-Dimethyl butane
(b) 𝑛-Heptane
(c) (c) 𝑛-Butane
(d) 𝑛-Hexane
12. An alkene on ozonolysis gives methanal as
one of the product. Its structure is : (2020)

(d)
8. Dihedral angle (D.A.) of least stable
conformer of ethane is (2021) (a)
(a) 120°
111
(b)
(a) Na/liquid NH3
(b) H2, Pd/C, quinoline
(c) Zn/HCl
(d) Hg2+/H+, H2O
(c) 17. An alkene “A” on reaction with O3 and Zn –
H2O gives propanone and ethanal in
equimolar ratio. Addition of HCl to alkene
“A” gives “B” as the major product. The
structure of product “B” is: (2019)
(d)
13. Which of the following is a free radical
substitution reaction?
(2020 Covid Re-NEET)
(a) Acetylene with HBr (a)
(b) Methane with Br2/hv
(c) Propene with HBr/(C6H5COO)2
(d) Benzene with Br2/AlCl3 (b)
14. Which of the following compound is most
reactive in electrophilic aromatic
substitution? (2020 Covid Re-NEET)

(a) (d)
18. Among the following, the reaction that
proceeds through an electrophilic
(b) substitution, is: (2019)

(a)

(c)
(b)

(d)
15. The number of sigma (𝜎) and pi (𝜋) bonds in
pent-2-en-4-yne is (2019) (c)
(a) 10 𝜎 bonds and 3 𝜋 bonds
(b) 8 𝜎 bonds and 5 𝜋 bonds
(c) 11 𝜎 bonds and 2 𝜋 bonds
(d) 13 𝜎 bonds and no 𝜋 bonds
(d)
16. The most suitable reagent for the following
19. Hydrocarbon (A) reacts with bromine by
conversion, is: (2019)
substitution to form an alkyl bromide which
by Wurtz reaction is converted to gaseous

112
hydrocarbon containing less than four (d) 𝐶𝐻 ≡ 𝐶𝐻 > 𝐶𝐻2 = 𝐶𝐻2 > 𝐶𝐻3 − 𝐶 ≡ 𝐶𝐻 >
carbon atoms. (A) is (2018)
𝐶𝐻3 − 𝐶𝐻3
(a) 𝐶𝐻 ≡ 𝐶𝐻
(b) 𝐶𝐻2 = 𝐶𝐻2 24. The correct statement regarding ethane
(c) 𝐶𝐻4 conformation is (2017 – Gujarat)
(d) 𝐶𝐻3 − 𝐶𝐻3 (a) Rotation around carbon-carbon bond in
ethane molecule is not possible,
20. The compound C7H8 undergoes the because ethane molecule contains both
following reactions: (2018) sigma (𝜎) bond and pi (𝜋) bond between
the carbon and carbon and ethane has
very high boiling point
The product ‘C’ is
(b) Rotation around carbon-carbon bond in
(a) m-bromotoluene
ethane molecule is not possible,
(b) o-bromotoluene
because ethane molecule contains a
(c) p-bromotoluene
pi(𝜋) bond between the carbon and
(d) 3-bromo-2,4,6-trichlorotoluene
carbon and ethane has very high boiling
21. With respect to the conformers of ethane, point
which of the following statements is true? (c) Rotation around carbon-carbon bond in
(a) Both bond angles and bond length ethane molecule is not possible,
remains same because ethane molecule contains both
(b) Bond angle remains same but bond sigma (𝜎) bond and pi (𝜋) bond between
length changes the carbon and carbon
(c) Bond angle changes but bond length (d) Rotation around carbon-carbon bond in
remains same
ethane molecule is possible because of
(d) Both bond angle and bond length
cylindrical symmetry of sigma(𝜎) bond
change
between carbon-carbon atoms
22. Predict the correct intermediate and
product in the following reaction 25. The compound that will react most readily
(2017-Delhi) with gaseous bromine has the formula:
𝐻3 𝐶 − 𝐶 ≡ 𝐶𝐻 →
𝐻2 𝑂,𝐻2 𝑆𝑂4
𝑖𝑛𝑡𝑒𝑟𝑚𝑒𝑑𝑖𝑎𝑡𝑒 → 𝑝𝑟𝑜𝑑𝑢𝑐𝑡 (2016-II)
𝐻𝑔𝑆𝑂4
(a) C4H10
(A) (B)
(b) C2H4
(c) C3H6
(d) C2H2
(a)
26. The correct structure of the product A
formed in the reaction: (2016-II)
(b)

(c)

(d)
23. Which one is the correct order of acidity?
(2017-Delhi) (a)
(a) 𝐶𝐻3 − 𝐶𝐻3 > 𝐶𝐻2 = 𝐶𝐻2 > 𝐶𝐻3 − 𝐶 ≡
𝐶𝐻 > 𝐶𝐻 ≡ 𝐶𝐻
(b) 𝐶𝐻2 = 𝐶𝐻2 > 𝐶𝐻3 − 𝐶𝐻 = 𝐶𝐻2 > 𝐶𝐻3 −
𝐶 ≡ 𝐶𝐻 > 𝐶𝐻 ≡ 𝐶𝐻
(b)
(c) 𝐶𝐻 ≡ 𝐶𝐻 > 𝐶𝐻3 − 𝐶 ≡ 𝐶𝐻 > 𝐶𝐻2 = 𝐶𝐻2 >
𝐶𝐻3 − 𝐶𝐻3

113
(c) (b)

(d)
(c)
27. Which of the following compounds shall not
produce propene by reaction with HBr
followed by elimination or direct only
elimination reaction? (2016-II)
(a) H2C=C=O

(d)
31. In the reaction: (2016 - I)
(b)

(a) X=1-Butyne; Y = 2-Hexyne


(b) X=1-Butyne; Y = 3-Hexyne
(c) X=2-Butyne; Y = 3-Hexyne
(c) (d) X=2-Butyne; Y = 2-Hexyne
(d) H3 C– CH2 − CH2 OH
The pair of electron in the given carbanion,
28. Which of the following can be used as the ⊖
halide component for Friedel-Crafts 𝐶𝐻 ≡ 𝐶 , is present in which of the following
reaction? (2016-II) orbitals? (2016 - I)
(a) Chloroethene (a) sp
(b) Isopropyl chloride (b) d2sp
(c) Chlorobenzene (c) sp3
(d) Bromobenzene (d) sp2
29. In pyrole, the electron density is maximum 33. The correct statement regarding the
on (2016-II) comparison of staggered and eclipsed
conformations of ethane, is: (2016-I)
(a) The staggered conformation of ethane is
more stable than eclipsed conformation,
because staggered conformation has no
torsional strain.
(a) 2 and 3 (b) The staggered conformation of ethane is
(b) 3 and 4 less stable than eclipsed conformation,
(c) 2 and 4 because staggered conformation has
(d) 2 and 5 torsional strain.
(c) The eclipsed conformation of ethane is
30. In the given reaction, more stable than staggered
conformation, because eclipsed
conformation has not torsional strain.
(d) The eclipsed conformation of ethane is
the product P is: (2016-II)
more stable than staggered
conformation even though the eclipsed
conformation has torsional strain.
(a) 34. For the following reaction:
114
(A) 𝐶𝐻3 𝐶𝐻2 𝐶𝐻2 𝐵𝑟 + 𝐾𝑂𝐻 → 𝐶𝐻3 𝐶𝐻 = 𝐶𝐻2 +
𝐾𝐵𝑟 + 𝐻2 𝑂
(d)
38. A single compound of the structure
(B)

(C)
Which of the following statements is correct is obtainable from ozonolysis of which of the
(2016 - I) following cyclic compounds? (2015)
(a) (A) is substitution, (B) and (C) are
addition reactions.
(b) (A) and (B) are elimination reactions and
(a)
(C) is addition reaction.
(c) (A) is elimination, (B) is substitution and
(C) is addition reaction.
(d) (A) is elimination, (B) and (C) are (b)
substitution reactions.
35. The number of structural isomers possible
from the molecular formula C3H9N is: (c)
(2015 Re)
(a) 3
(b) 4
(c) 5 (d)
(d) 2 39. Given:
36. In the reaction with HCl, an alkene reacts
in accordance with the Markovnikov’s rule,
to give a product 1-chloro-1-
methylcyclohexane. The possible alkene is:
(2015 Re)
The enthalpy of hydrogenation of these
compounds will be in the order as: (2015)
(a) III > II > I
(b) II > III > I
(a)
(c) II > I > III
(d) I > II > III
40. Identity Z in the sequence of reactions
(b) (2014)
(c) (a) and (b)
(a) (CH3)2CH2 – O – CH2CH3
(b) CH3(CH2)4 – O – CH3
(c) CH3CH2 – CH(CH3) – O – CH2CH3
(d) (d) CH3 – (CH2)3 – O – CH2CH3

37. 2, 3-Dimethyl-2-butene can be prepared by 41. What products are formed when the
heating which of the following compounds following compound is treated with Br2 in
with a strong acid? (2015 Re) the presence of FeBr3? (2014)
(a) (CH3 )3 CCH = CH2
(b) (CH3 )2 C = CHCH2 CH3
(c) (CH3 )2 CH − CH2 CH = CH2

115
(a)

(b)

(c)

(d)
42. Which of the following organic compounds
has same hybridization as its combustion
product (CO2)? (2014)
(a) Ethane
(b) Ethyne
(c) Ethene
(d) Ethanol
43. Which of the following compounds will not
undergo Friedal-Craft reaction easily?
(2013)
(a) Cumene
(b) Xylene
(c) Nitrobenzene
(d) Toluene
44. The radical, is aromatic because it has:
(2013)
(a) 6 p-orbitals and 6 unpaired electrons
(b) 7 p-orbitals and 6 unaired electrons
(c) 7 p-orbitals and 7 unpaired electrons
(d) 6 p-orbitals and 7 unpaired electrons
45. Nitrobenzene on reaction with conc.
HNO3/H2SO4 at 80°–100°C forms which one
of the following products? (2013)
(a) 1,2-Dinitrobenzene
(b) 1,3-Dinitrobenzene
(c) 1,4-Dinitrobenzene
(d) 1,2,4-Trinitrobenzene

116
Answer Key
S1. Ans. (d) S25. Ans. (c)

S2. Ans. (d) S26. Ans. (d)

S3. Ans. (b) S27. Ans. (a)

S4. Ans. (a) S28. Ans. (b)

S5. Ans. (c) S29. Ans. (d)

S6. Ans. (d) S30. Ans. (d)

S7. Ans. (1) S31. Ans. (c)

S8. Ans. (d) S32. Ans. (b)

S9. Ans. (a) S33. Ans. (a)

S10. Ans. (c) S34. Ans. (c)

S11. Ans. (b) S35. Ans. (b)

S12. Ans. (b) S36. Ans. (c)

S13. Ans. (b) S37. Ans. (a)

S14. Ans. (c) S38. Ans. (d)

S15. Ans. (a) S39. Ans. (a)

S16. Ans. (b) S40. Ans. (d)

S17. Ans. (c) S41. Ans. (b)

S18. Ans. (b) S42. Ans. (b)

S19. Ans. (c) S43. Ans. (c)

S20. Ans. (a) S44. Ans. (a)

S21. Ans. (a) S45. Ans. (b)

S22. Ans. (b)

S23. Ans. (c)

S24. Ans. (d)

117
Solutions
S1. Ans.(d)

S2. Ans.(d)
S6. Ans.(d)
(1) From vicinal dihalides

(2) From alkynes

S3. Ans.(b)
(3) From alkyl halide
can not act as Lewis acid because they
does not contain vacant orbital
BF3 ⟶ Contains vacant orbital on central
atom (Boron).
S4. Ans.(a)
(4)
With increase in number of carbons in
alkane, boiling point increases.
In case of isomeric alkanes, greater in the
number of branches, lesser is the boiling
point. S7. Ans.(a)
Boiling point order: Heptane > Hexane > 2-
(a) (e)

methylbutane > butane > 2-methylpropane


(c) (b) (a)

S5. Ans.(c)

S8. Ans.(d)
Dihedral angle (D.A.) of least stable
conformer of ethane = 0°

118
S9. Ans.(a)

Dihedral angle (D.A.) of least stable


conformer of ethane = 0°
S13. Ans.(b)
In the presence of peroxide, addition of
Reaction of methane with Br2 in the
HBr to unsymmetrical alkenes take presence of light is a free radical
place by antimarkonikov’s rule/peroxide substitution reaction.

effect/kharash effect.

S10. Ans.(c)

Decarboxylation takes place by soda- S14. Ans.(c)


lime (NaOH + CaO). Greater is the electron density on
S11. Ans.(b) benzene ring, greater is the reactivity
Symmetrical alkane with even number towards electrophilic aromatic
of carbon atoms can be prepared by substitution.
Wurtz reaction. Electron withdrawing group like NO2,Cl
decreases the electron density on
benzene group.
If R1 and R2 are different, then mixture Phenol is more reactive than benzene
of alkanes may be obtained as towards electrophilic substitution
reaction. The donation of the oxygen’s
lone pair into the ring system increases
the electron density around the ring.
If R1 and R2 are different, then mixture of
That makes the ring much more reactive
alkanes may be obtained as than it is in benzene. The intermediate
R1 – R1 + R1 – R2 + R2 – R2 + 2NaX carbocation is more resonance
stabilized.
S12. Ans.(b)

119
S15. Ans.(a) S19. Ans.(c)
F
Number of 𝜎 bonds = 10
And number of 𝜋 bonds = 3
S16. Ans.(b)

S20. Ans.(a)
Lindar’s catalyst give cis product
S17. Ans.(c)

S18. Ans.(b)
S21. Ans.(a)
With respect to the conformers of
ethane, both bond length and bond
angles remain same.
Electron distribution of the sigma
molecular orbital is symmetrical around
the internuclear axis of C – C single
bond.

120
S22. Ans.(b)
H2SO4 = Provide acidic medium.
HgSO4 = Act as a solvent

This reaction will not give propene


Whereas cyclic propyne, 𝐶𝐻3 − 𝐶𝐻2 −
𝐶𝐻2 − 𝐵𝑟 & 𝐶𝐻3 − 𝐶𝐻2 − 𝐶𝐻2 − 𝑂𝐻 will
give propene by elimination reaction.
S28. Ans.(b)

S23. Ans.(c)
𝐶𝐻 ≡ 𝐶𝐻 > 𝐶𝐻3 − 𝐶 ≡ 𝐶𝐻 > 𝐶𝐻2 = 𝐶𝐻2 >
𝐶𝐻3 − 𝐶𝐻3
→ Alkanes, alkenes and alkynes follow
the following trend in their acidic In chlorobenzene, bromobenzene or
behaviour:- chloroethane lone pair of electron on
(i) 𝐻𝐶 ≡ 𝐶𝐻 > 𝐻2 𝐶 = 𝐶𝐻2 > 𝐶𝐻3 − 𝐶𝐻3 halogen atom are delocalized because of
(=) double bond character.
(ii) 𝐻𝐶 ≡ 𝐶𝐻 > 𝐶𝐻3 − 𝐶 ≡ 𝐶𝐻 ≫ 𝐶𝐻3 − 𝐶 ≡
𝐶 − 𝐶𝐻3 S29. Ans.(d)
S24. Ans.(d)
Rotation around carbon-carbon bond in
ethane molecule is possible because of
cylindrical symmetry of sigma (𝜎) bond
between carbon-carbon atoms as ethane
does not have 𝜋-bond.
S25. Ans.(c)
Gaseous bromine reacts with alkenes to
give allyl substituted alkyl halides via
free radical mechanism.

S26. Ans.(b)

It is a simple reduction reaction.


S27. Ans.(a)
S30. Ans.(c)

121
(ii) It is a substitution reaction because
Br group is replaced/substituted by–OK
group
(iii)

S31. Ans.(b)

This is an addition reaction as 2 Br


groups are getting added, conversion of
saturated compound to an unsaturated
one.
S35. Ans.(b)
C3H9N Total possible isomers.

S32. Ans.(a)
(−)
𝐶𝐻3 − 𝐶 ≡ 𝐶 carbanion – sp hybridized.
S33. Ans.(a)
The correct statement is that staggered
configuration of any molecule will
always be more stable than eclipsed
form because in staggered configuration S36. Ans.(c)
the atom attached are far apart from
each other & exhibits less strain also,
the bond pair electron suffers less
repulsion.

Markovnikov rule says that the halogen


group goes to C with less number of H,
S34. Ans.(c) in both the cases the same product will
(i) Here, 𝐶𝐻3 𝐶𝐻2 𝐶𝐻2 𝐵𝑟 + 𝐾𝑂𝐻 → be formed.
𝐶𝐻3 𝐶𝐻 = 𝐶𝐻2 + 𝐾𝐵𝑟 + 𝐻2 𝑂
Br is getting eliminated & a (𝜋) double
bond is getting created that is
conversion of saturated compound to
unsaturated one.

122
S37. Ans.(a) An antimarkonikov reaction in presence
of peroxide linkage:

Now for ether formation:

Now for ether formation:


S44. Ans.(a)

S38. Ans.(d)
Only benzene ring is aromatic with 6
unpaired & 6-p-orbitals verifying the
Huckel’s rule (4n + 2).

S45. Ans.(b)
S39. Ans.(a)
Enthalpy of hydrogenation is inversely
proportional to stability of alkenes.
Stability of alkenes:

Nitro group is a meta directing group

S40. Ans.(d)

123
Chapter 17
Haloalkanes and Haloarenes

1. Which amongst the following reactions of


alkyl halides produces isonitrile as a major
product? (a)
A. R – X + HCN ⟶
B. B – X + AgCN ⟶
C. R – X + KCN ⟶ (b)
H2 O
D. R – X + NaCN →
C2 H5 OH
Choose the most appropriate answer from (c)
the options given below: (2023)
(a) D only
(b) C and D only
(d)
(c) B only
(d) A and B only 5. The given compound
2. Choose the correct sequence of reagents in
the conversion of 4-nitrotoluene to 2-
bromotoluene. (2023)
(a) NaNO2/HCl; Sn/HCl; Br2;H2O/H3PO2
(b) Sn/HCl; NaNO2/HCl; Br2; H2O/H3PO2
(c) Br2; Sn/HCl; NaNO2/HCl; H2O/H3PO2 is an example of _______. (2023)
(d) Sn/HCl; Br2; NaNO2/HCl; H2O/H3PO2 (a) aryl halide
3. Identify the product in the following (b) allylic halide
reaction: (2023) (c) vinylic halide
(d) benzylic halide
6. The correct sequence of bond enthalpy of
‘C–X’ bond is: (2021)
(a) CH3 – F > CH3 – Cl > CH3 – Br > CH3 – I
(b) CH3 – F < CH3 – Cl > CH3 – Br > CH3 – I
(c) CH3 – Cl > CH3 – F > CH3 – Br > CH3 – I
(d) CH3 – F < CH3 – Cl < CH3 – Br < CH3 – I
(a)
7. The major product formed in
dehydrohalogenation reaction of 2-Bromo
pentane is Pent-2-ene. This product
(b)
formation is based on? (2021)
(a) Hund’s rule
(b) Hofmann rule
(c) (c) Huckel’s rule
(d) Saytzeff’s rule
8. Elimination reaction of 2-Bromo-pentane to
(d)
form pent-2-ene is (2020)
4. Identify 'X' in the following reaction. (1) 𝛽 -Elimination reaction
(2) Follows Zaitsev rule
(3) Dehydrohalogenation reaction
(4) Dehydration reaction
[1.0 mol] [1.0 mol] (2023)
(a) (1), (3), (4)

124
(b) (2), (3), (4) 14. Which of the following biphenyls is optically
(c) (1), (2), (4) active: (2016-I)

(d) (1), (2), (3)


9. Which of the following will NOT undergo SN1
reaction with OH − ? (2020 Covid Re-NEET)
(a)
(a) (CH3)3CCl

(b)
(b)

(c)

(c)
(d) CH2=CH – CH2Cl
10. An example of a sigma bonded (d)
organometallic compound is: (2017-Delhi)
(a) Cobaltocene 15. Two possible stereo-stereo-structures of
(b) Ruthenocene CH3CHOHCOOH, which are optically active,
(c) Grignard’s are called: (2015 Re)
(d) Ferrocene
(b) Diastereomers
11. Which of the following will react faster (c) Atropisomers
through SN1 mechanism? (2017-Gujarat) (d) Enantiomers
(a) CH3CH2Cl
(b) H2C = CH – CH2Cl 16. In a SN1 reaction on chiral centres, there is:
(2015 Re)
(a) 100% inversion
(b) 100% racemisation
(c) (c) Inversion more than retention leading to
(d) CH2 = CHCl partial racemisation
12. Of the following alcohols, the one that (d) 100% retention
would react fastest with conc. HCl and 17. Which of the following reaction (s) can be
anhydrous ZnCl2 is: (2017-Gujarat) used for the preparation of alkyl halides?
(a) 2-methylpropanol (2015 Re)
(b) Butan-1-ol 𝑎𝑛ℎ.𝑍𝑛𝐶𝑙2
(c) Butan-2-ol (I) CH3CH2OH + HCl →
(d) 2-methylpropan-2-ol (II) CH3CH2OH + HCl →
(III) (CH3)3 COH + HCL →
13. Consider the reaction 𝑎𝑛ℎ.𝑍𝑛𝐶𝑙2
(IV) (CH3)2 CHOH + HCl →
CH3CH2CH2Br+NaCN→
CH3CH2CH2CN+NaBr (a) (III) and (IV) only
(b) (I), (III) and (IV) only
This reaction will be the fastest in:
(c) (I) and (II) only
(2016-II) (d) (IV) only
(a) N,N’-dimethylformamide (DMF) 18. Which of the following compounds will
(b) Water undergo racemization when solution of
(c) Ethanol KOH hydrolyses? (2014)
(d) Methanol

125
(i)
(iv)
(ii)
(a) (ii) and (iv)
(b) (ii) and (iv)
(iii) (c) (i) and (iv)
(d) (i) and (ii)

126
Answer Key
S1. Ans. (c) S10. Ans. (c)

S2. Ans. (c) S11. Ans. (b)

S3. Ans. (b) S12. Ans. (d)

S4. Ans. (a) S13. Ans. (a)

S5. Ans. (b) S14. Ans. (c)

S6. Ans. (a) S15. Ans. (d)

S7. Ans. (d) S16. Ans. (c)

S8. Ans. (d) S17. Ans. (b)

S9. Ans. (b) S18. Ans. (c)

127
Solutions
S1. Ans. (c)
[B]R – X + Ag – C ≡ N ⟶ NC isonitrile
S2. Ans. (c)

S3. Ans. (b) (a) The process is known as 𝛽-


elimination as it involves elimination
of 𝛽-Hydrogen.
(b) Pent-2-ene is major product known
as Saytzeff’s product and it is more
stable alkene. Since more substituted
alkene is formed, it follows zaitsev’s
rule.
Since ‘H’ and ‘Br’ are removed, it is
dehydrohalogenation Thus
statements 1, 2 and 3 are correct.
S4. Ans. (a) So, option d is correct
S9. Ans.(b)
The rates of SN1 reactions decrease in the
order
Tertiary > secondary > primary > >
methyl.

S5. Ans. (b)

This compound contain Primary alkyl


S10. Ans.(c)
Allylic halide 𝑑𝑟𝑦 𝑒𝑡ℎ𝑒𝑟
𝑅 − 𝐵𝑟 + 𝑀𝑔 → 𝑅 − 𝑀𝑔𝐵𝑟
S6. Ans.(a)
Grignard reagent
CH3–F > CH3 – Cl > CH3 – Br > CH3–I
∴ It is a single bond organometallic
S7. Ans.(d) compound.
S8. Ans.(d) S11. Ans.(b)
The order of reaction in SN1 reaction is
3° > 2° > 1°. But in this case : H2C = CH
– CH2Cl.

128
⊕ S16. Ans.(c)
CH2 = CH − C H2 is highly stable due to
resonance. In case of optically active alkyl halides,
SN1 reaction is accompanied by
S12. Ans.(d)
racemization. The carbocation formed in
Reactivity of alcohol with Lucas reagent the slow step being sp2 hybridised is
3°R − OH > 2°R − OH > 1°R − OH. planar and attack of nucleophile may
take place from either side resulting in a
mixture of products, one having the
same configuration and other having
inverted configuration.
D is tertiary in nature. The isomer corresponding to inversion is
Therefore, they react fastly. present in slight excess because SN1 also
S13. Ans.(a) depends upon the degree of shielding of
the front side of the reacting carbon.
CH3CH2CH2Br + NaCN → CH3CH2CH2CN
+ NaBr S17. Ans.(b)
This is a substitution reaction & follows
SN2 pathway. Reagents which gives
fastest SN2 reaction are DMSO (dimethyl
sulphoxide) or DMF (N, N’ dimethyl for
amide), these are non polar solvents.
S14. Ans.(c)
The above 3 reaction will give alkyl
halides, the reaction of primary &
secondary alcohol requires the use of
catalyst (ZnCl ), but tertiary alcohol does
not need any catalyst because the
Ortho substituted biphyenyls are carbocation formed will be most stable.
optically active as both rings do not lie in S18. Ans.(c)
the same plane & their mirror images are
Racemisation is shown by those
non – super imposable.
compounds which show optical activity,
S15. Ans.(d) out of the above 4 options only
Optically active pairs are called
enantiomers.

129
Chapter 18
Alcohols, Phenols & Ethers

1. Consider the following reaction C. Zn dust, ∆ 3. Salicyl


aldehyde

D. Na2Cr2O7, H2SO4 4. Salicylic acid


Choose the correct answer from the options
Identify products A and B: (2023) given below: (2022)
(a) A-4, B-2, C-1, D-3
(b) A-3, B-4, C-1, D-2
(a) (c) A-2, B-1, C-4, D-3
(d) A-4, B-3, C-2, D-1
4. Which one of the following reaction sequence
(b)
is incorrect method to prepare phenol?
(2022)

(c)

(a)
(b) Aniline, NaNO2 + HCl, H2O, heating
(d)
2. Which amongst the following will be most
readily dehydrate under acidic conditions?
(2023)
(d)
5. Given below are two statements:
(a) Statement I: The acidic strength of
monosubstituted nitrophenol is higher than
phenol because of electron withdrawing
nitro group.
Statement II: o-nitrophenol, m-nitrophenol
(b) and p-nitrophenol will have same acidic
strength as they have one nitro group
attached to the phenolic ring.
In the light of the above statements, choose
(c) the most appropriate answer from the option
given below: (2022)
(a) Both Statement I and Statement II are
(d) correct.
(b) Both Statement I and Statement II are
3. Match the reagents (List I) with the product
incorrect.
(List II) obtained from phenol.
(c) Statement I is correct but Statement II
List I List II
is incorrect.
A. (i) NaOH (ii) CO2 1. Benzoquinone (d) Statement I is incorrect but Statement II
( iii) H+ is correct.
B. (i) Aqueous 2. Benzene 6. Given below are two statements:
NaOH + CHCl3 Statement I: In Lucas test, primary,
(ii) H+ secondary and tertiary alcohols are
distinguished on the basis of their reactivity
130
with conc. HCl + ZnCl2, known as Lucas
Reagent.
Statement II: Primary alcohols are most
reactive and immediately produce turbidity
at room temperature on reaction with Lucas
(d)
Reagent.
In the light of the above statements, choose 9. Reaction between acetone and methyl
the most appropriate answer from the option magnesium chloride followed by hydrolysis
given below: (2022) will give: (2020)
(a) Both Statement I and Statement II are (a) Sec. butyl alcohol
correct. (b) Tert. Butyl alcohol
(b) Both Statement I and Statement II are (c) Isobutyl alcohol
incorrect. (d) Isopropyl alcohol
(c) Statement I is correct but Statement II 10. Anisole on cleavage with HI gives: (2020)
is incorrect.
(d) Statement I is incorrect but Statement II
is correct.
7. What is the IUPAC name of the organic
compound formed in the following chemical
reaction? (2021) (a)

(a) Pentan-2-ol
(b) Pentan-3-ol
(c) 2-methyl butan-2-ol
(d) 2-methyl propan-2-ol
8. The product formed in the following
chemical reaction is: (2021)

(c)

(d)
11. Which of the following substituted phenols
(a) is the strongest acid?
(2020 Covid Re-NEET)

(b)

(a)

(c)

(b)

131
(c) (a)

(d) (b)

12.

What is Z? (2020 Covid Re-NEET)

(c)
(a)

(b)

(c)

(d) (d)
13. The compound that is most difficult to 15. Identify the major products P, Q and R in
protonate is: (2019) the following sequence of reactions: (2018)

(a)

(b)

(c)

(d) (a)
14. The structure of intermediate A in the
following reaction, is (2019)

(b)

(c)

`132
(d)
16. The compound A on treatment with Na gives
B, and with PCl5 gives C. B and C react
(d)
together to give diethyl ether. A, B and C are
in the order: (2018) 20. The oxidation of phenol with chromic acid
(a) C2H5OH, C2H6, C2H5Cl gives (2017-Gujarat)
(b) C2H5OH, C2H5Cl, C2H5ONa (a) An aldehyde
(c) C2H5OH, C2H5Ona, C2H5Cl (b) A simple diketone
(d) C2H5Cl, C2H6, C2H5OH (c) A conjugated diketone
(d) Ortho benzoquinone
17. In the reaction, the electrophile involved is:
(2018) 21. Identify A and predict the type of reaction:
(2017-Delhi)


(a) Dichloromethyl cation (C HCl2 )

(b) Formyl cation (C HO) (a)
(c) Dichlorocarbene (:CCl2)

(d) Dichloromethyl anion (C HCl2)
18. The heating of phenyl-methyl ethers with HI
produces. (2017-Delhi) (b)
(a) Benzene
(b) Ethyl chlorides
(c) Iodobenzene
(d) Phenol (c)

19. Which one is the most acidic compound?


(2017-Delhi)
(d)
22. The reaction:

(a) can be classified as? (2016-I)


(a) Williamson alcohol synthesis reaction
(b) Williamson ether synthesis reaction
(c) Alcohol formation reaction
(d) Dehydration reaction
23. Reaction of phenol with chloroform in
(b) presence of dilute sodium hydroxide finally
introduces which one of the following
functional group? (2015 Re)
(a) –CHO
(c) (b) –CH2Cl
(c) –COOH
(d) –CHCl2

133
24. The reaction
(c)
(d)
27. Which of the following will not be soluble in
sodium hydrogen carbonate? (2014)
(a) Benzoic acid
(b) o-Nitrophenol
(c) Benzenesulphonic acid
is called: (2015) (d) 2,4,6-trinitrophenol

(a) Williamson continuous esterification 28. Among the following ethers, which one will
process produce methyl alcohol on treatment with
(b) Etard reaction hot concentrated HI? (2013)
(c) Gatterman – Koch reaction (a)
(d) Williamson synthesis
25. Among the following sets of reactants which
one produces anisole? (2014) (b)
(a) C6H5OH; NaOH; CH3I
(b) C6H5OH; neutral FeCl3
(c) C6H5 – CH3; CH3COCl; AlCl3
(d) CH3CHO; RMgX (c)
26. Identify Z in the sequence of reactions:

(a)
(b)

134
Answer Key
S1. Ans. (b) S15. Ans. (c)

S2. Ans. (a) S16. Ans. (c)

S3. Ans. (d) S17. Ans. (c)

S4. Ans. (d) S18. Ans. (d)

S5. Ans. (c) S19. Ans. (a)

S6. Ans. (c) S20. Ans. (c)

S7. Ans. (c) S21. Ans. (b)

S8. Ans. (c) S22. Ans. (b)

S9. Ans. (b) S23. Ans. (a)

S10. Ans. (d) S24. Ans. (d)

S11. Ans. (d) S25. Ans. (a)

S12. Ans. (d) S26. Ans. (d)

S13. Ans. (d) S27. Ans. (c)

S14. Ans. (b) S28. Ans. (b)

135
Solutions
S1. Ans. (b)

S2. Ans. (a)


Due to presence of conjugation in
product.

S3. Ans. (d) At STP condition substitution at sp2


carbon atom is not feasible.
S5. Ans. (c)

• Nitro group has electron withdrawing


tendency. It can withdraw electrons
both by −I effect and −R effect. Thus
the acidic strength of monosubstituted
nitrophenol is higher than phenol.
• Nitro group present at o- and p-
positions will have strong −R effect
while nitro group present at m-
position will influence only −I effect
hence acidity or o/p isomer will be
more meta isomer.
S6. Ans. (c)
Primary, secondary and tertiary alcohols
can be differentiated by their reaction
with (HCl + anhy ZnCl2) Lucas reagent.
ZnCl2 +HCl
• 3° alcohol → Immediate
turbidity at room temperature.
ZnCl2 +HCl
• 2° alcohol → Turbidity after 5
S4. Ans. (d) minutes at room temperature.
ZnCl2 +HCl
• 1° alcohol → Do not gives
turbidity at room temperature.

S7. Ans.(c)

136
Anisole on cleavage with HI gives phenol
and methyliodide

S11. Ans.(d)
Acidic strength of phenol is enhanced by
the presence of electron- withdrawing
groups which stabilized the phenoxide
ion.
While presence of electron- donating
group like CH3,C2H5 destabilizes the
S8. Ans.(c) phenoxide ion, thus decreasing the
acidic strength.
Due to the presence of strong electron-
withdrawing group–NO2(–I, –R), p-nitro
phenol is the strongest acid.
S12. Ans.(d)
The given reaction is hydroboration–
oxidation reaction. It is a two-step
hydration reaction that converts an
alkene into an alcohol.

NaBH4 reduces aldehyde/ketone but


does not reduce ester.
S9. Ans.(b)

Butan-1-ol
S10. Ans.(d) Thus, Z is CH3CH2CH2CH2OH

137
S13. Ans.(d)
Due to involvement of lone pair of
electrons present on O-atom is in
resonance in phenol. So, it will have
positive charge (partial), hence incoming
proton will not be able to attack easily.
S14. Ans.(b)

S17. Ans.(c)
It is Reimer-Tiemann reaction the
electrophile formed is : CCl2
(Dichlorocarbene) according to the
following reaction

S15. Ans.(c)

In phenyl methyl ether C-O bond is sp2


hybridised. In sp and sp2 hybridisation
(Clarge – O – Csmall) cleavage takes place
but in sp3 hybridisation (Clarge – O –
Csmall) cleavage takes place.
S19. Ans.(a)

Now,

Is a most acidic compound. –NO2 is an


electron withdrawing group. In this
compound, NO2 is attached to the ortho
and para position which increases the
acidic strength of phenol.
S20. Ans.(c)
Phenol + chromic acid = A conjugate
diketone
S16. Ans.(c)

(conjugate diketone)
138
S21. Ans.(b)

S22. Ans.(b)

S26. Ans.(d)
In anti Markonikov reaction (presence of
peroxide linkage):
CH3CH2CH=CH2+HBr/H2O2 → CH3CH2–
Williamson ether synthesis CH2Br
S23. Ans.(a) For ether formation:
C2H5ONa → CH3CH2–CH2–CH2–O–C2H5 +
NaBr
Williamson synthesis

The above reaction is Riemer – Tiemann S27. Ans.(b)


reaction which introduces a –CHO Sodium hydrogen carbonate (NaHCO3)
(aldehyde) group to phenol. being an alkali solution will dissolve an
S24. Ans.(d) acid not a base so o-nitrophenol being a
very weak acid than NaHCO3 will not get
The above reaction is called Williamson dissolve.
synthesis of ether.
S28. Ans.(c)

As 3° carbocation formed is stabilized


because of 9 hyperconjugated H’s and 3
S25. Ans.(a) inductive effect inducing –CH3.

139
Chapter 19
Aldehydes, Ketones and Carboxylic Acids

1. Consider the given reaction:


dil. Ba(OH)3
CH3COCH3 → "X"
The functional groups present in compound
"X" are: (2023)
(a) ketone and double bond (2023)
(b) double bond and aldehyde
(c) alcohol and aldehyde
(d) alcohol and ketone
(a)
2.

(b)
Identify 'X' in above reactions (2023)
(a) B2H6
(b) LiAlH4
(c) NaBH4
(d) H2/Pd
3. The following conversion is known as:

(d)
6. Complete the following reaction:

(2023)
(a) Stephen reaction
(b) Gattermann-Koch reaction
(c) Etard reaction [C] is: (2023)
(d) Rsenmund reaction
4. Reagents which can be used to convert
alcohols to carboxylic acids, are
(A) CrO3 – H2SO4 (a)
(B) K2Cr2O7 + H2SO4
(C) KMnO4 + KOH/H3O+
(D) Cu, 573 K
(b)
(E) CrO3, (CH3CO)2O
Choose the most appropriate answer from
the options given below: (2023)
(a) B, C and D only
(c)
(b) B, D and E only
(c) A, B and C only
(d) A, B and E only
5. The major product formed in the following (d)
conversion is _______.

140
7. Identify the major product obtained in the
following reaction:

(d)
9. Which of the following reactions is not an
example for nucleophilic addition-
elimination reaction? (2022)
(a) CH3CHO + NH3 ⇌ CH3CH = NH + H2O
OH
|
(2023) (b) CH3CHO + NaHSO3 ⇌ CH3 – C − OSO2Na
|
H
(c) CH3CHO + NH2OH ⇌ CH3CH = N – OH +
H 2O
(d) CH3CHO + C6H5NHNH2 ⇌ CH3CH = N –
(a) NHC6H5 + H2O
10. The product formed from the following
reaction sequence is

(b)

(2022)

(c)
(a)

(b)
(d)
8. Identify the final product [D] obtained in the
following sequence of reactions. (c)

(d)
11. Match List I with List II:
List I List II
(Reaction) (Product formed)
(2023)
A. Gabriel 1. Benzaldehyde
synthesis
B. Kolbe synthesis 2. Ethers
(a) Williamson 3. Primary
C.
(b) C4H10 synthesis amines
(c) HC ≡ C ⊝ Na+
D. Etard reaction 4. Salicylic acid

141
Choose the correct answer from the options (d) (RCOO)2Mg
given below: (2022) 15. Match List I with List II.
(a) A-3, B-4, C-2, D-1
List I List II
(b) A-3, B-1, C-2, D-4
(Product formed) (Reaction of carbonyl
(c) A-2, B-3, C-1, D-4
compound with)
(d) A-4, B-3, C-1, D-2
A. Cyanohydrin 1. NH2OH
12. The incorrect method to synthesize
benzaldehyde is: (2022) B. Acetal 2. RNH2

C. Schiff's base 3. Alcohol

D. Oxime 4. HCN
(a) Choose the correct answer from the options
given below. (2022)
(a) A-3, B-4, C-2, D-1
(b) A-2, B-3, C-4, D-1
(c) A-1, B-3, C-2, D-4
(b)
(d) A-4, B-3, C-2, D-1
16. Compound X on reaction with O3 followed
by Zn/H2O gives formaldehyde and 2-
methyl propanal as products. The
(c)
compound X is (2022)
(a) 3-Methylbut-1-ene

(d)
13. Given below are two statements: (d) Pent-2-ene
Statement I: The boiling points of 17. Which one of the following is not formed
aldehydes and ketones are higher than when acetone reacts with 2-pentanone in
hydrocarbons of comparable molecular the presence of dilute NaOH followed by
masses because of weak molecular heating? (2022)
association in aldehydes and ketones due
to dipole-dipole interactions.
Statement II: The boiling point of
aldehydes and ketones are lower than the (a)
alcohols of similar molecular masses due to
the absence of H-bonding.
In the light of the above statements, choose
the most appropriate answer from the given
below. (2022)
(b)
(a) Both Statement I and Statement II are
correct.
(b) Both Statement I and Statement II are
incorrect. (c)
(c) Statement I is correct but Statement II
is incorrect.
(d) Statement I is incorrect but Statement II
is correct.
dry H3 O+
14. RMgX + CO2 → Y→ RCOOH (d)
ether
What is Y in the above reaction? (2022)
18.
(a) RCOO− Mg+X Consider the above reaction and identify
(b) R3CO− Mg+X the missing reagent/chemical. (2021)
(c) RCOO− X+
142
(a) Red Phosphorus
(b) CaO
(c) DIBAL-H
(d) B2H6
19. Match List-I with List-II.
(b)

(c)

(d)
22. Which of the following acid will form an (i)
Choose the correct answer from the options Anhydride on heating and (ii) Acid imide on
given below. (2021) strong heating with ammonia?
(a) A-iii B-ii C-i D-iv (2020 Covid Re-NEET)
(b) A-i B-iv C-iii D-ii
(c) A-ii B-iii C-iv D-i
(d) A-iv B-I C-ii D-iii
20. Reaction between benzaldehyde and
acetophenone in presence of dilute NaOH is (a)
known as: (2020)
(a) Cannizzaro’s reaction
(b) Cross Cannizzaro’s reaction
(c) Cross Aldol condensation
(d) Aldol condensation
21. Identify compound X in the following (b)
sequence of reactions: (2020)

(c)

(d)
23. Identify compound (A) in the following
(a) reaction (2020 Covid Re-NEET)

143
(a) Toluene
(b) Acetophenone
(c) Benzoic acid (a)
(d) Benzoyl chloride
24. The major product of the following reaction
is: (2019) (b)

(c)

(d)
27. Of the following which is the product formed
(a) when cyclohexanone undergoes aldol
condensation followed by heating?
(2017-Delhi)

(b)

(b)
(c)

(c)

(d)
25. Carboxylic acids have higher boiling points
than aldehydes, ketones and even alcohols (d)
of comparable molecular mass. It is due to
their: (2018) 28. Consider the reactions: (2017-Delhi)
(a) Formation of intramolecular H-bonding
(b) Formation of carboxylate ion
(c) Formation of intermolecular H-bonding
(d) More extensive association of carboxylic
acid via van der Waals force of attraction
26. Compound A, C8H10O, is found to react with
NaOI (produced by reacting Y with NaOH) Identify A, X, Y and Z
and yields a yellow precipitate with (a) A-Ethanol, X-Acetaldehyde, Y-
characteristic smell. A and Y are Butanone, Z-Hydrazone
respectively: (2018) (b) A-Methoxymethane, X-Ethanoic acid, Y-
Acetate ion, Z-hydrazine

144
(c) A-Methoxymethane, X-Ethanol, Y- equilibrates with its corresponding enol
Ethanoic acid, Z-Semicarbazide and this process is known as
(d) A-Ethanal, X-Ethanol, Y-But-2-enal, Z- carbonylation
Semicarbazone (b) A carbonyl compound with a hydrogen
atom on its alpha-carbon rapidly
29. Consider the following sequence of equilibrates with its corresponding enol
reactions: and this process is known as keto-enol
tautomerism
(c) A carbonyl compound with a hydrogen
atom on its alpha-carbon never
equilibrates with its corresponding enol
(d) A carbonyl compound with a hydrogen
The substance ‘B’ is: (2017-Gujarat) atom on its alpha-carbon rapidly
(a) Acetone equilibrates with its corresponding enol
(b) Benzene and this process is known as aldehyde-
(c) Acetophenone ketone equilibration
(d) Benzaldehyde 35. The correct order of strengths of the
30. If the rate of the reaction: (2017-Gujarat) carboxylic Acids is: (2016-II)

is fastest, then Z is:


(a) OCOCH3
(b) Cl
(c) NH2 (c) I > II > III
(d) OC2H5 (d) II > III > I
31. Among the following acids, the strongest 36. Reaction of carbonyl compound with one of
acid is: (2017-Gujarat) the following reagents involves nucleophilic
(a) Cl3CCOOH addition followed by elimination of water.
(b) NCCH2COOH The reagent is: (2015 Re)
(c) O2NCH2COOH (a) Sodium hydrogen sulphite
(d) F3CCOOH (b) A Grignard reagent
(c) Hydrazine in presence of feebly acidic
32. The product formed by the reaction of an
solution
aldehyde with a primary amine is: (2016-I)
(d) Hydrocyanic acid
(a) Aromatic acid
37. An organic compound ‘X’ having molecular
(b) Schiff base
formula C5H10O yields phenyl hydrazone
(c) Ketone
and gives negative response to the Iodoform
(d) Carboxylic acid
test and Tollen’s test. It produces n-pentane
33. Which of the following reagents would on reduction. ‘X’ could be: (2015)
distinguish ciscyclopenta-1, 2-diol from the (a) 2-pentanone
trans-isomer? (2016-I) (b) 3-pentanone
(a) MnO2 (c) n-amyl alcohol
(b) Aluminium isopropoxide (d) Pentanal
(c) Acetone 38. The oxidation of benzene by V2O5 in the
(d) Ozone presence of air produces: (2015 Re)
(a) Maleic anhydride
34. The correct statement regarding a carbonyl
(b) Benzoic acid
compound with a hydrogen atom on its
(c) Benzaldehyde
alpha-carbon, is: (2016-I)
(d) Benzoic anhydride
(a) A carbonyl compound with a hydrogen
atom on its alpha-carbon rapidly

145
39. Which one of the following esters gets
hydrolysed most easily under alkaline
conditions? (2015 Re) (b)

(c)
(a)

(d)
41. The order of stability of the following
tautomeric compounds is: (2013)
(b)

(c)

(d)
40. Reaction by which Benzaldehyde cannot be
prepared: (2013)

(a) (b) II > III > I


(c) I > II > III
(d) III > II > I

146
Answer Key
S1. Ans. (d) S22. Ans. (d)

S2. Ans. (a) S23. Ans. (d)

S3. Ans. (d) S24. Ans. (b)

S4. Ans. (c) S25. Ans. (c)

S5. Ans. (a) S26. Ans. (d)

S6. Ans. (c) S27. Ans. (c)

S7. Ans. (b) S28. Ans. (d)

S8. Ans. (d) S29. Ans. (c)

S9. Ans. (b) S30. Ans. (b)

S10. Ans. (c) S31. Ans. (d)

S11. Ans. (a) S32. Ans. (b)

S12. Ans. (a) S33. Ans. (c)

S13. Ans. (a) S34. Ans. (b)

S14. Ans. (a) S35. Ans. (d)

S15. Ans. (d) S36. Ans. (c)

S16. Ans. (a) S37. Ans. (b)

S17. Ans. (b) S38. Ans. (a)

S18. Ans. (b) S39. Ans. (d)

S19. Ans. (c) S40. Ans. (d)

S20. Ans. (c) S41. Ans. (d)

S21. Ans. (b)

147
Solutions
S1. Ans. (d)

Functional groups present in product


are alcohol and ketone. S8. Ans. (d)
S2. Ans. (a)

S3. Ans. (d)


Resenmund reaction
S9. Ans. (b)
It is an example of nucleophilic addition
reaction

S4. Ans. (c)

S5. Ans. (a)


S10. Ans. (c)

S11. Ans. (a)


• Gabriel phthalimide synthesis is used
S6. Ans. (c) for preparation of aliphatic primary
amines.
• Kolbe synthesis with phenol gives
salicylic acid.
• Williamson synthesis gives ether on
reaction of alkyl halide and alcoxide.
• Etard reaction gives benzaldehyde
from benzene.
S12. Ans. (a)
S7. Ans. (b)

148
S13. Ans. (a)
S16. Ans. (a)
• The boiling points of aldehydes and
ketones are higher than hydrocarbons
of comparable molecular masses due
to weak molecular association in
aldehydes and ketones arising out of
the dipole-dipole interaction.
• Alcohol involved intermolecular
hydrogen bonding, because of which The given reaction is the reductive
the boiling point of aldehydes and ozonolysis of an alkene. The alkene will
ketones are lower than the alcohols of be
similar molecular masses.
S14. Ans. (a)

S17. Ans. (b)


Cross Aldol condensation reaction:
Both reactants contain 𝛼-Hydrogens, so
multiple products are possible.
S18. Ans.(b)

Here Y is RCOO− Mg+Y


S15. Ans. (d)
A-4, B-3, C-2, D-1
Decarboxylation takes place by Soda-
lime (NaOH + CaO)
S19. Ans.(c)

149
S20. Ans.(c)
Reaction between benzaldehyde and S23. Ans.(d)
acetophenone in presence of dilute NaOH
is known as Cross Aldol condensation

The given reaction is Rosenmund


reaction.
So ‘A’ is benzoyl chloride
S24. Ans.(b)

S21. Ans.(b)

S22. Ans.(d)
S25. Ans.(c)
Due to formation of intermolecular H-
bonding in carboxylic acid, association
occurs. Hence boiling point increases
and become more than the boiling point
of aldehydes, ketones and alcohols of
comparable molecular masses.

150
S26. Ans.(d) ∴ It is a substitution reaction.
Option (d) is secondary alcohol which on S29. Ans.(c)
oxidation gives phenylmethyl ketone
(Acetophenone). This on reaction with I2
and NaOH form iodoform and sodium
benzoate.

S27. Ans.(c)
S30. Ans.(b)
Because Cl− is a better leaving group
Reactivity order during nucleophilic acyl
substitution.

S31. Ans.(d)
–CF3 is the strongest withdrawing group.
Therefore; F3CCOOH is the strongest
acid, among the given acid
S32. Ans.(b)

S28. Ans.(d)

S33. Ans.(c)
Trans isomer does not react with
acetone.

151
S34. Ans.(b) S37. Ans.(b)
If negative response to iodoform test is
observed,

Then group is missing, among


4 options.

S35. Ans.(d)
R – O – R bond (ether) exhibits – I effect 3 – Pentanone will not give iodoform test.
and order for increasing acidic character S38. Ans.(a)
goes as:

S36. Ans.(c)

S39. Ans.(d)
Reactivity of nucleophilic substitution
reaction ∝ Electron withdrawing Group.
And – NO2 is a strong electron
withdrawing group
S40. Ans.(d)

Options A, B & C are Etard’s reaction, S41. Ans.(d)


Rosenmund reaction, Gatterman-Koch
reaction and forms benzaldehyde.

Clemmensen reduction, will not form


benzaldehyde.

Stable due to intermolecular H –


Bonding.

152
Chapter 20
Amines

1. Identify the product in the following


reaction: (2023)

(a)

(a)
(b)

(b)

(c)

(d)
4. Which of the following sequence of reactions
is suitable to synthesize chlorobenzene?
(2022)
(d)
(a) Benzene, Cl2, anhydrous FeCl3
2. Which of the following reaction will NOT give (b) Phenol, NaNO2, HCl, CuCl
primary amine as the product? (2023)
(i) LiAlH4
(a) CH3CN → Product
(ii) H3 O⊕
(i) LiAlH4
(b) CH3NC → Product (c)
(ii) H3 O⊕
(i) LiAlH4
(c) CH3CONH2 → Product
(ii) H3 O⊕
Br2 /KOH
(d) CH3CONH3 → Procuct
3. The product formed from the following
(d)
reaction sequence is: (2022)
5. Given below are two statements.
Statement I: Primary aliphatic amines
react with HNO2 to give unstable diazonium
salts.
Statement II: Primary aromatic amines
react with HNO2 to form diazonium salts
which are stable even above 300 K.
153
In the light of the above statements, choose
the most appropriate answer from the
(d)
options given below. (2022)
(a) Both Statement I and Statement II are 8. The reagent ‘R’ in the given sequence of
correct. chemical reaction is: (2021)
(b) Both Statement I and Statement II are
incorrect.
(c) Statement I is correct but Statement II
is incorrect.
(d) Statement I is incorrect but Statement II
is correct.
6. The product formed from the following
reaction sequence is (2022)

(a) CH3CH2OH
(b) HI
(c) CuCN/KCN
(d) H 2O
9. Which of the following amine will give the
carbylamine test? (2020)
(a)

(a)
(b)

(b)
(c)

(c)
(d)
7. Identify the compound that will react with
Hinsberg’s reagent to give a solid which
dissolves in alkali. (2021)
(d)
10. Reaction of propenamide with ethanolic
(a) sodium hydroxide and bromine will give
(2020 Covid Re-NEET)
(b) (a) Methylamine
(b) Propylamine
(c) Aniline
(d) Ethylamine
(c)

154
11. The correct order of the basic strength of
methyl substituted amines in aqueous
solution is: (2019)
(a) (CH3)2NH > CH3NH2>(CH3)3N (a)
(b) (CH3)3N>CH3NH2>(CH3)2NH
(c) (CH3)3N>(CH3)2NH>CH3NH2
(d) CH3NH2>(CH3)2NH>(CH3)3N
12. Nitration of aniline in strong acidic medium (b)
also gives m-nitroaniline because: (2018)
(a) Inspite of substituents nitro group
always goes to only m-position.
(b) In electrophilic substitution reactions,
(c)
amino group is meta directive.
(c) In acidic (strong) medium, aniline is
present as anilinium ion.
(d) Inabsence of substituents nitro group
(d)
always goes tom-position.
17. A given nitrogen-containing aromatic
13. The correct increasing order of basic compound A reacts with Sn/HCl, followed
strength for the following compounds is: by HNO2 to give an unstable compound B.
(2017-Delhi) B, on treatment with phenol, forms a
beautiful coloured compound C with the
molecular formula C12H10N2O. The
structure of compound A is: (2016-II)

(a)
(a) II < I < III
(b) II < III < I
(c) III < I < II (b)
(d) III < II < I
14. Which of the following reactions is
appropriate for converting acetamide to (c)
methanamine? (2017-Gujarat)
(a) Gabriel phthalimide synthesis
(b) Carbylamine reaction (d)
(c) Hoffman bromamide reaction 18. Which one of the following nitro-compounds
(d) Stephens reaction does not react with nitrous acid? (2016-II)
Cu/HCl
15. The reaction: ArN2Cl → ArCl + N2 is
known as: (2017-Gujarat)
(a) Balz Schiemann reaction (a)
(b) Sandmeyers reaction
(c) Finkelstein reaction
(d) Gattermann reaction
16. The product (P) of the following reaction (b)
(2017-Gujarat)

(c)

(d)
155
19. The correct statement regarding the basicity 23. In the following reaction, the product (A) is:
of arylamines is: (2016-I) (2014)
(a) Arylamines are generally more basic
than alkylamines, because the nitrogen
atom in arylamines is sp-hybridised.
(b) Arylamines are generally less basic than
alkylamines because the nitrogen lone
pair electrons are delocalized by
interaction with the aromatic ring 𝜋
electrons system.
(c) Arylamines are generally more basic (a)
than alkylamines because the nitrogen
lone pair electrons are not delocalised
by interaction with the aromatic ring 𝜋 (b)
electron system.
(d) Arylamines are generally more basic
than alkylamines because of aryl group (c)

20. The following reaction


(d)
24. Which of the following will be most stable
diazonium salt RN2 + X −? (2014)
(a) C6H5N2 + X −

(d) CH N X
is known by the name: (2015 Re)
(a) Schotten-Baumen reaction 25. In the reaction: (2013)
(b) Friedel-Craft’s reaction
(c) Perkin’s reaction
(d) Acetylation reaction
21. Method by which Aniline cannot be
prepared is: (2015 Re)
(a) Potassium salt of phthalimide treated
(a) HgSO4/H2SO4
with chlorobenzene followed by
(b) Cu2Cl2
hydrolysis with aqueous NaOH solution
(c) H3PO2 and H2O
(b) Hydrolysis of phenylisocyanide with
(d) H+/H2O
acidic solution
(c) Degradation of benzamide with bromine
in alkaline solution
(d) Reduction of nitrobenzene with H2/Pd
in ethanol
22. The electrolytic reduction of nitrobenzene in
strongly acidic medium produces (2015)
(a) Azobenzene
(b) Aniline
(c) p-aminophenol
(d) Azoxybenzene

156
Answer Key
S1. Ans. (a) S15. Ans. (d)

S2. Ans. (b) S16. Ans. (d)

S3. Ans. (b) S17. Ans. (d)

S4. Ans. (a) S18. Ans. (a)

S5. Ans. (c) S19. Ans. (b)

S6. Ans. (d) S20. Ans. (a)

S7. Ans. (b) S21. Ans. (a)

S8. Ans. (a) S22. Ans. (c)

S9. Ans. (d) S23. Ans. (c)

S10. Ans. (d) S24. Ans. (a)

S11. Ans. (a) S25. Ans. (c)

S12. Ans. (c)

S13. Ans. (a)

S14. Ans. (c)

157
Solutions
S1. Ans.(a) S5. Ans.(c)
Primary aliphatic amines react with
HNO2 and give unstable diazonium salt
which turns into alcohol
H2 O
R–NH2 + HNO2 ⟶ [R–N2+ –Cl− ] → ROH
+ N2 + HCl
Primary aromatic amines reacts with
HNO2 and give stable diazonium salt
which are stable at 273 to 278 K.
273−278 K
C6H5 – NH2 + HNO2 → C6H5N2+ Cl−
S6. Ans.(d)

S2. Ans.(b)
(i) LiAlH4
(1) CH3–CN → CH3–CH2–NH2 1°
(ii) H3O⊕
Amine
(i) LiAlH4
(2) CH3NC → CH3–NH–CH3 2°
Amine
O
|| (i) LiAlH4 S7. Ans.(b)
(3) CH3–C–NH2 → CH3–CH2–NH2 1°
(ii) H3 O⊕ 1° amines react with Hingberg’s reagent
Amine to give a solid, which dissolve in alkali.
O
|| Br2 +OH−
(4) CH3–C–NH2 → CH3–NH2 1°
Amine
S3. Ans.(b)

S8. Ans.(a)

S4. Ans.(a)

R − CH3 − CH2 OH
Benzene reacts with chlorine in presence
Certain mild reducing agents like
of anhydrous FeCl3 to give hypophosphorus acid or ethanol reduce
chlorobenzene. diazonium salts to arene and
themselves get oxidized to phosphorous
acid and ethanal respectively.
158
S9. Ans.(d) formed contains one carbon less than
S10. Ans.(d) that present in the amide.
S11. Ans.(a) S15. Ans.(d)
In aqueous solution, electron donating S16. Ans.(d)
inductive effect, solvation effect (H-
bonding) and steric hindrance all
together will affect basic strength of
substituted amines
Basic character will be
S17. Ans.(d)
Compound A must be –NO2 containing
compound:
S12. Ans.(c)


−NH3 is m-directing, hence besides para S18. Ans.(a)
(51%) and ortho (2%), meta product 3° - NO compounds do not react with
(47%) is also formed in significant yield. HNO2 (nitrous acid) because of absence
S13. Ans.(a) of 𝛼-H.

S19. Ans.(b)
Is more basic as compared to other two Arylamines are less basic than
because CH3 is a donating group which alkylamines as N on the benzene ring is
increases the basic strength of the in conjugation. Where its lone pair of
group. electron are delocalized and partial-𝜋-
bond character is present thus making
S14. Ans.(c)
it unavailable for incoming electrophile,
Hoffmann bromamide reaction whereas in alkylamines such a process
is not present and also –I effect
increases the basic character on N.
S20. Ans.(a)

Hoffmann developed a method for


preparation of primary amines by
treating an amide with bromine in an
aqueous or ethanoic solution, of sodium
hydroxide. In this degradation reaction, Synthesis of amides from amines from
migration of an alkyl or aryl group takes acyl halides or anhydrides in the
place from carbonyl carbon of the amide presence of a base is known as
to the nitrogen atom. The amide so Schotten-Baumen reaction.

159
S21. Ans.(a) S23. Ans.(c)

will not undergo nucleophilic


S24. Ans.(a)
substitution under normal conditions.
Diazonium salt is most stabilized as the
S22. Ans.(c)
(–) charge on halogen is resonance
stabilized.
In the rest options, stabilization is only
because of +R & +I effect.
S25. Ans.(c)

For removal of diazonium (ion) H3PO2 &


H2O are used as reducing agents.

160
Chapter 21
Biomolecules

1. Given below are two statements: (d) A protein is formed from amino acids
Statement I: A unit formed by the linked by peptide bonds
attachment of a base to 1' position of sugar 6. The incorrect statement regarding enzymes
in known as nucleoside. is (2022)
Statement II: When nucleoside is linked to (a) Enzymes are biocatalysts.
phosphorous acid at 5' position of sugar (b) Like chemical catalysts enzymes reduce
moiety, we get nucleotide. the activation energy of bio processes.
In the light of the above statements, choose (c) Enzymes are polysaccharides.
the correct answer from the options given (d) Enzymes are very specific for a
below: (2023) particular reaction and substrate.
(a) Both Statement I and Statement II are 7. The RBC deficiency is deficiency disease of:
false. (2021)
(b) Statement I is true but Statement II is (a) Vitamin B6
false. (b) Vitamin B1
(c) Statement I is false but Statement II is (c) Vitamin B2
true. (d) Vitamin B12
(d) Both Statement I and Statement II are
8. Which of the following is a basic amino
true.
acid? (2020)
2. Cheilosis occurs due to deficiency of _______
(b) Tyrosine
(a) thiamine (c) Lysine
(b) nicotinamide
(d) Serine
(c) pyridoxamine
(d) riboflavin 9. Sucrose on hydrolysis gives: (2020)
(a) 𝛼-D-Glucose + 𝛽-D-Glucose
3. Which amongst the following is used in
(b) 𝛼-D-Glucose + 𝛽-D-Fructose
controlling depression and hypetension?
(c) 𝛼-D-Fructose + 𝛽-D-Fructose
(2023)
(d) 𝛽-D-Glucose + 𝛼-D-Fructose
(a) Seldane
(b) Valium 10. Deficiency of which vitamin causes
(c) Equanil osteomalacia? (2020 Covid Re-NEET)
(d) Prontosil (a) Vitamin D
(b) Vitamin K
4. Some tranquilizers are listed below. Which
(c) Vitamin E
one from the following belongs to
(d) Vitamin A
barbiturates? (2023)
(a) Meprobamate 11. Which of the following statement is not true
(b) Valium about glucose? (2020 Civid Re-NEET)
(c) Veronal (a) It contains five hydroxyl groups
(d) Chlordiazepoxide (b) It is a reducing sugar
(c) It is an aldopentose
5. The incorrect statement about denaturation
(d) It is an aldohexose
of proteins is: (2022)
(a) Uncoiling of the helical structure takes 12. The non-essential amino acid among the
place following is: (2019)
(b) It results due to change of temperature (a) Valine
and/or pH (b) Leucine
(c) It results in loss of biological activity of (c) Alanine
proteins (d) Lysine

161
13. Which of the following compounds can form
a zwitterion? (2018)
(a) Aniline
(b) Acetanilide
(c) Glycine
(d) Benzoic acid
14. The difference between amylose and
amylopectin is (2018)
(a) L-erythrose, L-threose, L-erythrose, D-
(a) Amylopectin have 1 → 4 𝛼-linkage and 1
threose
→ 6 𝛼 -linkage
(b) D-threose, L-erythrose, L-threose, L-
(b) Amylose have 1 → 4 𝛼-linkage and 1 → 6
erythrose
𝛽 -linkage
(c) L-erythrose, L-threose, L-erythrose, D-
(c) Amylose is made up of glucose and
threose
galactose
(d) L-erythrose, D-threose, L-erythrose, L-
(d) Amylopectin have 1 → 4 𝛼-linkage and 1
threose
→ 6 𝛽 -linkage
20. The correct statement regarding RNA and
15. Which of the following amino acid is not
DNA, respectively is: (2016-I)
optically active? (2017-Gujarat) (a) The sugar component in RNA is 2’-
(a) Glycine deoxyribose and the sugar component
(b) Proline
in DNA is arabinose
(c) Serine
(b) The sugar component in RNA is
(d) Leucine
arabinose and the sugar component in
16. Which of the following statements is not DNA is 2’-deoxyribose
correct? (2017-Gujarat) (c) The sugar component in RNA is ribose
(a) Denaturation makes the proteins more and the sugar component in DNA is 2’-
active deoxyribose
(b) Insulin maintains sugar level in the (d) The sugar component in RNA is
blood of a human
arabinose and the sugar component in
(c) Ovalbumin is a simple food reserve in
egg-white DNA is ribose
(d) Blood proteins thrombin and fibrinogen 21. Which one given below is a non-reducing
are involved in blood clotting sugar? (2016-I)
(a) Sucrose
17. The letter ‘D’ in D-glucose signifies: (b) Maltose
(2017-Gujarat) (c) Lactose
(a) Configuration at the penultimate Chiral (d) Glucose
Carbon
(b) Configuration at all Chiral Carbons 22. In a protein molecule various amino acids
(c) Dextrorotatory are linked together by: (2016-I)
(d) That it is a monosaccharide (a) Dative bond
(b) 𝛼-glycosidic bond
18. The central dogma of molecular genetics
(c) 𝛽-glycosidic bond
states that the genetic information flows
(d) Peptide bond
from: (2016-II)
(a) Amino acids → Proteins → DNA 23. D(+) glucose reacts with hydroxyl amine
(b) DNA → Carbohydrates → Proteins and yields and oxime. The structure of the
(c) DNA → RNA → Proteins oxime would be: (2014)
(d) DNA → RNA → Carbohydrates
19. The correct corresponding order of names of
four aldoses with configuration given below:
(2016-II)

162
(a)
(d)
24. Which of the following hormones is
produced under the conditions of stress
which stimulate glycogenolysis in the liver
of human beings? (2014)
(a) Thyroxin
(b) Insulin
(c) Adrenaline
(b) (d) Estradiol

(c)

163
Answer Key
S1. Ans. (b) S14. Ans. (a)

S2. Ans. (d) S15. Ans. (a)

S3. Ans. (c) S16. Ans. (a)

S4. Ans. (b) S17. Ans. (a)

S5. Ans. (d) S18. Ans. (c)

S6. Ans. (c) S19. Ans. (d)

S7. Ans. (d) S20. Ans. (c)

S8. Ans. (c) S21. Ans. (a)

S9. Ans. (b) S22. Ans. (d)

S10. Ans. (a) S23. Ans. (c)

S11. Ans. (c) S24. Ans. (c)

S12. Ans. (c)

S13. Ans. (c)

164
Solutions
S1. Ans.(b) Enzymes are complex nitrogenous
organic compounds which are produced
by living plants and animals. They are
protein molecules of high molecular
mass. They are not polysaccharides.
S7. Ans.(d)
Vitamin-B12
Base link with 1' position of sugar in S8. Ans.(c)
nucleoside so statement I is correct.

Basic amino acid: Lysine,


Neutral Amino acids: Alanine, serine,
tyrosine
S9. Ans.(b)
Sucrose is composed of one mole of 𝛼-D-
glucose and 𝛽-D-fructose, connected by
⟶ When nucleoside in linked to glycosidic linkage at 1 and 4 carbon
phosphoric acid at 5' position of sugar atoms of glucose and fructose
moiety we get a nucleotide. respectively.
Statement II is incorrect because not link When sucrose is hydrolysed by HCl in
with phosphorous acid. alcoholic medium, mixture of 𝛼-D-
glucose and 𝛽-D-fructose is obtained.
S2. Ans.(d)
Cheilosis (Fissuring at corners of mouth S10. Ans.(a)
and lips) occurs due to deficiency of
Osteomalacia is a vitamin-D deficiency
vitamin B2 (Riboflavin)
disease, in which bone density decrease
S3. Ans.(c) and bones get softer.
Equanil is used in controlling depression S11. Ans.(c)
and hypertension.
S4. Ans.(b)
Veronal is an example of barbiturates.
S5. Ans.(d)
∎ Proteins are polymers of α-amino acids
and they are connected to each other by
peptide bond, but this is not
denaturation process. Vitamin (also known as dextrose) is a
carbohydrate compound consisting of six
∎ Due to denaturation globules unfold carbon atoms and an aldehyde group
and helix get uncoiled and protein loses and known as aldohexose.
its biological activity. There are 5–OH groups are present.
Denaturation can be caused if protein in It is a reducing sugar as it contains –CHO
its native form, is subjected to physical group.
change like change in temperature or S12. Ans.(c)
chemical change like change in pH. Valine, Leucine and Lysine are essential
S6. Ans.(c) amino acids.
Alanine are one of the non-essential
amino acid.
165
S13. Ans.(c) The letter “D” in D-glucose signifies the
configuration at the penultimate chiral
carbon.
S20. Ans.(c)

S14. Ans.(a)
S21. Ans.(d)
Amylose and Amylopectin are polymers
of 𝛼-D-glucose, so 𝛽 -link is not possible.
Amylose is linear with 1 → 4 𝛼 -linkage
whereas Amylopectin is branched and
has both 1 → 4 and 1→ 6 𝛼 -linkages.
So option (a) should be the correct
option.
S15. Ans.(a)
Glycine is not optically active as it does
not have chiral carbon (H2N–CH2–COOH)
S16. Ans.(d)
Denaturation will not make protein more
active, but it can denatured secondary, S22. Ans.(c)
tertiary or quaternary structure of
protein but does not affect primary RNA (ribonucleotide acid) have ribose as
structure of the protein. sugar moiety, whereas DNA (

S19. Ans.(a)
deoxyribonucleotide acid) have 2–
deoxyribose sugar.
S17. Ans.(a)
Sucrose is a dissacharide, a non-
reducing sugar as the anomeric carbons
which are responsible for reduction are
bonded to each other.
S18. Ans.(d)
In protein molecule the adjacent amino
acids are linked via peptide linkages
S24. Ans.(c)
Adrenaline hormone helps to release
D amide bond.
fatty acids from fat and glucose from liver
glycogen under the condition of stress.

S23. Ans.(c)

166
Chapter 12
The p-Block Elements (Part – B)

1. Match List I with List II: (a) Both Statement I and Statement II are
List I List II correct.
(Oxoacids of sulphur) (Bonds) (b) Both Statement I and Statement II are
Peroxodisulphuric 1. Two S-OH, incorrect.
A. (c) Statement I is correct but Statement II
acid Four S=O,
One S-O-S is incorrect.
(d) Statement I is incorrect but Statement II
B. Sulphuric acid 2. Two S-OH, is correct.
One S=O
4. Noble gases are named because of their
C. Pyrosulphuric 3. Two S-OH, inertness towards reactivity. Identify an
acid Four S=O, incorrect statement about them. (2021)
One S-O-O-S (a) Noble gases have very high melting and
boiling points
D. Sulphurous acid 4. Two S-OH,
Two S=O (b) Noble gases have weak dispersion forces
(c) Noble gases have large positive values of
Choose the correct answer from the options electron gain enthalpy
given below: (2023) (d) Noble gases are sparingly soluble in
(a) A-3, B-4, C-1, D-2 water
(b) A-1, B-3, C-4, D-2 5. Statement-I : Acid strength increases in
(c) A-3, B-4, C-2, D-1 the order given as HF << HCl << HBr << HI.
(d) A-1, B-3, C-2, D-4 Statement-II : As the size of the elements
2. Which of the following reactions is part of F, Cl, Br, I increases down the group, the
the large-scale industrial preparation of bond strength of HF, HCl, HBr and HI
nitric acid? (2022) decreases and so the acid strength
Pt increases.
(a) Cu(NO3)2 + 2NO2 + 2H2O →
500 K,9 bar In the light of the above statements, choose
4HNO3 + Cu the correct answer from the options given
Pt
(b) NaNO3 + H2SO4 → NaHSO4 + below. (2021)
500 K,9 bar
(a) Both statement I and statement II are
HNO3
Pt
false
(c) 4NH3 + 5O2(from air) → 4NO + (b) Statement I in correct but statement II
500 K,9 bar
6H2O is false
Pt (c) Statement I is incorrect but statement II
(d) 4HPO3 + 2N2O5 → 4HNO3 + P4O10 is true
500 K,9 bar
(d) Both statement I and statement II are
3. Given below are two statements
Statement I: The boiling points of the true
following hydrides of group 16 elements 6. In which one of the following arrangements
increases in the order- the given sequence is not strictly according
H2O < H2S < H2Se < H2Te to the properties indicated against it?
Statement II: The boiling points of these (2021)
hydrides increase with increase in molar (a) H2O < H2S < H2Se < H2Te : Increasing
mass. pKa values
In the light of the above statements, choose (b) NH3 < PH3 < AsH3 < SbH3 : Increasing
the most appropriate answer from the acidic character
options given below: (2022) (c) CO2 < SiO2 < SnO2 < PbO2 : Increasing
oxidizing power
167
(d) HF < HCl < HBr < HI : Increasing acidic (d) A-(ii), B-(iii), C-(i), D-(iv)
strength 13. Match the Xenon compounds in Column-I
7. Urea reacts with water to form A which will with its structure in Column-II and assign
decompose to form B. B when passed the correct code: (2019)
through Cu2+ (aq), deep blue colour solution Column I Column II
C is formed. What is the formula of C from
the following? (2020) A. XeF4 (i) Pyramidal
(a) [𝐶𝑢(𝑁𝐻3 )4 ]2+
B. XeF6 (ii) Square planar
(b) 𝐶𝑢(𝑂𝐻)2 XeOF4 (iii) Distorted
C.
(c) 𝐶𝑢𝐶𝑂3 ∙ 𝐶𝑢(𝑂𝐻)2 octahedral
(d) 𝐶𝑢𝑆𝑂4
D. XeO3 (iv) Square pyramidal
8. Which of the following oxoacid of sulphur
has – O – O – linkage? (2020) Code :
(a) H2SO4, sulphuric acid A B C D
(b) H2S2O8, peroxodisulphuric acid (a) (i) (ii) (iii) (iv)
(c) H2S2O7, pyrosulphuric acid (b) (ii) (iii) (iv) (i)
(d) H2SO3, sulphorous acid (c) (ii) (iii) (i) (iv)
(d) (iii) (iv) (i) (ii)
9. The reaction of concentrated sulphuric acid
with carbohydrates (C12H22O11) is an 14. Which is the correct thermal stability order
example of (2020 Covid Re-NEET) for H2E (E = O, S, Se, Te and Po)? (2019)
(a) Oxidation (a) 𝐻2 𝑆 < 𝐻2 𝑂 < 𝐻2 𝑆𝑒 < 𝐻2 𝑇𝑒 < 𝐻2 𝑃𝑜
(b) Reduction (b) 𝐻2 𝑂 < 𝐻2 𝑆 < 𝐻2 𝑆𝑒 < 𝐻2 𝑇𝑒 < 𝐻2 𝑃𝑜
(c) Sulphonation (c) 𝐻2 𝑃𝑜 < 𝐻2 𝑇𝑒 < 𝐻2 𝑆𝑒 < 𝐻2 𝑆 < 𝐻2 𝑂
(d) Dehydration 𝐻2 𝑆𝑒 < 𝐻2 𝑇𝑒 < 𝐻2 𝑃𝑜 < 𝐻2 𝑂 < 𝐻2 𝑆

10. Which one of the following reactions does 15. Match the following: (2019)
not come under hydrolysis type reaction? Column I Column II
(2020 Covid Re-NEET) Pure (i) Chlorine
A.
(a) 𝐿𝑖3 𝑁(𝑠) + 3𝐻2 𝑂(𝑙) → 𝑁𝐻3 (𝑠) + 3𝐿𝑖𝑂𝐻(𝑎𝑞) nitrogen
(b) 2𝐹2 (𝑔) + 2𝐻2 𝑂(𝑙) → 4𝐻𝐹(𝑎𝑞) + 𝑂2 (𝑔) Haber (ii) Sulphuric acid
B.
(c) 𝑃4 𝑃10 (𝑠) + 6𝐻2 𝑂(𝑙) → 4𝐻3 𝑃𝑂4 (𝑎𝑞) process
(d) 𝑆𝑖𝐶𝑙4 (𝑙) + 2𝐻2 𝑂(𝑙) → 𝑆𝑖𝑂2 (𝑠) + 4𝐻𝐶𝑙(𝑎𝑞) Contact (iii) Ammonia
C.
11. Among the compounds shown below which process
one revealed a linear structure? D. Deacon’s (iv) Sodium azide or
(2020 Covid Re-NEET) process Barium azide
(a) HOCl
Which of the following is the correct option?
(b) O3
A B C D
(c) N2O
(a) (i) (ii) (iii) (iv)
(d) NO2
(b) (ii) (iv) (i) (iii)
12. Match the compounds of Xe in column I (c) (iii) (iv) (ii) (i)
with the molecular structure in column II. (d) (iv) (iii) (ii) (i)
(2020 Covid Re-NEET)
16. The correct order of N-compounds in its
Column I Column II
decreasing order of oxidation states is
A. XeF2 (i) Square planar (2018)
XeF4 (ii) Linear (a) HNO3, NO, N2, NH4Cl
B.
(b) HNO3, NO, NH4Cl, N2
C. XeO3 (iii) Square pyramidal
(c) NH4Cl, N2, NO, HNO3
D. XeOF4 (iv) Pyramidal (d) HNO3, NH4Cl, NO, N2
(a) A-(ii), B-(i), C-(iv), D-(iii) 17. Which of the following statements is not
(b) A-(ii), B-(i0, C-(iii), D-(iv) true for halogens? (2018)
(c) A-(ii), B-(iv), C-(iii), D-(i) (a) All form monobasic oxyacids

168
(b) All are oxidizing agents 23. Which of the following absorbs carbon
(c) Chlorine has the highest electron-gain dioxide and releases oxygen?
enthalpy (2017-Gujarat)
(d) All but fluorine show positive oxidation (a) K2O
states (b) CaO
18. In thew structure of ClF3, the number of (c) KO2
lone pair of electrons on central atom ‘Cl’ is (d) KOH
(2018) 24. Which of the following pairs shows highest
(a) One bond dissociation enthalpy among halogens
(b) Two and lowest bond dissociation enthalpy
(c) Three among hydrogen halides?
(d) Four (2017-Gujarat)
19. Which oxide of nitrogen is not a common (a) I2, HI
pollutant introduced into the atmosphere (b) F2, HF
both due to natural and human activity? (c) Cl2, HF
(2018) (d) Br2, HBr
(a) N2O5 25. Strong reducing behaviour of H3PO2 is due
(b) NO2 to : (2017-Gujarat)
(c) NO (a) Low coordination number of P
(d) N2O (b) Low oxidation state of P
20. Which of the following pairs of compounds (c) Presence of one – OH group and two P –
is isoelectronic and isostructural? H bonds
(2017-Delhi) (d) Presence of two – OH groups and one P
(a) 𝐼𝐹3 , 𝑋𝑒𝐹2
(b) 𝐵𝑒𝐶𝑙2 , 𝑋𝑒𝐹2
(c) 𝑇𝑒𝑙2 , 𝑋𝑒𝐹2 26. Among halogens, the one which can oxidise
(d) 𝐼𝐵𝑟2− , 𝑋𝑒𝐹2 water to oxygen is : (2017-Gujarat)
(a) Iodine
21. Match the interhalogen compounds of (b) chlorine
column I with the geometry in column II and (c) Bromine
assign the correct code: (2017-Delhi) (d) Fluorine
Column I Column II
27. The correct geometry and hybridization for
A. 𝑋𝑋′ (i) T-shape XeF4 are : (2016-II)
B. 𝑋𝑋3′ (ii) Pentagonal (a) Planar triangle, sp3d3
bipyramidal (b) Square planar, sp3d2
𝑋𝑋5′ (iii) Linear (c) Octahedral, sp3d2
C. (d) Trigonal bipyramidal, sp3d
D. 𝑋𝑋7′ (iv) Square-pyramidal
28. Match the compound given in column I with
(v) Tetrahedral the hybridization and shape given in
Code: column II and mark the correct option.
A B C D (2016-I)
(a) (iv) (iii) (ii) (i) Column I Column II
(b) (iii) (iv) (i) (ii) XeF6 (i) Distorted
A.
(c) (iii) (i) (iv) (ii) octahedral
(d) (v) (iv) (iii) (ii)
B. XeO3 (ii) Square planar
22. In which pair of ions both the species XeOF4 (iii) Pyramidal
contain S–S bond? (2017-Delhi) C.
2−
(a) 𝑆4 𝑂6 , 𝑆2 𝑂7 2− D. XeF4 (iv) Square pyramidal
(b) 𝑆2 𝑂72− , 𝑆2 𝑂32− Code:
(c) 𝑆4 𝑂62− , 𝑆2 𝑂32− A B C D
(d) 𝑆2 𝑂72− , 𝑆2 𝑂82− (a) (iv) (i) (ii) (iii)
169
(b) (i) (iii) (iv) (ii) (c) There is strong hydrogen bonding
(c) (i) (ii) (iv) (iii) between HF molecules
(d) (iv) (iii) (i) (ii) (d) The bond energy of HF molecules is
29. Which one of the following orders is correct greater than in other hydrogen halides
for the bond dissociation enthalpy of 35. In which of the following pairs, both the
halogen molecules? (2016-I) species are not isostructural? (2015 Re)
(a) F2 > Cl2 > Br2 > I2 (a) XeF4, XeO4
(b) I2 > Br2 > Cl2 > F2 (b) SiCl4, PCl+
4
(c) Cl2 > Br2 > F2 > I2 (c) Diamond, silicon carbode
(d) Br2 > I2 > F2 > Cl2 (d) NH3, PH3

30. Among the following, the correct order of 36. Which of the statements given below is
acidity is: (2016-I) incorrect? (2015 Re)
(a) HClO4 < HClO2 < HClO < HClO3 (a) O3 molecule is bent
(b) HClO3 < HClO4 < HClO2 < HClO (b) ONF is isoelectronic with O2N–
(c) HClO < HClO2 < HClO3 < HClO4 (c) OF2 is an oxide of fluorine
(d) HClO2 < HClO < HClO3 < HClO4 (d) Cl2O7 is an anhydride of perchloric acid

31. Which is the correct statement for the given 37. Nitrogen dioxide and sulphur dioxide have
acids? (2016-I) some properties in common. Which
(a) Phosphinic acid is a diprotic acid while property is shown by one of these
phosphonic acid is a monoprotic acid compounds, but not by the other? (2015)
(b) Phosphinic acid is a monoprotic acid (a) Is soluble in water
while phosphonic acid is a diprotic acid (b) Is used as a food preservtive
(c) Both are diprotic acids (c) Forms ‘acid-rain’
(d) Both are triprotic acids
32. When copper is heated with conc. HNO3 it 38. Acidity of diprotic acids in aqueous
produces: (2016-I) solutions increases in the order: (2014)
(a) Cu(NO3)2 and N2O (a) H2Se < H2S < H2Te
(b) Cu(NO3)2 and NO2 (b) H2Te < H2S < H2Se
(c) Cu(NO3)2 and NO (c) H2Se < H2Te < H2S
(d) Cu(NO3)2, NO and NO2 (d) H2S < H2Se < H2Te

33. Strong reducing behaviour of H3PO2 is due 39. XeF2 is isostructural with: (2013)
to : (2015 Re) (a) TeF2
(a) Presence of two –OH groups and one P– (b) ICl−
2
H bond (c) SbCl3
(b) Presence of one –OH group and two P–H (d) BaCl2
bonds 40. Which of the following does not give oxygen
(c) High electron gain enthalpy of on heating? (2013)
phosphorus (a) KClO3
(d) High oxidation state of phosphorus (b) Zn(ClO3)2
34. The variation of the boiling points of the (c) K2Cr2O7
hydrogen halides is in the order HF > HI > (d) (NH4)2Cr2O7
HBr > HCl. What explains the higher boiling 41. Which is the strongest acid in the following?
point of hydrogen fluoride? (2015 Re) (2013)
(a) The effect of nuclear shielding is much (a) H2SO4
reduced in fluorine which polarizes the (b) HClO3
HF molecule (c) HClO4
(b) The electronegativity of fluorine is much (d) H2SO3
higher than for other elements in the
group

170
Answer Key
S1. Ans. (a) S26. Ans. (d)

S2. Ans. (c) S27. Ans. (c)

S3. Ans. (b) S28. Ans. (b)

S4. Ans. (a) S29. Ans. (c)

S5. Ans. (d) S30. Ans. (c)

S6. Ans. (a) S31. Ans. (b)

S7. Ans. (a) S32. Ans. (b)

S8. Ans. (b) S33. Ans. (b)

S9. Ans. (d) S34. Ans. (c)

S10. Ans. (b) S35. Ans. (a)

S11. Ans. (c) S36. Ans. (c)

S12. Ans. (a) S37. Ans. (b)

S13. Ans. (b) S38. Ans. (d)

S14. Ans. (c) S39. Ans. (b)

S15. Ans. (d) S40. Ans. (d)

S16. Ans. (a) S41. Ans. (c)

S17. Ans. (a)

S18. Ans. (b)

S19. Ans. (a)

S20. Ans. (d)

S21. Ans. (c)

S22. Ans. (c)

S23. Ans. (c)

S24. Ans. (c)

S25. Ans. (c)

171
Solutions
S1. Ans. (a) S6. Ans.(a)
Order of pKa is H2O > H2S > H2Se > H2Te
S7. Ans.(a)

S8. Ans.(b)

Peroxodisulphuric acid H2S2O8, has –O–


O– linkage
S2. Ans. (c) Ans.(d)
Pt
4NH3 + 5O (from air) → 4NO + Concentrated sulphuric acid is a strong
500 K,9 bar
6H2O dehydrating agent and it readily
dehydrate carbohydrates into carbon
2NO(g) + O2(g) ⇌ 2NO2(g)
2𝐶12 𝐻22 𝑂11 + 𝑐𝑜𝑛𝑐. 𝐻2 𝑆𝑂4 (𝑐𝑜𝑛𝑐) →
3NO2(g) + H2O(l) ⟶ 2HNO3(aq) + NO(g)
This is industrial method of preparation 12𝐶 + 11𝐻2 𝑂
of nitric acid. The reaction of concentrated sulphuric
S3. Ans. (b) acid with carbohydrates (C12H22O11) is
an example of dehydration.
Compound Boiling point (K)
S10. Ans.(b)
H 2O 373
Reaction of F2 with H2O gives products
H 2S 213
HF(aq) and O2(g) in which fluorine
H2Se 232 oxidizes water into oxygen that does not
H2Te 269 come under hydrolysis type reaction.
• The boiling points of these hybrids not S11. Ans.(c)
exactly with increase in molar mass. N2O has linear, unsymmetrical
• H2O has maximum boiling point due structure
to intermolecular hydrogen bonding.
S4. Ans.(a)
Noble gases have weak dispersion forces
so they have low melting and boiling
points.
S5. Ans.(d)
The correct order of acidic strength is
HI > HBr > HCl > HF
172
S12. Ans.(a) S16. Ans.(a)

Hence, the correct option is (a).


S17. Ans.(a)
All halogens show +ve and –ve oxidation
state while F show –ve oxidation state.
Due to high electronegativity and small
size, F forms only one oxoacid, HOF
known as Fluoric (I) acid. Oxidation
number of F is +1 in HOF.
S18. Ans.(b)
S13. Ans.(b) The structure of ClF3 is

The number of lone pair of electrons on


central Cl is 2.
S19. Ans.(a)

S20. Ans.(d)

S14. Ans.(c)
Thermal stability order for H2E decrease S21. Ans.(c)
down the group because H–E bond
energy decreases on going down the
group.
∴ Order of stability would be :
H2Po < H2Te < H2Se < H2S < H2O
S15. Ans.(d)
(a) Pure nitrogen : Sodium azide or
Barium azide
(b) Haber : Ammonia
(c) Contact processs : Sulphuric acid
(d) Deacon’s process : Chlorine

173
S22. Ans.(c) S29. Ans.(c)
Bond dissociation energy of halogen
family decreases down the group as the
size of the atom increases. The bond
S23. Ans.(c) dissociation energy of fluorine is,
KO2 absorb CO2 and release oxygen. however, lower than that of chlorine and
bromine because of inter electronic
2𝐾𝑂2 + 𝐶𝑂2 → 𝐾2 𝐶𝑂3 + 3/2𝑂2 repulsions present in the small atom of
S24. Ans.(c) fluorine.
Cl2 : Have highest bond dissociation
enthalpy because Cl2 > Br2 > F2 > I2 bond Hence bond energy decreases in the
dissociation enthalpy. order:
HF : Have highest bond dissociation 𝐶𝑙2 > 𝐵𝑟2 > 𝐹2 > 𝐼2
enthalpy because
1 S30. Ans.(c)
Bond dissociation enthalpy ∝ Bond length
Correct order of acidity among oxo –
S25. Ans.(c)
acids of Cl is HClO4 > HClO3 > HClO2 >
HClO because the oxidation number of
central atom increases, acidic nature
increases.
Strong reducing behaviour of H3PO2 is S31. Ans.(b)
due to presence of 1 –OH group and 2 Phosphinic acid (Hypophosphorous acid)
P–H group. is a monoprotic acid (H3PO3).
S26. Ans.(d)
Fluorine is the most electronegative
element of the periodic table. Therefore,
it can oxidise water to oxygen.
While phosphonic acid –
2𝐹2 + 2𝐻2 𝑂 → 4𝐻𝐹 + 𝑂2 .
is a diprotic acid.
S27. Ans.(c)
S32. Ans.(b)
𝐶𝑢 + 𝐻𝑁𝑂3 (𝑐𝑜𝑛𝑐. ) → 𝐶𝑢(𝑁𝑂3 )2 +
(𝐵𝑟𝑜𝑤𝑛 𝑔𝑎𝑠)

2𝑁𝑂2 + 𝐻2 𝑂
XeF4 is a AB4L2 type molecule with 4 –
bond pair and 2 lone pair. Octahedral With HNO3 (dil) gives NO gas.
geometry and shape is square planar
with hybridization sp3d2. S33. Ans.(b)
S28. Ans.(b) In H3PO2, presence of 2H makes H3PO2
a reducing agent.

S34. Ans.(c)
Fluorine forms strongest hydrogen bond
among all halogens.

174
S35. Ans.(a) S41. Ans.(c)
HClO4 is the strongest acid, as it has
greater number of ‘O’ atoms, so more e–s
will be pulled away from O–H bond and
more this bond will be weakened.
Between H2SO4 and HClO4, HClO4 is
strong because perchlorate ion formed
by removal of hydrogen atom is more
stabilized than sulphate ion as negative
charge is more dispersed in perchlorate
S36. Ans.(c) ion.
OF2 (oxygen difluoride) is a fluoride of
oxygen because fluorine is more
electronegative than oxygen.
S37. Ans.(b)
NO2 is not used as a food preservative.

S38. Ans.(d)
Order for acidity is H2S < H2Se < H2Te,
as we move down the group atomic
radius of atom increases because of
which size also increases and bond
dissociation enthalpy decreases and
such atoms can easily furnish H in
aqueous medium.
S39. Ans.(b)
XeF2 is isostructural with ICl−
2

Sp3d hybridization and linear geometry


S40. Ans.(d)
Heating of ammonium dichromate yields
nitrogen:
It is a laboratory preparation for
nitrogen.

(𝑁𝐻4 )2 𝐶𝑟2 𝑂7 → 𝑁2 + 𝐶𝑟2 𝑂3 + 𝐻2 𝑂

2𝐾𝐶𝑙𝑂3 → 2𝐾𝐶𝑙 + 3𝑂2

𝑍𝑛(𝐶𝑙𝑂3 )2 → 𝑍𝑛𝐶𝑙2 + 3𝑂2
∆ 3
𝐾2 𝐶𝑟2 𝑂7 → 2𝐾2 𝐶𝑟𝑂4 + 𝐶𝑟2 𝑂2 + 2 𝑂2

175

You might also like